Download as pdf or txt
Download as pdf or txt
You are on page 1of 326

You can join our FREE test series at Click here

You can also join telegram channel for latest updates

Telegram Exam Group (For Updates on UPSC)

Telegram UGC NET Channel (UGC Material)

UGC NET Group (To Meet Other Aspirants and Free Test Series)

UPSC NET Group (To Meet Other Aspirants and Free Test
Series)

You can also enjoy our Youtube videos on our channel.

Don’t Forget to rate us on Amazon and claim a Free UPSC or UGC


(Any One) test series worth ₹ 1999 on our telegram group given
above.
Index

1 Q) Which of the following pairs are correctly matched?


51 Q) Which of the following age is demarcated to be qualified as a
member of Rajya Sabha?
101 Q) Which of the following is/are true
151 Q) Which of the following state was never a union territory ?
201 Q) Which of the following does not participate in the elections of
President?
251 Q) Which of the following is/are true?
301 Q) Which of the following is/are true?
351 Q) Which of the following is/are true?
401 Q) Which of the following is/are true?
451 Q) how many sub rights are given under article 19?
500 Q) How many proposers and seconders are required for the nomination
of a candidate in Vice presidential election?

INDIAN POLITY
PART -II

1 Q) How many members Committee on Papers Laid on the


Table consists?
51 Q) Which of the following term is used in state government?
101 Q) In which one of the following cases Judicial Review is not used?
151 Q) Which of the following appoints Auditor General?
201 Q) Who acts as a vice-chairman of the concerned zonal council by
rotation?
251 Q) To be recognised as a national party, in how many states it has to be
recognised?
301 Q) Consider the following statements regarding the National Human
Rights Commission of India:
330 Q) Given below are two statements, one labelled as Assertion (A) and
the other labelled as Reason (R):

PREFACE

The book contains 500 Multiple choice questions on issues of Indian


Polity . The objective is to provide relevant MCQs to the aspirants who
are preparing for competitive exams and want to excel in Indian Polity
section for UGC NET/SLET exam.

The questions are followed by answers and detailed explanations which


would further enrich your knowledge. On certain places in the book,
links are also provided which would take you to the website Click Here
so that you could practice MCQs for Polity, Geography, Ecology,
aptitude and Mock Tests. You would also get a vault for current affairs
which is updated regularly.

For any query, you can reach us here and ask any of your exam related
doubts.

Happy Reading!

Copyright

Written by: DemyNotes

Copyright © 2017
All Rights Reserved

All rights reserved. No part of the book may be reproduced or copied in


any form or by any means [graphic, electronic or mechanical, including
photocopying, recording, taping or information retrieval systems] or
reproduced on any disc, tape, perforated media or other information
storage device, etc., without the explicit written permission of the editor.
Breach of the condition is liable for legal action. However, the
permission to reproduce this material does not extend to any material on
this site, which is explicitly identified as being the copyright of a third
party. Authorization to reproduce such material must be obtained from
the copyright holders concerned.

Warning-Disclaimer

Though every effort has been made to avoid errors or omissions in this
site, errors might creep in. Any mistake, error or discrepancy noted by
the readers may be brought to the notice of the editor along with
evidence of it being incorrect. All such errors shall be corrected at the
earliest preferably within seven days. It is notified that neither the
content provider nor the editor or any person related with the book in any
manner shall be responsible for any damage or loss of action to anyone,
of any kind, in any manner, therefrom. It is suggested that to avoid any
doubt, the readers should cross-check all the facts, law and contents of
the publication with the original Government publication or
notifications.

Your questions, suggestions and feedback are welcome at:


netmockprep@gmail.com
Dedication

This book is dedicated to my parents who have always motivated me to


enjoy the state of blissful dissatisfaction.
1) Which of the following pairs are correctly
matched?
Feature of Indian constitution borrowed from countries
1. Council of Ministers responsible to Lower House UK
2. Executive head of state known as President and his being the U.S.
Supreme Commander of the Armed Forces
3. Vice- President as the ex-officio Chairman of Rajya Sabha U.S.
4. Five year plan Russia
a) 1,2 and 3
b) 1,2,3 and 4
c) 2,3 and 4
d) None
Correct Answer is B (All are correct )

2) Consider the following statements with respect to lapsing of the bill...


1. A bill pending in Lok Sabha, which originated in Lok Sabha
lapses
2. A bill pending in Lok Sabha, which is given to it by Rajya Sabha
also lapses
3. A bill passed by Lok Sabha but pending in Rajya Sabha lapses
Which of the above correctly matched?
(a) 1 and 2
(b) 3 only
(c) 1, 2 and 3
(d) None
Correct Answer is C (When the Lok Sabha is dissolved, all business
including bills, resolutions, motions , notices, petitions and so on pending
before it lapse. Such Bills must be reintroduced in the newly-constituted
Lok Sabha to be pursued further.)

3) Which statement/statements is/are correct?


1. Adjournment terminates the sitting and not the session; However,
on the contrary, prorogation terminates the sitting and the session
both
2. Adjournment is done by the presiding officer while prorogation is
done by President
3. Adjournment does affect the bills or any other pending business,
While prorogation does not affect the bills, but the pending notices
lapse
(a) 1 and 2
(b) 2 only
(c) 2 and 3
(d) 3 only
Correct Answer is A (Adjournment does not affect the bills or any other
pending business, While prorogation does not affect the bills, but the
pending notices lapse)

4) What is the number determined for the quorum?


(a) 1/5 of the total number in each house
(b) 1/10th of the total number in each house
(c) 1/10th of the total number in each house including the
presiding officer
(d) 1/5th of the total number in each house, including
the presiding Officer
Correct Answer is C (1/10th of the total number in each house including the
presiding officer.)

5) Which statement/statements is/are correct?


1. Constitution has declared English as the languages for
transacting business
2. The presiding officer can allow a mother tongue to a member
to address the house
3. Official language act-1963 allowed english language to be
continued with Hindi language
Select the correct answer using the codes given below.
(a) 1, 2 and 3
(b) 2 only
(c) 1 only
(d) 3 only
Correct Answer is A (All are Correct.)

6) Which of the following comes first in the Parliament sitting?


(a) Starred Question
(b) Zero Hour
(c) Agenda
(d) Question hour
Correct Answer is D (Question hour)

7) How many days notices are required in the case of short notice question?
(a) 5 days
(b) 3 days
(c) 20 days
(d) 15 days
Correct Answer is D (15 days’ notice is given in the case of short notice
question.)

8) Which statement/statements is/are correct?


1. A minister in the LokSabha cannot participate in the proceedings
of the Rajya Sabha
2. The members who are not elected in the new Lok Sabha are
called the lame-ducks
Select the correct answer using the codes given below...
(a) 1 only
(b) 2 only
(c) None of the above
(d) Both of the above
Correct Answer is B (A minister in the LokSabha can also participate in the
proceedings of the Rajya Sabha)
9) Which statement/statements is/are correct?
1. Starred questions can be answered orally
2. Unstarred questions are answered in written form
3. Short notice questions are answered in writing
Which of the statements given above is are correct...
(a) only 3
(b) 1 and 2 only
(c) 1, 2 and 3
(d) None
Correct Answer is B (Short notice questions are answered orally.)

10) where does the question hour mention?


(a) Representative of people act
(b) Rules of procedure
(c) Constitution
(d) None of the above
Correct Answer is A (Representative of people act)

11) When does the zero hour come?


(a) After the question hour
(b) After the agenda
(c) Before the question hour
(d) Before the agenda
Correct Answer is A (The time between the question hour and that agenda
is called the zero hour.)

12) Which of the following is/are not a motion:


1. Substantive motion
2. Substitute motion
3. Subsidiary motion
Which of the above is/are correct?
(a) 1 and 2
(b) 3 only
(c) 1, 2 and 3
(d) None
Correct Answer is D (All these motions exist.)

13) Which statement/statements is/are correct?


1. In Half-an-Hour Discussion, the matter of sufficient public
importance are discussed.
2. Speaker can allow three days in a week Half-an-Hour
Discussion.
3. Short Duration Discussion is known as two hour discussion.
(a) 1 and 2
(b) 2 only
(c) 1, 2 and 3
(d) 3 only
Correct Answer is C (All are Correct.)

14) Which motion has ancillary motion, superseding motion an amendment


as the three subcategories?
(a) Subsidiary motion
(b) Substitute motion
(c) Substantive motion
(d) Suspensive Motion
Correct Answer is A (Subsidiary motion.)

15) Which statement/statements is/are correct?


1. Closure motion is used by member to cut short the debate.
2. Simple closure is a type of Closure motion
3. In Kangaroo closure, important clauses are taken for the day
and intervening clauses are skipped
4. In Guillotine closure, the clauses which remained
undiscussed are passed
Select the correct answer using the codes given below.
(a) 1, 2 and 4 only
(b) 2 only
(c) 1,3 and 4 only
(d) 1, 2, 3 and 4
Correct Answer is D (Closure by Compartments: In this case, the clauses
of a bill or a lengthy resolution are grouped into parts before the
commencement of the debate. The debate covers the part as a whole and the
entire part is put to vote..)

16) Where does the calling attention motion mention?


(a) Constitution
(b) Representative People's Act 1951
(c) Representative People's Act 1950
(d) Rules of procedure
Correct Answer is D(Calling attention motion is an Indian invention just
like the zero hour. In this, Issues which are urgent and are of public
importance are discussed.)

17) How many members need to support an adjournment motion so that it


could be admitted?
(a) 10 members
(b) 100 members

(c) 20 members
(d) 50 members
Correct Answer is 50 members (50 members.)

18) Which statement/statements is/are correct?


1. Censure motion should not state the reason of its adoption,
Which is not in the case of no confidence motion
2. Censure motion can be against one minister or more, or even
against the entire council; On the contrary, no-confidence motion
is always against the Council of ministers
3. If the Lok Sabha passes, in censure motion Council of
ministers need not resign; However, in the case of no-confidence
motion Council of ministers must resign
Select the correct answer using the codes given below...
(a) 1 only
(b) 2 and 3 only
(c) 1 and 3 only
(d) 1, 2 and 3
Correct Answer is B (Censure motion should state the reason of its
adoption. Which is not in the case of no-confidence motion.)

19) Which statement/statements is/are correct?


1. No-Day-Yet-Named Motion is a motion that has been admitted
by the President
2. A member can escalate a point of order when the proceedings
of the House do not follow the normal rules of procedure
3. A Debate is allowed on a point of order.
Which of the statements given above is are correct...
(a) only 2
(b) 1 and 2 only
(c) 1, 2 and 3
(d) None
Correct Answer is A (No debate is allowed on a point of order. No-Day-
Yet-Named Motion is a motion that has been admitted by the Speaker.)

20) Who addresses the first session after each general election and
procession every fiscal year?
(a) Prime Minister
(b) The President
(c) Chairman
(d) Speaker
Correct Answer is B (The President)
21) Which of the following pairs are correctly matched?
1 Public Servants Enquiry act 1850
2 Indian Penal Code 1860
3 Delhi Police Establishment Act 1946
4 All India Service conduct rules 1968
a) Only 1
b) None
c) Only 2
d) Both of the Above
D is the Correct Answer (All are correct.)

22) Consider the following statements:


1. Speaker can decide whether bill is money bill or otherwise,
the Chairman cannot do so
2. Speaker can preside over joint sitting which Chairman cannot
do
3. In the case of resignation or removal, speaker can vote on the
first instance, while the chairman cannot do so
Which of the above correctly matched?
(a) 1 and 2
(b) 3 only
(c) 1, 2 and 3
(d) None
Correct Answer is C (All are Correct.)

23) Which statement/statements is/are correct?


1. Speaker is a member of the house
2. Chairman is also a member of the house
3. Deputy chairman is subordinate to chairman
(a) 1 only
(b) 2 only
(c) 2 and 3
(d) 3 only
Correct Answer is A (Chairman is not a member of the house. Deputy
chairman is not subordinate to chairman)

24) Who is called the leader of the house?


(a) President
(b) Speaker
(c) Prime Minister
(d) Leader of opposition
Correct Answer is Prime Minister (In American setup, the leader of the
house is known as Majority Leader.)

25) Which statement/statements is/are correct?


1. The leader of opposition comes from the party having not less
than 1/10th of the seats of the total strength of the house
2. The leader of opposition is known as minority leader in
American setup
3. The concept of Shadow Cabinet is practiced by US political
system
4. The Deputy Prime Minister is sometimes referred as
alternative Prime Minister
Select the correct answer using the codes given below.
(a) 1, 2 and 4 only
(b) 1 and 2 only
(c) 1,3 and 4 only
(d) 1, 2, 3 and 4
Correct Answer is B (The concept of Shadow Cabinet is practiced by
British political system. The leader of opposition is sometimes referred as
alternative Prime Minister)

26) Which of the following is applicable over Lok Sabha only?


(a) Adjournment
(b) Prorogued
(c) Dissolution
(d) Adjournment Sine Die
Correct Answer is C (Dissolution is applicable on Loksabha only.)

27) After how many years, automatic dissolution takes place ?


(a) 1 Year
(b) 2 Years
(c) 5 years
(d) 3 Years
Correct Answer is C (The time period can be extended to the National
emergency is implemented.)

28) Which statement/statements is/are correct?


1. The terminologies such as “Leader of opposition” is not
mentioned in the Constitution of India
2. “Leader of the house” as a terminology is mentioned in the
Constitution of India
3. The “Office of Whip” is not mentioned anywhere in the
Constitution of India
Select the correct answer using the codes given below...
(a) 1 only
(b) 2 and 3 only
(c) 1 and 3 only
(d) 1, 2 and 3
Correct Answer is C (The leader of the house and the leader of
the opposition are not mentioned in the Constitution of India. However,
they are mentioning rules of the house and Parliamentary statute
respectively. Furthermore, the office of the whip is neither mentioned in the
rule of the house nor in the Parliamentary statute. the office of Whip is
merely a convention.)

29) Which statement/statements is/are correct?


1. The maximum gap in two Parliament session can be of
six months.
2. There are usually three sessions in the year.
3. It is said that Parliament should meet at least twice in a year.
Which of the statements given above is are correct...
(a) only 3
(b) 1 and 2 only
(c) 1, 2 and 3
(d) None
Correct Answer is C (The three sessions are Budget session, monsoon
session and the winter session Which are transpired throughout the year.)

30) Which of the following terminates the sitting of Parliament for an


indefinite period?
(a) Adjournment Sine Die
(b) Adjournment
(c) Prorogation
(d) Suspension
Correct Answer is A (Adjournment Sine Die.)

31) The resolution of removal of Speaker is considered when it is


supported by_____ members?
(a) 50 members
(b) 20 members
(c) 100 members
(d) 25 members
Correct Answer is 50 members (50 members)

32) Consider the following statements:


1. The salary and allowances of the speaker are decided by the
Parliament and charged from consolidated fund of India
2. Speaker's working conduct cannot be discussed in the Lok
Sabha in any case
3. Speaker can vote only in the case of tie, he/she cannot vote in
the first instance.
Which of the above correctly matched?
(a) 1 and 3
(b) 3 only
(c) 1, 2 and 3
(d) None
Correct Answer is A (Speaker's working conduct cannot be discussed in the
Lok Sabha, except in the case of substantive motion.)

33) Which statement/statements is/are correct?


1. Speaker is a non-partisan member
2. Speaker is provided with a security of tenure
3. Speaker can be removed by a resolution which is passed by
absolute majority in the Lok Sabha
(a) 1 and 2
(b) 2 only
(c) 1, 2 and 3
(d) 3 only
Correct Answer is C (All are Correct.)

34) In order of precedence, Which of the following is the rank conferred


upon the Speaker?
(a) 5th
(b) 7th
(c) 3rd
(d) 6th
Correct Answer is 7th (In order of precedence, Speaker's rank is 7th.)

35) Which statement/statements is/are correct?


1. Deputy Speaker is elected in Lok Sabha
2. Deputy Speaker can never preside the joint sitting of both the
houses in any case
3. Deputy Speaker is subordinate to the Speaker
4. Deputy speaker cannot vote in the first instance when
presiding over the house
Select the correct answer using the codes given below.
(a) 1 and 4 only
(b) 2 only
(c) 1,3 and 4 only
(d) 1, 2, 3 and 4
Correct Answer is A (Deputy Speaker may preside the joint sitting of both
the houses when the speaker is absent. Deputy Speaker is not subordinate to
the Speaker. When the speaker is presiding then the Deputy Speaker sits
just like any other member of the house.)

36) In which of the following Lok Sabha, it was decided that speaker comes
from the ruling party and Deputy Speaker comes from the opposition party?
(a) 9th Lok Sabha
(b) 10th Lok Sabha
(c) 11th Lok Sabha
(d) 12th Lok Sabha
Correct Answer is C (Up to 10 Lok Sabha, both the speaker and the Deputy
Speaker used to be appointed from the ruling party.)

37) How many maximum number of chairpersons can be nominated by the


Speaker, who could preside in his absence and the absence of Deputy
Speaker?
(a) 5
(b) 10
(c) 15
(d) 20
Correct Answer is B (10 Chairperson can be nominated by the speaker.)

38) Which statement/statements is/are correct?


1. The institution of speaker originated in 1910 under the
provision of Government of India act 1909.
2. Before 1921, Governor-general of India used to preside on
Central legislative Council meetings.
3. In 1921, Fredrick Whyte was the first speaker appointed by
the Governor-general of India.
Select the correct answer using the codes given below...
(a) 1 only
(b) 2 and 3 only
(c) 1 and 3 only
(d) 1, 2 and 3
Correct Answer is B (The institution of speaker originated in 1921 under
the provision of Government of India act 1919)

39) Which statement/statements is/are correct?


1. Ananthasayanam Ayyangar became the first speaker of
Central's legislative assembly.
2. GV Mavlankar was the first speaker of the Lok Sabha.
3. VithalBhai Patel was the first Deputy Speaker of the Lok
Sabha.
Which of the statements given above is are correct...
(a) only 2
(b) 1 and 2 only
(c) 1, 2 and 3
(d) None
Correct Answer is A (VithalBhai Patel became the first speaker of Central's
legislative assembly. Ananthasayanam Ayyangar was the first Deputy
Speaker of the Lok Sabha.)

40) Which of the following was the first Deputy Speaker of Central's
legislative assembly?
(a) Ananthasayanam Ayyangar
(b) Sachidanand Sinha
(c) GV Mavlankar
(d) VithalBhai Patel
Correct Answer is B (Sachidanand Sinha)

41) A member inif sits in the house before taking the prescribed oath or
affirmation then how much penalty he or she is supposed to bear?
(a) 1000
(b) 500
(c) No fine
(d) 10000
Correct Answer is B (A penalty of 500 rupees is imposed on the member.)

42) Consider the following statements:


1. There is no provision of pension for the speaker in the
Constitution.
2. The salary and allowances of Speaker and chairman are
charged from the consolidated fund of India.
3. The salary and allowances of Speaker and chairman are
subject to annual vote of Parliament.
Which of the above correctly matched?
(a) 1 and 2
(b) 3 only
(c) 1, 2 and 3
(d) None
Correct Answer is A(The salary and allowances of Speaker and chairman
are not subject to annual vote of Parliament.)

43) Which statement/statements is/are correct?


1. Speaker can preside in the house when a resolution for his
removal is under consideration.
2. Speaker does not vacate the seat whenever the Lok Sabha
dissolves.
3. The principal spokesman of the houses is the Speaker.
(a) 1 and 2
(b) 2 only
(c) 2 and 3
(d) 3 only
Correct Answer is C (Speaker cannot preside in the house when a resolution
for his removal is under consideration. Speaker remains there till the time
newly Elected Lok Sabha meets.)

44) Who of the following fix the date of election of the Speaker?
(a) The speaker himself
(b) Chairman
(c) The President
(d) Lok Sabha
Correct Answer is C (The President decides the date of election of the
Speaker)

45) Which statement/statements is/are correct?


1. Speaker can allow a secret meeting of the house if the leader
of the house requests so.
2. Speaker of the house has the power to determine whether a
bill is money bill or otherwise
3. Sspeaker decides on the question of disqualification of a
member of Lok Sabha.
4. Decision of the Speaker regarding disqualification of member
does not come under the purview of judicial review.
Select the correct answer using the codes given below.
(a) 1, 2 and 3 only
(b) 2 only
(c) 1,3 and 4 only
(d) 1, 2, 3 and 4
Correct Answer is A (Decision of the Speaker regarding disqualification of
a member comes under the purview of judicial review as
per 10th schedule.)

46) Which of the following preside over a joint sitting which is summoned
by President?
(a) Speaker
(b) President
(c) Prime Minister
(d) Chairman
Correct Answer is Speaker (Speaker himself presides over the joint sitting.)

47) Which of the following is the final interpreter of the provision of


Constitution of India within the house?
(a) Supreme Court
(b) High Court
(c) CJI (Chief Justice of India)
(d) Speaker
Correct Answer is D (Within the house, Speaker is the final interpreter of
the provisions of the Constitution.)

48) Which of the following source or sources a speaker derives his or her
power from?
1. Constitution of India
2. Parliamentary conventions
3. Rules of procedure and conduct of business of Lok Sabha
Select the correct answer using the codes given below...
(a) 1 only
(b) 2 and 3 only
(c) 1 and 3 only
(d) 1, 2 and 3
Correct Answer is D (All the three sources.)

49) Which of the following committee or committees, a Speaker is


chairman of?
1. Business advisory committee
2. The rules committee
3. General-purpose committee
Which of the statements given above is are correct...
(a) only 3
(b) 1 and 2 only
(c) 1, 2 and 3
(d) None
Correct Answer is C (All the aforementioned committees.)

50) Which of the following is the chairman of all the Parliamentary


committees of the Lok Sabha?
(a) President
(b) Speaker
(c) Prime Minister
(d) Minister
Correct Answer is B (Speaker is the chairman of parliamentary committees,
while home Minister is the chairman of Parliamentary affairs committee.)
51) Which of the following age is demarcated to
be qualified as a member of Rajya Sabha?
(a) 25
(b) 30
(c) 21
(d) 35
Correct Answer is 30 (30 years is the age in the case of Rajya Sabha and 25
in the case of Lok Sabha.)

52) Consider the following statements:


1. A candidate contesting election from Rajya Sabha should be
an electoral in that state.
2. If a member is convicted for any offence which caused the
imprisonment for two or more years than he or she will be
disqualified.
3. A candidate can be disqualified if he or she fails to lodge an
account of his or her election expenses.
Which of the above correctly matched?
(a) 1 and 2
(b) 3 only
(c) 2 and 3
(d) None
Correct Answer is C (A candidate contesting election from Rajya Sabha
need not be an electoral in that state. Earlier, it was mandatory for the
candidate to come from the same state. However, this condition was
dispensed in 2003.)

53) Under defection law, when can a candidate be disqualified?


1. If he voluntary gives up membership of the party he fought for.
2. He votes contrary to the direction of the party.
3. Independently elected member joins any political party.
(a) 1 and 2
(b) 2 only
(c) 1, 2 and 3
(d) 3 only
Correct Answer is C (Nominated member joins any political party after the
expiry of six months will also lead to disqualification.)

54) After how many days a seat can be declared vacant if a member
remains absent?
(a) 365 days
(b) 60 days
(c) 90 days
(d) 28 days
Correct Answer is B (However, if the house is prorogued or adjourned for
4 consecutive days, then the same is not counted while computing 60 days.)

55) Which of the following social crime if the candidate is punished for
preaching and practicing, leads to disqualification?
1. Dowry
2. Sati
3. Untouchability
Select the correct answer using the codes given below.
(a) 1, 2 and 3
(b) 2 only
(c) 1 only
(d) 3 only
Correct Answer is A (The person can be disqualified if he or she is
punished for preaching and practicing social crimes such as Sati,
untouchability or dowry.)

56) Which of the following schedule is related to the defection?


(a) 10th schedule
(b) 9th schedule
(c) 8th schedule
(d) 7th schedule
Correct Answer is A (10th schedule)

57) Which of the following authority can declare an election void if


disqualified candidate is elected?
(a) Supreme Court
(b) Speaker
(c) High Court
(d) President
Correct Answer is High Court (Representation of the people act, 1951
enables the High Court to declare an election void in case disqualified
candidate is elected. The aggrieved party can appeal in the supreme court.)

58) Which statement/statements is/are correct?


1. Any individual cannot be a member of the both the house
simultaneously.
2. If anyone is elected to both the houses, he is supposed to
intimate within 10 days about the house he wants to serve.
3. If a sitting member of one house is also elected on the other
then the seat in the first house becomes vacant.
Select the correct answer using the codes given below...
(a) 1 only
(b) 2 and 3 only
(c) 1 and 3 only
(d) 1, 2 and 3
Correct Answer is D (Apart from this, if a person is elected on two seats in
the house, he should choose one. Otherwise both the seats will become
vacant.)

59) Which statement/statements is/are correct?


1. A person cannot be a member of Parliament and State
Legislature simultaneously
2. Elected member in parliament and state legislature gets a time
of 7 days to resign from state legislature
3. If the elected person fails to resign, his seat in the Parliament
becomes vacant
Which of the statements given above is are correct...
(a) only 3
(b) 1 and 3 only
(c) 1, 2 and 3
(d) None
Correct Answer is B (Elected member in parliament and state legislature
gets a time of 14 days to resign from state legislature.)

60) Which of the following authority provides an oath of affirmation to the


member of Parliament?
(a) The President
(b) Prime Minister
(c) Speaker
(d) Chairman
Correct Answer is The President (The President)

61) Which of the following union territory has the representation in Rajya
Sabha?
(a) Puducherry
(b) Dadra and Nagar Haveli
(c) Andaman
(d) Chandigarh
Correct Answer is Puducherry (Only, Puducherry and Delhi have the
representation in Rajya Sabha.)

62) Consider the following statements:


1. The Constitution adopted proportional representation system
in Lok Sabha
2. The Constitution adopted territorial representation system in
Rajya Sabha
3. Single transferable vote system is adopted in the elections of
Rajya Sabha, President, Vice president and Legislative Council.
Which of the above correctly matched?
(a) 1 and 2
(b) 3 only
(c) 1, 2 and 3
(d) None
Correct Answer is B (The Constitution adopted proportional representation
system in Rajya Sabha. The Constitution adopted territorial representation
system in Lok Sabha.)

63) Which statement/statements is/are correct?


1. Rajya Sabha is considered as a continuing chamber, where
1/3rd of the members are retired every second year.
2. The retiring members are not eligible for re-election.
3. In the first batch of Rajya Sabha, a lottery system was used to
determine the members who would retire.
(a) 1 and 3
(b) 2 only
(c) 2 and 3
(d) 3 only
Correct Answer is A (The retiring members are eligible for re-election.)

64) Once an emergency comes to cease, how long the extension for the Lok
Sabha can run?
(a) 1 Year
(b) 3 Months
(c) Indefinite
(d) 6 months
Correct Answer is D (6 months)

65) Which statement/statements is/are correct with respect to special


knowledge of nominated members in the Rajya Sabha, the nominated
members can come from?
1. Art
2. Science
3. Literature
4. Social science
Select the correct answer using the codes given below.
(a) 1, 2 and 4 only
(b) 2 only
(c) 1,3 and 4 only
(d) 1, 2, 3 and 4
Correct Answer is D (All are correct)

66) Which of the following is the number which represents Lok Sabha
members as representatives of the states?
(a) 530
(b) 550
(c) 552
(d) 545
Correct Answer is 530 (530 members come from state. The maximum limit
of the members is 550 and at present there are 545 members who sit in the
Loksabha.)

67) Which of the following constitutional amendment act reduced the age
of voting from 21 to 18?
(a) 60th constitutional amendment act
(b) 61st constitutional amendment act
(c) 62nd constitutional amendment act
(d) 63rd constitutional amendment act
Correct Answer is B (61st constitutional amendment act)

68) Which statement/statements is/are correct?


1. 84th constitutional amendment act extended the provision of
reservation till 2020.
2. 84th constitutional amendment act of 2001, opted for re-
fixing of the reserved seats taking 1991 population figures as
base.
Select the correct answer using the codes given below...
(a) 1 only
(b) 2 only
(c) None
(d) Both of the above
Correct Answer is B (The Constitution (Ninety-fifth Amendment) Act,
2009, extended the period of reservation of seats for the Scheduled Castes
and Scheduled Tribes and representation of the Anglo-Indians in the Lok
Sabha and the State Legislative Assemblies for another ten years, i.e. up to
26 January 2020.)

69) Which statement/statements is/are correct?


1. The delimitation act has enacted 4 times so far.
2. 42nd constitutional amendment act froze the allocation of
seats in the Lok Sabha till 2000 at 1971 level.
3. 84th Constitutional amendment act changed the year 2000 for
25 more years, till 2026.
Which of the statements given above is are correct...
(a) only 3
(b) 1 and 2 only
(c) 1, 2 and 3
(d) None
Correct Answer is C ( 84th constitutional amendment act came in 2001. The
act also empowers the government to go for readjustment and
rationalisation of territorial constituencies.)

70) Which of the following amendment extended the duration of two


members in Lok Sabha coming from an Anglo-Indian community to 2020?
(a) 94th constitutional amendment act 2009
(b) 91st constitutional amendment act 2009
(c) 90th constitutional amendment act 2009
(d) 95th constitutional amendment act 2009
Correct Answer is D (95th constitutional amendment act 2009.)

71) what is the membership number of Cabinet committee?


(a) 2
(b) 3 to 8
(c) 5 to 7
(d) 10
Correct Answer is B (Cabinet committees generally include only the
Cabinet ministers.)

72) Consider the following statements:


1. The Cabinet committee is an extra constitutional in
the emergence
2. Standing committee and ad hoc committee are parts of
Cabinet committee
3. Cabinet committees are set up by the President
Which of the above correctly matched ?
(a) 1 and 2
(b) 3 only
(c) 1, 2 and 3
(d) None
Correct Answer is A (Cabinet committees are set up by the Prime Minister)

73) Which statement/statements is/are correct?


1. The decisions of the Cabinet committee cannot be reviewed by
Cabinet.
2. Political Affairs Committee deals with all policy matters
pertaining to domestic and foreign affairs.
3. Parliamentary Affairs Committee looks after the progress of
government business in the Parliament.
(a) 1 and 2
(b) 2 only
(c) 2 and 3
(d) 3 only
Correct Answer is C (The decisions of the Cabinet committee can be
reviewed by Cabinet.)

74) Which of the following authority heads the political affairs committee?
(a) Home Minister
(b) President
(c) Prime Minister
(d) Leader of Opposition
Correct Answer is Prime Minister (The political affairs committee is
considered as the most powerful committee of all.)

75) Which statement/statements is/are correct?


1. At present, the Rajya Sabha has 229 members from the states.
2. Rajya Sabha can have 12 nominated members.
3. President is the authority to nominate the members in the
Rajya Sabha.
4. In Rajya Sabha, the states are represented by proportional
representation and single transferable vote.
Select the correct answer using the codes given below.
(a) 1, 2 and 4 only
(b) 2 only
(c) 1,3 and 4 only
(d) 1, 2, 3 and 4
Correct Answer is D (Unlike America, In India different number of
members comes from different states. For instance, from Uttar Pradesh 31
members, and from Tripura only one member comes.)
76) Which of the following authority chairs the Parliamentary affairs
committee?
(a) Home Minister
(b) Prime Minister
(c) President
(d) Leader of Opposition
Correct Answer is Home Minister (Home Minister)

77) Which of the following heads the appointment committee in higher


offices such as Central Secretariat?
(a) Leader of Opposition
(b) Home Minister
(c) President
(d) Prime Minister
Correct Answer is Prime Minister (Appointment committee deals with
appointments at a higher level, such as appointments in Central Secretariat
public enterprises, banks and financial institutions)

78) Which statement/statements is/are correct?


1. Part V of the Constitution deals with Parliament and its
organisation.
2. President of India is neither a member of Lok Sabha nor Rajya
Sabha.
3. In context of Parliament, India has modeled on British pattern.
Select the correct answer using the codes given below...
(a) 1 only
(b) 2 and 3 only
(c) 1 and 3 only
(d) 1, 2 and 3
Correct Answer is D (All are correct.)

79) Which statement/statements is/are correct?


1. Group of ministers (GoM) is an ad hoc body.
2. GoM can give its recommendations the Cabinet.
3. GoM cannot take decisions on behalf of the Cabinet.
Which of the statements given above is are correct...
(a) only 3
(b) 1 and 2 only
(c) 1, 2 and 3
(d) None
Correct Answer is B (GoM can also take decisions on behalf of the
Cabinet.)

80) Which of the following chairs the economic affairs committee?


(a) President
(b) Finance Minister
(c) Home Minister
(d) Prime Minister
Correct Answer is Prime Minister (Economic Affairs committee deals with
government activities in the economic sphere.)

81) Which of the following is the political head of the services?


(a) President
(b) Prime Minister
(c) Home Minister
(d) None of the above
Correct Answer is Prime Minister (Prime Minister)

82) Consider the following statements:


1. The oath of office and secrecy for the Prime Minister is similar
to that of union ministers
2. Council of ministers including the Prime Minister should not
exceed 15% of the strength of Lok Sabha in number
3. All executive actions of the Government of India are taken in
the name of Prime Minister
Which of the above correctly matched ?
(a) 1 and 2
(b) 3 only
(c) 1, 2 and 3
(d) None
Correct Answer is A (All executive actions of the Government of India are
taken in the name of President. The 15% strength limit was added by 91st
Constitutional Amendment act.)

83) Which statement/statements is/are correct?


1. Article 74 says that there shall be a Council of ministers to
advise the Prime Minister.
2. Article 75 says the ministers shall hold office at the pleasure of
the President.
3. Article 78 refers to duties of the prime minister.
(a) 1 and 2
(b) 2 only
(c) 2 and 3
(d) 3 only
Correct Answer is C (Article 74 says that there shall be a Council of
ministers to advise the President. Under article 75, The Prime Minister is
appointed by the President. And, under Article 78 Prime Minister
communicates the important policy decisions to the President.)

84) For which one of the following office, Primus inter Pares phrase can be
used ?
(a) President
(b) Chief Minister
(c) Prime Minister
(d) None of the above
Correct Answer is C (Prime Minister)
85) Which statement/statements is/are correct?
1. As per article 74, Council of ministers can advise President.
2. The advice given by council to the president is binding.
3. 42nd Constitutional Amendment act made the advice by
Council binding on the President.
Select the correct answer using the codes given below.
(a) 1 only
(b) 2 and 3 only
(c) 1 and 3 only
(d) 1, 2 and 3
Correct Answer is D (All of the above.)

86) Which of the following article talks about the collective responsibility
of the Council?
(a) article 74
(b) article 75
(c) article 76
(d) article 78
Correct Answer is B (Article 75 says that the Council of ministers will
swim together and sink together. Although, there is also a provision of
individual responsibility.)

87) Which of the following article refers to the individual responsibility of


the ministers ?
(a) article 71
(b) article 72
(c) article 74
(d) article 75
Correct Answer is D (article 75)

88) Which of the following group or groups form the part of Council of
ministers?
1. Cabinet ministers
2. Ministers of State
3. Deputy ministers
Select the correct answer using the codes given below...
(a) 1 only
(b) 2 and 3 only
(c) 1 and 3 only
(d) 1, 2 and 3
Correct Answer is D (All of the above.)

89) Which statement/statements is/are correct?


1. An independent charge can be given to a Minister of State.
2. An independent charge cannot be given to the Deputy
Minister.
3. Ministers of State have the privilege to attend Cabinet
meetings.
Which of the statements given above is are correct...
(a) only 3
(b) 1 and 2 only
(c) 1, 2 and 3
(d) None
Correct Answer is B (Ministers of State do not attend Cabinet meetings
unless they're called for it.)

90) Which of the following is highest decision-making authority ?


(a) Council of Minister
(b) Cabinet
(c) Niti Ayog
(d) Planning Commission
Correct Answer is B (Cabinet)
91) For how long a vice president can act as a president in case of
latter removal of resignation?
(a) 9 months
(b) Till the next president takes over
(c) 1 Year
(d) 6 months
Correct Answer is 6 months (6 months)

92) Consider the following statements:


1. In case of Vice President, formal impeachment is not required
2. The Vice President has a tenure of 5 years
3. Vice President can be removed by an special majority in
Rajya Sabha
Which of the above correctly matched ?
(a) 1 and 2
(b) 3 only
(c) 1, 2 and 3
(d) None
Correct Answer is A
(Vice President can be removed by an absolute majority in Rajya Sabha.
A resolution can be reduced in Rajya Sabha after giving notice of at least 14
days. Once the resolution is passed in Rajya Sabha it is supposed to be
agreed by the Lok Sabha.)

93) Which statement/statements is/are correct?


1. Vice president's election can be challenged on the ground of
incomplete electoral College.
2. Supreme Court can declare the vice president election as void.
3. If Vice president's election is declared void then the actions
executed by the Vice President's before the declaration are not
invalidate.
(a) 1 and 2
(b) 2 only
(c) 2 and 3
(d) 3 only
Correct Answer is C (Vice president's election cannot be challenged on the
ground of incomplete electoral College.)

94) Which of the following is an ex-officiol chairman of Rajya Sabha?


(a) Vice President
(b) President
(c) Speaker
(d) Deputy Speaker
Correct Answer is Vice President (Vice President)

95) Which of the following is a ground for vacancy in the Vice President's
office?
1. death
2. removal
3. resignation
4. disqualification
Select the correct answer using the codes given below.
(a) 1, 2 and 4 only
(b) 2 only
(c) 1,3 and 4 only
(d) 1, 2, 3 and 4
Correct Answer is D (Any of the above.)

96) Which of the following authority usually advises the President to


summon or prorogue the house?
(a) Home Minister
(b) Leader of the Opposition
(c) Speaker
(d) Prime Minister
Correct Answer is Prime Minister (It is the Prime Minister who advises the
president to summon or prorogue the house.)

97) Which of the following Prime Minister was not from Rajya Sabha ?
(a) Indira Gandhi
(b) Manmohan Singh
(c) Deve Gowda
(d) Morarji Desai
Correct Answer is Morarji Desai (Morarji Desai)

98) Which of the following vice presidents have acted as the president
when the president died in the office?
1. VV Giri
2. BD Jatti
3. Zakir Hussain
Select the correct answer using the codes given below...
(a) 1 and 2 only
(b) 2 and 3 only
(c) 1 and 3 only
(d) 1, 2 and 3
Correct Answer is A (VV GIRI acted as a president when the then President
Dr Zakir Hussain died. Similarly, BD Jatti acted as President when
Fakhruddin Ali Ahmed died in the office.)

99) Which statement/statements is/are correct?


1. Presidents has the authority to appoint Prime Minister.
2. President can use his personal discretion while appointing
Prime Minister in certain cases.
3. When the sitting Prime Minister dies during the office,
President may use his or her situational discretion.
Which of the statements given above is are correct...
(a) only 3
(b) 1 and 2 only
(c) 1, 2 and 3
(d) None
Correct Answer is C (When there is no clear majority, then the President
has a leeway for selection and appointment of Prime Minister.)

100) For how long a person who is not a member of either house can
become the Prime Minister of the country?
(a) 1 Year
(b) six months
(c) He/she cannot become
(d) 3 months
Correct Answer is six months (six months)
101. Which of the following is/are true
1. Simple majority is required in slate assembly to either create or
abolish legislative council.
2. To create or abolish legislative council, simple majority is
required in Parliament.
a) Only 1
b) None
c) Only 2
d) Both of the Above
C is the Correct Answer (special majority is required in state assembly..)

You can also join telegram channel for latest updates

Telegram Exam Group (For Updates on UPSC)

Telegram UGC NET Channel (UGC Material)

UGC NET Group (To Meet Other Aspirants and Free Test Series)

UPSC NET Group (To Meet Other Aspirants and Free Test
Series)

You can also enjoy our Youtube videos on our channel.

Don’t Forget to rate us on Amazon and claim a Free UPSC or UGC


(Any One) test series worth ₹ 1999 on our telegram group given
above.
102. Which of the following is/are true
1. Maximum Strength of legislative assembly is 500, while usual
minimum strength is 60.
2. In case of Arunachal, Goa and Sikkim, the minimum strength is
40.
a) Only 1
b) None
c) Only 2
d) Both of the Above
A is the Correct Answer (Minimum Strength is 30 and not 40 In case of
Arunachal, Goa and Sikkim.)

103. Which of the following is/are true


1. In state assembly such as Nagaland, the minimum Strength is 46.
2. As per 95th amendment, two Anglo Indian community member
can be nominated by Governor.
a) Only 1
b) None
c) Only 2
d) Both of the Above
A is the Correct Answer (As per 95th amendment, one Anglo Indian
community member can be nominated by Governor.)

104. Which of the following is/are true


1. 42nd amendment froze the number of seats in assembly till year
2020.
2. 84th amendment extended the seat readjustment for another 30
years
a) Only 1
b) None
c) Only 2
d) Both of the Above
B is the Correct Answer (42nd amendment adjusted the seats till 2000. 84th
amendment further expanded it till 2025.)

105. Which of the following is/are true


1. 87th amendment act of 2003 set 2001 as the base year census
figure for delimitation of constituency.
2. Size of council depends upon strength of assembly.
a) Only 1
b) None
c) Only 2
d) Both of the Above
D is the Correct Answer (Both are correct.)

106. Which of the following is/are true


1. Strength of state council is 1/3rd of State assembly, given the
minimum Strength as 40.
2. Actual strength of council is determined by Governor.
a) Only 1
b) None
c) Only 2
d) Both of the Above
A is the Correct Answer (Actual strength of council is determined by
Parliament.)

107. Which of the following is/are true


1. 1/12 members of Legislative council are elected by graduates and
Teachers each.
2. 1/6th members of council are elected by local bodies and
legislative assemblies each.
a) Only 1
b) None
c) Only 2
d) Both of the Above
A is the Correct Answer (1/3rd members of council are elected by local
bodies and legislative assemblies each.)

108. Which of the following is/are true concerning Council...


1. 1/6th members of council are nominated by Governor.
2. 5/6th members of council are indirectly elected.
a) Only 1
b) None
c) Only 2
d) Both of the Above
D is the Correct Answer (Both are correct.)

109. Which of the following is/are true


1. During Emergency, the term of assembly can be extended one
year at a time
2. Once the emergency is revoked, this extension cannot go beyond
6 months.
a) Only 1
b) None
c) Only 2
d) Both of the Above
D is the Correct Answer (Both are correct.)

110. Which of the following is/are true


1. In Legislative Council, nomination of members can be done by
CM.
2. In Legislative council, the retiring member can be re-elected.
a) Only 1
b) None
c) Only 2
d) Both of the Above
C is the Correct Answer (Nomination is done by Governor.)

111. Which of the following is/are true


1. Under article 371 special Provisions for the states have been
added via amendments as they did not exist in original
constitution.
2. Articles 371 A to 371 J deal with special provisions for various
states.
a) Only 1
b) None
c) Only 2
d) Both of the Above
D is the Correct Answer (Both are correct.)

112. Which of the following is/are true


1. Article 371, the president is authorised to confer special
responsibility over Governor of Maharashtra and Gujarat
2. Article 371 A makes special provisions for Assam state.
a) Only 1
b) None
c) Only 2
d) Both of the Above
A is the Correct Answer (Article 371 A makes special provisions for
Nagaland state.)

113. Which of the following is/are true


1. Article 371 A ensures that state does not intervene in Naga
Customary laws.
2. Under Article 371 A, the administration of Tuensang district is in
the hands of Central government.
a) Only 1
b) None
c) Only 2
d) Both of the Above
A is the Correct Answer (Under Article 371 A, the administration of
Tuensang district is in the hands of Governor.)

114. Which of the following is/are true


1. Under Article 371B, President can create a committee for which
the members are elected from tribal areas.
2. Under Article 371 C, President can form a committee for Which
members are elected from hill area.
a) Only 1
b) None
c) Only 2
d) Both of the Above
D is the Correct Answer (Both are correct.)

115. Which of the following is/are true


1. Article 371 D and 371 E Contain special provision for state of
Andhra Pradesh.
2. Article 371 E empowers the Parliament to form Central
university in the state.
a) Only 1
b) None
c) Only 2
d) Both of the Above
D is the Correct Answer (Both are correct.)

116. Which of the following is/are true


1. Article 371 F contains special provision Sikkim.
2. Sikkim legislative assembly consists of not more than 40
members.
a) Only 1
b) None
c) Only 2
d) Both of the Above
A is the Correct Answer (Sikkim legislative assembly consists of not than
30 members.)

117. Which of the following is/are true


1. Article 371 G deals with state of Manipur.
2. As per special Provision of article 371 G, Centre cannot
intervene in the Customary laws of state.
a) Only 1
b) None
c) Only 2
d) Both of the Above
C is the Correct Answer (Article 371 G deals with state of Mizoram.)

118. Which of the following is/are true


1. Under Article 371-H, special Provisions have been given to state
of Arunachal Pradesh.
2. Arunachal assembly is to consist of not less than 30 members.
a) Only 1
b) None
c) Only 2
d) Both of the Above
D is the Correct Answer (Both are correct.)

119. Which of the following is/are true


1. Article 371J, President is empowered to provide special
responsibility to Governor of North east.
2. Under special provisions, a separate board is established for
North East.
a) Only 1
b) None
c) Only 2
d) Both of the Above
B is the Correct Answer (Article 371J, President is empowered to provide
special responsibility to Governor of Karnataka. Under special provisions, a
separate establishment board for Hyderabad and Karnataka region is
formed.)

120. Which of the following is/are true


1. 98th constitutional amendment act inserted Article 371 J.
2. 371J deals with development of Hyderabad Karnataka region
a) Only 1
b) None
c) Only 2
d) Both of the Above
D is the Correct Answer (Both are correct.)

121. Which of the following is/are true


1. "Local Government" as a subject is given under 7th Schedule in
state list.
2. Balwant Rai Mehta committee recommended a 2 tier system of
government.
a) Only 1
b) None
c) Only 2
d) Both of the Above
A is the Correct Answer (Balwant Rai Mehta committee recommended a 3
tier system of government.)

122. Which of the following is/are true


1. Three tier system of Balwant rai mehta demarcated village level,
Block level and district level.
2. At block and district levels, members are directly elected.
a) Only 1
b) None
c) Only 2
d) Both of the Above
A is the Correct Answer (At block and district levels, members are
indirectly elected.)

123. Which of the following is/are true Concerning Balwant Rai Mehta
recommendations
1. District magistrate is the head of Zila Parishad.
2. Panchayat Samiti is an executive body while Zila parishad is an
advisory one.
a) Only 1
b) None
c) Only 2
d) Both of the Above
D is the Correct Answer (Both are correct.)

124. Which of the following is/are true


1. Panchayat's Raj was first started in Rajasthan.
2. Rajasthan installed a three tier system.
a) Only 1
b) None
c) Only 2
d) Both of the Above
D is the Correct Answer (Panchayat's Raj was first started in "Nagaur"
district of Rajasthan.)

125. Which of the following is/are true


1. Ashok Mehta committee suggested a two tier system of
Panchayati Raj.
2. Ashok Mehta committee suggested that Zila Parishad should be
an executive body.
a) Only 1
b) None
c) Only 2
d) Both of the Above
D is the Correct Answer (Both are correct.)

126. Which of the following is/are true


1. K. Santhanam committee studied finances of Panchayati Raj.
2. K. Santhanam committee studied Panchayati Raj elections.
a) Only 1
b) None
c) Only 2
d) Both of the Above
D is the Correct Answer (Both are correct.)

127. Which of the following is/are true


1. Reservation for SC/ST in PRI was recommended by Balwant Rai
Mehta committee
2. Balwant Rai Mehta committee recommended to Provide
constitutional status to Panchayati Raj.
a) Only 1
b) None
c) Only 2
d) Both of the Above
B is the Correct Answer (Ashok Mehta committee suggested both of the
above recommendations.)

128. Which of the following is/are true


1. GVK Rao committee was formed for Administrative
arrangement for Rural Development and Poverty Alleviation.
2. GVK Rao committee recommended to bestow significant role
over Zila Parishad.
a) Only 1
b) None
c) Only 2
d) Both of the Above
D is the Correct Answer (Both are correct.)

129. Which of the following is/are true


1. GVK Rao committee advocated to create district planning
committee under DC or Minister.
2. Ashok Mehta and GVK Rao committee advocated the reduction
of the role of DC.
a) Only 1
b) None
c) Only 2
d) Both of the Above
C is the Correct Answer (Hanumant Rao committee advocated to create
district planning committee under DC or Minister.)

130. Which of the following is/are true


1. LM Singhvi committee was formulated for Revitalisation of
Panchayati Raj.
2. Rajiv Gandhi appointed LM Singhvi as committee chairman.
a) Only 1
b) None
c) Only 2
d) Both of the Above
D is the Correct Answer (Both are correct.)

131. Which of the following is/are true


1. Cut motion represents disapproval of policy.
2. When a specified amount is deducted from the demand then it is
called token Cut.
a) Only 1
b) None
c) Only 2
d) Both of the Above
A is the Correct Answer (When a specified amount is deducted from the
demand then it is called economy Cut.)

132. Which of the following is/are true


1. When the amount of demand is reduced merely by 100 then the
same is called Token Cut.
2. Cut motions are generally moved by the opposition.
a) Only 1
b) None
c) Only 2
d) Both of the Above
D is the Correct Answer (Both are correct.)

133. Which of the following is/are true


1. Cut motion leads to discussion over demand for grant.
2. Speaker has no power whatsoever to decide whether discussion
on a cut motion will be admissible.
a) Only 1
b) None
c) Only 2
d) Both of the Above
A is the Correct Answer (Speaker has the power to decide whether
discussion on a cut motion will be admissible or not.)

134. Which of the following is/are true


1. To withdraw money from consolidated fund of India, we need
parliament's approval.
2. For parliament's approval, a bill is enacted which is called
Appropriation Bill.
a) Only 1
b) None
c) Only 2
d) Both of the Above
D is the Correct Answer (Both are correct.)

135. Which of the following is/are true


1. Amendment can be introduced in the appropriation bill.
2. No debate can be done on appropriation bill.
a) Only 1
b) None
c) Only 2
d) Both of the Above
C is the Correct Answer (No amendment can be introduced in the
appropriation bill. However, Debate can be done.)

136. Which of the following is/are true


1. Article 110 (1)(a) of the constitution deals with Appropriation
Bill.
2. Finance bill talks about alteration and abolition of taxes.
a) Only 1
b) None
c) Only 2
d) Both of the Above
C is the Correct Answer (Article 110 (1)(a) of the constitution deals with
Finance Bill.)

137. Which of the following is/are true


1. A Finance bill can be a money bill.
2. Finance bill gives effect to the financial proposals of government
for upcoming year.
a) Only 1
b) None
c) Only 2
d) Both of the Above
D is the Correct Answer (Both are correct.)
138. Which of the following is/are true
1. LM SINGHVI committee recommended to establish Nyaya
Panchayats.
2. LM SINGHVI committee did not support the constitutional
recognition of Panchayati Raj Institution.
a) Only 1
b) None
c) Only 2
d) Both of the Above
A is the Correct Answer (The committee supported constitutional
recognition and also recommended to given more financial Resources to
village Panchayats.)

139. Which of the following is/are true


1. 64th constitutional amendment bill bestowed constitutional status
to PRI.
2. The first Constitutional amendment bill concerning constitutional
status to PRI was passed by LokSabha, not RajyaSabha.
a) Only 1
b) None
c) Only 2
d) Both of the Above
C is the Correct Answer (64th constitutional amendment bill made an
attempt to bestow constitutional status to PRI. The opposition opposed the
bill on the that it would strengthen centralization in federal structure)

140. Which of the following is/are true


1. 73rd amendment added Part IX and schedule XI in the
constitution.
2. There were temporary and compulsory provisions in the act.
a) Only 1
b) None
c) Only 2
d) Both of the Above
D is the Correct Answer (Both are correct.)

41. Which of the following is/are true


1. 73rd amendment act does not apply to J&k, Nagaland, Mizoram
and Meghalaya
2. Minimum age for contesting panchayat elections is 25 years.
a) Only 1
b) None
c) Only 2
d) Both of the Above
A is the Correct Answer (Minimum age for contesting panchayat elections
is 21 years.)

142. Which of the following is/are true


1. Part IX of our constitution does not apply to schedule five areas.
2. The Subject matter for article 243A is Gram Sabha.
a) Only 1
b) None
c) Only 2
d) Both of the Above
D is the Correct Answer (Both are correct)

143. Which of the following is/are true


1. Article 243 D talks about reservation seats.
2. Article 243 F is about disqualification of membership.
a) Only 1
b) None
c) Only 2
d) Both of the Above
D is the Correct Answer (Both are correct)

144. Which of the following is/are true


1. Village Panchayats find a mention in Directive Principles of
state Policies.
2. Local government is a subject mentioned under "Residuary list
" 7th schedule.
a) Only 1
b) None
c) Only 2
d) Both of the Above
A is the Correct Answer (Local government is a subject mentioned under
"state list " 7th schedule. DPSPs article 40 talk about local government.)

145. Which of the following is/are true


1. First municipal corporation was setup in madras in 1687.
2. Lord Ripon resolution of 1882 was hailed as "Magana Karta" for
Self Government.
a) Only 1
b) None
c) Only 2
d) Both of the Above
D is the Correct Answer (Both are correct.)

146. Which of the following is/are true


1. As per GOI act 1935, local self government was not discussed at
all.
2. 74th amendment act added 12th schedule to the constitution.
a) Only 1
b) None
c) Only 2
d) Both of the Above
C is the Correct Answer (As per GoI act 1935, local government was a
provincial subject.)

147. Which of the following is/are true


1. 1/3rd members of metropolitan planning committee should be
elected by elected members of municipalities and panchayat
Chairpersons.
2. Municipal corporations are created in big cities such as Delhi,
Mumbai, Hyderabad etc.
a) Only 1
b) None
c) Only 2
d) Both of the Above
C is the Correct Answer (2/3rd members of metropolitan planning
committee should be elected by elected members of municipalities and
panchayat Chairpersons.)

148. Which of the following is/are true


1. Municipalities are for the administration of smaller cities.
2. A municipality has three authorities Viz. Council, Standing
committee and Chief executive officer.
a) Only 1
b) None
c) Only 2
d) Both of the Above
D is the Correct Answer (Both are correct

149. Which of the following is/are true


1. Notified area committee is created to administer a town which is
backward.
2. Notified area committee is an elected body.
a) Only 1
b) None
c) Only 2
d) Both of the Above
B is the Correct Answer (Notified area committee is created to administer a
town which is rapidly industrialising. Notified area committee is not an
elected body as all the members of the committee are nominated by state
government.)

150. Which of the following is/are true


1. Town area Committee looked after areas such as drainage, roads
and lighting.
2. Town area committee can be wholly elected or wholly
nominated.
a) Only 1
b) None
c) Only 2
d) Both of the Above
D is the Correct Answer (Both are correct)
151) Which of the following state was never a
union territory ?
(a) Himachal Pradesh
(b) Manipur
(c) Goa
(d) Haryana
Correct Answer is D. (Himachal Pradesh, Manipur, Tripura, Mizoram,
Arunachal Pradesh and Goa, which are states today were formerly union
territories.)

152) Consider the following pairs :


1. Delhi : Lt. Governor
2. Puducherry : Lt. Governor
3. Andaman : Commissioner
Which of the above correctly matched ?
(a)1 and 2
(b) 3 only
(c) 1, 2 and 3
(d) None
Correct Answer is A. (Lieutenant Governor in the case of Delhi,
Puducherry and Andaman and Nicobar Islands and Administrator in the
case of Chandigarh, Dadra and Nagar Haveli, Daman and Diu and
Lakshadweep.)

153) Which statement/statements is/are correct ?


1. Delhi is the only union territory that has a high court of its own
2. Delhi can make laws on any subject of the State List and the
Concurrent List
3. Legislative assembly of Puducherry can make laws on any
subject of the State List and the Concurrent List

(a)1 and 3
(b) 2 only
(c) 2 and 3
(d) 3 only
Correct Answer is A. ( Delhi can make laws on any subject of the State
List (except public order, police and land) and the Concurrent List)

154) To which UT, 69th Constitutional Amendment Act of 1991 provided


a special status ?
(a) Puducherry
(b) Chandigarh
(c) Delhi
(d) Andaman
Correct Answer is C. (Delhi)

155) Which statement/statements is/are correct ?


1. Lt. governor is empowered to promulgate ordinances during
recess of the assembly.
2. Every ordinance by LG must be approved by the assembly
within six weeks from its reassembly.
3. LG can withdraw an ordinance at any time.
4. LG cannot promulgate an ordinance when the assembly is
dissolved or suspended.
Select the correct answer using the codes given below.
(a) 1, 2 and 4 only
(b) 2 only
(c) 1,3 and 4 only
(d) 1, 2, 3 and 4
Correct Answer is D. (All are Correct.)

156) Under which high court jurisdiction does Lakshadweep come?


(a) Mumbai High court
(b) Madras High court
(c) Kerala High court
(d) Delhi High court
Correct Answer is C. (Lakshadweep is under Kerala high court. Daman
and Diu comes under Bombay high court. )

157) Which Committee advocated separate district planning bodies under


either the District Collector or a minister?
(a) LM Singhvi committee
(b) Santhnam Committee
(c) Ashok Mehta Committee
(d) Hanumanth Rao committee
Correct Answer is D (Hanumanth Rao Committee.)

158) Which statement/statements is/are correct ?


1. Ministry of Home Affairs is the nodal ministry for the matters
of Union Territories.
2. UTs without a legislature have the forum of Home Minister’s
Advisory Committee (HMAC).
3. Meetings of the HMAC are chaired by the Prime Minister.
Select the correct answer using the codes given below.
(a) 1 only
(b) 1 and 2 only
(c) 1 and 3 only
(d) 1, 2 and 3
Correct Answer is B (Meetings of the HMAC are chaired by the Union
Home Minister.)

159) Which statement/statements is/are correct ?


1. Each state having scheduled areas has to establish a tribes
advisory council to advise on welfare and advancement of the
scheduled tribes.
2. Tribal Affairs consists of 20 members, 3/4th of whom are to
be the representatives of the scheduled tribes
3. District council consisting of 30 members, of whom four are
nominated by the governor and the remaining 26 are elected on
the basis of adult franchise.
Which of the statements given above is are correct?
(a) only 3
(b) 1 and 2 only
(c) 1, 2 and 3
(d) None
Correct Answer is C(All are correct.)

160) How many tier of government was suggested by Ashok Mehta


committee ?
(a) Three Tier
(b) Two tier
(c) None
(d) Single system
Correct Answer is B(Two tier system.)

161) Which of the following case determined that the preamble is not a part
of Indian constitution ?
(a) Berubari Case
(b) Keshvananda Bharati Case
(c) LIC of India case
(d) Vinit Narayan Case
Correct Answer is A
CASE Preamble part of Preamble part of
constitution constitution
Berubari Case No Non-Justiciable
Yes (Can be amended
Keshvananda Bharati but Basic Structure
Non-Justiciable
case should not be
tempered. )
LIC of India case Yes Non-Justiciable
162) Consider the following pairs:
1. Habeas Corpus : Issued against both public authorities as well
as private individuals.
2. Prohibition : Not available against administrative authorities,
legislative bodies, and private individuals or bodies
3. Certiorari : Can be issued even against administrative
authorities affecting rights of individuals.
Which of the above correctly matched ?
(a) 1 and 2
(b) 3 only
(c) 1, 2 and 3
(d) None
Correct Answer is C (Like prohibition, certiorari is also not available
against legislative bodies and private individuals.)

163) Which statement/statements is/are correct ?


1. The writ jurisdiction of the Supreme Court is narrower than
that of high court
2. Territorial jurisdiction of the High court is wider than that of a
Supreme court.
3. A remedy under Article 32 is in itself a Fundamental Right and
hence, the Supreme Court may not refuse to exercise its writ
jurisdiction.
(a) 1 and 3
(b) 2 only
(c) 2 and 3
(d) 3 only
Correct Answer is A (Territorial jurisdiction of the Supreme Court is wider
than that of a high court.)

164) Unlike the other four writs, this can be sought by any interested
person and not necessarily by the aggrieved person ?
(a) Mandmus
(b) Quo Warranto
(c) Prohibition
(d) Certiorari
Correct Answer is B (Quo Warranto)

165) Writ of mandamus cannot be issued for?


1. To enforce departmental instruction that does not possess
statutory force
2. When the duty is discretionary and not mandatory
3. To enforce a contractual obligation
4. Against the president of India or the state governors
Select the correct answer using the codes given below.
(a) 1, 2 and 4 only
(b) 2 only
(c) 1,3 and 4 only
(d) 1, 2, 3 and 4
Correct Answer is D (Writ of mandamus cannot be issued (a) against a
private individual or body; (b) to enforce departmental instruction that does
not possess statutory force; (c) when the duty is discretionary and not
mandatory; (d) to enforce a contractual obligation; (e) against the president
of India or the state governors; and (f) against the chief justice of a high
court acting in judicial capacity.)

166) Which of the following commission is not related to linguistic


reorganisation of states?
(a) Fazl Ali Commission
(b) JVP Commission
(c) Dhar Commission
(d) Santhnam Commission
Correct Answer is D (In 1962, Lal Bahadur Sastri appointed Santhanam to
preside over the committee on anti-corruption. Because of its thorough
investigative work and recommendations, the Committee earned a
reputation as Santhanam’s Committee on Anti-Corruption.)

167) Which of the following Fundamental Right is available to foreigners


as well ?
(a) Article 15
(b) Article 16
(c) Article 17
(d) Article 19
Correct Answer is C (Article 15,16,19,29 and 30 are not available to
foreigners and can be exercised by citizens only.)

168) Which statement/statements is/are correct ?


1. Article 33 empowers the Parliament to restrict or abrogate the
fundamental rights of the members of armed forces, para-military
forces, police forces, intelligence agencies
2. Any such law made by Parliament cannot be challenged in
any court of law.
3. Armed forces do not cover such employees of the armed
forces as barbers, carpenters, mechanics, cooks, chowkidars,
boot-makers, tailors who are non-combatants.
Select the correct answer using the codes given below.
(a) 1 only
(b) 2 and 3 only
(c) 1 and 3 only
(d) 1and 2 only
Correct Answer is D (Armed forces also cover such employees of the
armed forces as barbers, carpenters, mechanics, cooks, chowkidars, boot-
makers, tailors who are non-combatants.)

169) Which statement/statements is/are correct ?


1. 44th Amendment Act of 1978 abolished the right to property
as a Fundamental Right by repealing Article 19(1)(f) and Article
31 from Part III.
2. Guaranteed right to compensation is given in case of
acquisition or requisition under 300A
3. Part III still carries two provisions which provide for the
guaranteed right to compensation
Which of the statements given above is are correct?
(a) only 3
(b) 1 and 3 only
(c) 1, 2 and 3
(d) None
Correct Answer is B (No guaranteed right to compensation is given in case
of acquisition or requisition under 300A.)

170) Which of the following requires constitutional amendment ?


(a) Altering the territory of state
(b) Altering the name of any state
(c) Ceding territory to another nation
(d) Establishment of new state or states
Correct Answer is C (BERUBARI CASE: Though, the power to amend the
boundary is available with parliament, which does not ask for any
constitutional amendment, but the ceding of territory needs an amendment.
The same was seen when territory was ceded to east Pakistan.)

171) What is the status of Zonal Councils?


(a) Statutory Bodies
(b) Constitutional Bodies
(c) Legislative Bodies
(d) Exemplary Bodies
Correct Answer is A (Zonal Councils are established by an Act of the
Parliament – States Reorganisation Act of 1956. The act divided the country
into five zones (Northern, Central, Eastern, Western and Southern))
172) Consider the following pairs :
1. Originally, the constitution contained a Preamble with 395
Articles (divided into 22 Parts) and 8 Schedules.
2. Laws made by Parliament on the state subjects during a
National Emergency become inoperative six months after the
emergency has ceased to operate.
3. If the Parliament is not in session, President cannot issue
ordinances on the state subjects.
Which of the above correctly matched ?
(a) 1 and 2
(b) 3 only
(c) 1, 2 and 3
(d) None
Correct Answer is A (If the Parliament is not in session, President can issue
ordinances on the state subjects also.)

173) Which statement/statements is/are correct ?


1. In Bommai case the Supreme Court laid down that the
Constitution is federal and labelled federalism as its ‘basic
feature’.
2. Government of India (GoI) Act of 1935 provided for a three-
fold enumeration viz. federal, provincial and concurrent.
3. Life of the Lok Sabha may be extended beyond its normal
term by a law of Parliament for one year at a time
(a) 1 and 2
(b) 2 only
(c) 1, 2 and 3
(d) 3 only
Correct Answer is C (All are Correct.)

174) Which of the following area State legislature cannot impose taxes on
?
(a) Profession
(b) Sale of Goods
(c) Purchase of Goods
(d) Newspaper
Correct Answer is D (Newspaper)

175) Which statement/statements is/are correct ?


1. Chairman of a state public service commission is appointed
by the governor
2. Member of a state public service commission is removed by
President
3. Chairman of PSC can be removed only by the President
Select the correct answer using the codes given below.
(a) 1 only
(b) 2 only
(c) 3only
(d) 1, 2 and 3
Correct Answer is D (The chairman and members of the JSPSC are
appointed by the president; Similarly, judges of a state high court are
appointed by the president in consultation with the Chief Justice of India
and the governor of the state. They can also be transferred and removed by
the president.)

176) Concerning Zonal Council, Where does Uttarakhand fall?


(a) Central Zonal Council
(b) Northern Zonal Council
(c) Eastern Zonal Council
(d) Western Zonal Council
Correct Answer is A (Central Zonal Council)

177) Who of the following Appoint and Remove the state election
commissioner ?
(a) Appointment by President and Removal by Governor
(b) Appointment by Governor and Removal by Governor
(c) Appointment by President and Removal by President
(d) Appointment by Governor and Removal by President
Correct Answer is D (Appointment by Governor and Removal by
President)

178) Which statement/statements is/are correct ?


1. Governor of Assam may direct that an act of Parliament does
not apply to a tribal area (autonomous district)
2. President enjoys the power with respect to tribal areas
(autonomous districts) in Meghalaya.
3. Both Governor and President share the power concerning
Tripura and Mizoram.
Select the correct answer using the codes given below.
(a) 1 only
(b) 2 and 3 only
(c) 1 and 2 only
(d) 1, 2 and 3
Correct Answer is C (President enjoys the power with respect to tribal
areas (autonomous districts) in Meghalaya, Tripura and Mizoram.)

179) Which statement/statements is/are correct ?


1. 38th Amendment Act of 1975 made the declaration of a
National Emergency immune from the judicial review.
2. 44th Amendment Act of 1978 suppressed 38th amendment.
3. Minerva Mills case (1980), the Supreme Court held that the
proclamation of a national emergency can be challenged in a
court on the ground of malafide.
Which of the statements given above is are correct?
(1) only 3
(b) 1 and 2 only
(c) 1, 2 and 3
(d) None
Correct Answer is C (All are correct.)
180) Which article authorises the president to suspend the rights to move
any court for the enforcement of Fundamental Rights during a National
Emergency?
(a) Article 352
(b) Article 359
(c) Article 355
(d) Article 356
Correct Answer is B (Article 359)

181) What could be the maximum life of an ordinance ?


(a) Six months and six weeks
(b) Six months
(c) Six weeks
(d) 1 Year
Correct Answer is A (Six months and six weeks. six months being the
maximum gap between the two sessions of Parliament.)

182) Consider the following pairs :


1. Article 358 deals with the suspension of the
Fundamental Rights guaranteed by Article 19,
2. Article 359 deals with the suspension of Fundamental Rights
(except those guaranteed by Articles 20 and 21)
3. 44th Amendment Act of 1978 restricted the scope of Article
358.
Which of the above correctly matched ?
(a) 1 and 2
(b) 3 only
(c) 1, 2 and 3
(d) None
Correct Answer is C (44th amendments has imposed restrictions viz. the
six Fundamental Rights under Article 19 can be suspended only if the
National Emergency is declared on the ground of war or external aggression
and not on the ground of armed rebellion. Furthermore, only the laws which
are related with the Emergency are protected from being challenged and not
other laws. Also, the executive action taken only under such a law is
protected..)

183) Which statement/statements is/are correct ?


1. Article 358 automatically suspends the fundamental rights
under Article 19
2. Article 359 automatically can suspend any Fundamental
Right.
3. Article 355 imposes a duty on the Centre to ensure that the
government of every state is carried on in accordance with the
provisions of the Constitution.
(a) 1 and 3
(b) 2 only
(c) 2 and 3
(d) 3 only
Correct Answer is A (Article 359 does not automatically suspend any
Fundamental Right. It only empowers the president to suspend the
enforcement of the specified Fundamental Rights.)

184) How many months are given to the houses to approve President’s
Rule?
(a) 2 months
(b) 1 month
(c) 6 month
(d) 3 month
Correct Answer is A (2 months are given)

185) Which statement/statements is/are correct ?


1. In National Emergency there is no maximum period prescribed
for its operation. It can be continued indefinitely with the
approval of Parliament for every six months.
2. In President Rule there is a maximum period prescribed for its
operation which is three years.
3. Lok Sabha can pass a resolution for revocation of National
Emergency.
4. President Rule can be revoked by President only.
Select the correct answer using the codes given below.
(a) 1, 2 and 4 only
(b) 2 only
(c) 1,3 and 4 only
(d) 1, 2, 3 and 4
Correct Answer is D (All are Correct.)

186) Which of the following statement is wrong?


(a) Article 323-A enables the Parliament to take out the
adjudication of disputes relating to service matters from the civil
courts and the high courts and put it before the administrative
tribunals
(b) CAT is a multi-member body consisting of a chairman and
members.
(c) CAT members have been given the status of judges of High
Courts.
(d) CAT members are drawn from judicial branch only.
Correct Answer is D (CAT members are drawn from both judicial and
administrative streams and are appointed by the president.)

187) Which of the following statement is wrong out of the following ?


(a) No criminal proceedings can be started against the president
and the governors in respect of their personal acts.
(b) Immunity of personal acts is limited to the period of the term
of their office only and does not extend beyond that.
(c) The president and the governors cannot be sued during the
term of their office or thereafter if the act is done by them in the
exercise and performance of their official powers and duties.
(d) Civil Proceedings cannot be started against President during
their term of office in respect of their personal acts
Correct Answer is D (Civil Proceedings can be started against President
during their term of office in respect of their personal acts after giving two
months’ advance notice.)

188) Which statement/statements is/are correct ?


1. BG Kher committee was formed on the issue of official
language.
2. All he languages are given in the VII schedule of our
constitution.
3. Santhali was the last language added by the 92nd Amendment
Act of 2003 in the lot of languages.
Select the correct answer using the codes given below.
(a) 1 only
(b) 2 and 3 only
(c) 1 and 3 only
(d) 1, 2 and 3
Correct Answer is C (All he languages are given in the VIII schedule of
our constitution. The languages are given in the Part VII.)

189) Which statement/statements is/are correct with respect to co-operative


society?
1. Maximum number of directors of a co-operative society shall
not exceed twenty-one
2. The reservation is provided in cooperative society with one
seat for the Scheduled Castes or the Scheduled Tribes and two
seats for women on the board of every co-operative society
3. The term of office and office bearers is 6 years.
Which of the statements given above is are correct?
(1) only 3
(b) 1 and 2 only
(c) 1, 2 and 3
(d) None
Correct Answer is B (The term of office is 5 years. Cooperative society has
to file the return within 6 months of closure of every financial year.)
190) Which article says that council of ministers shall be collectively
responsible to the Lok Sabha ?
(a) Article 72
(b) Article 74
(c) Article73
(d) Article 75
Correct Answer is D (Article 75.)

191) When does the question hour start?


(a) 10 AM
(b) 10:30 AM
(c) 11:00 AM
(d) 11:30 AM
Correct Answer is C (The question hour starts at 11 o’clock.)

192) Consider the following Question and The papers:


1. Starred Question : Green Paper
2. Unstarred Question : White Paper
3. Short Notice Question : Yellow Paper
Which of the above correctly matched ?
(a) 1 and 2
(b) 3 only
(c) 1, 2 and 3
(d) None
Correct Answer is A (Short notice question is on Pink paper. There are
Questions to private members which are on Yellow paper.)

193) Which statement/statements is/are correct ?


1. For answering questions each ministries and departments are
divided into 5 groups.
2. No question is asked on Saturday.
3. Each minister has one fixed day of the week to answer the
question.
(a) 1 and 2
(b) 2 only
(c) 1, 2 and 3
(d) 3 only
Correct Answer is C (All are correct. From Monday to Friday, each day is
dedicated to different departments. On that day only questions pertaining to
that department can be asked. )

194) Which of the following is not a type of question ?


(a) Starred Question
(b) Question to Private members
(c) Short notice question
(d) Long Notice Question
Correct Answer is D (There is short notice question only. There is no long
notice question. Apart from Starred, there is a provision of unstarred
question as well.)

195) Which statement/statements is/are correct ?


1. Answer to a starred question may be followed by
supplementary question.
2. Unstarred question does not require any supplementary
question to be asked.
3. In short notice question, the urgent issues can be raised with a
notice of 15 days.
Select the correct answer using the codes given below.
(a) 1 and 2
(b) 2 only
(c) 1 and 3
(d) 1, 2, and 3
Correct Answer is A (In short notice question, the urgent issues can be
raised with a notice of 10 days else Minimum 15 days notice is required.)

196) Minimum how many days notice is given in advance to ask a question
in the house, generally?
(a) 15 days
(b) 20 days
(c) 5 days
(d) 7 days
Correct Answer is A (Minimum 15 days notice is required but the time
period can be reduced to 10 if any urgent matter arises.)

197) What is the general number of questions in the Unstarred list for a day
?
(a) 245
(b) 230
(c) 250
(d) 200
Correct Answer is B (Generally, the limit is 230 but it can be increased by
25 of the state is under President’s rule.)

198) Which statement/statements is/are correct ?


1. A member can give the maximum of 10 notices in a day.
2. If the notices given surpass the stipulated limit then the same
is carry forward for the next day.
3. The identical notice questions are disallowed.
Select the correct answer using the codes given below.
(a) 1 only
(b) 2 and 3 only
(c) 1 and 3 only
(d) 1, 2 and 3
Correct Answer is D (All are Correct.)
199) Which statement/statements is/are correct concerning the
admissibility of the Question in the house?
1. Admissibility of the question is determined by Direction 10A
of directions of Speaker.
2. Questions which generally exceed 150 words are not
admitted.
3. Questions related to state matters are not admitted.
Which of the statements given above is are correct?
(1) only 3
(b) 1 and 2 only
(c) 1, 2 and 3
(d) None
Correct Answer is C (All are Correct.)

200) Which of the following is not an Unstarred Question ?


(a) Statistical Question
(b) Too Long to reply
(c) Question on Local interest
(d) Where there is a scope of supplementaries
Correct Answer is D. (Where there is a scope of supplementaries requires a
question to be starred question.)
201) Which of the following does not participate
in the elections of President?
(a) Legislative Assemblies
(b) Legislative Councils
(c) Elected Members
(d) Both the houses
Correct Answer is B
(The nominated members of both of Houses of Parliament, the nominated
members of the state legislative assemblies, the members (both elected and
nominated) of the state legislative councils (in case of the bicameral
legislature) and the nominated members of the Legislative Assemblies of
Delhi and Puducherry do not participate in the election of the President.)

You can also join telegram channel for latest updates

Telegram Exam Group (For Updates on UPSC)

Telegram UGC NET Channel (UGC Material)

UGC NET Group (To Meet Other Aspirants and Free Test Series)

UPSC NET Group (To Meet Other Aspirants and Free Test
Series)

You can also enjoy our Youtube videos on our channel.

Don’t Forget to rate us on Amazon and claim a Free UPSC or UGC


(Any One) test series worth ₹ 1999 on our telegram group given
above.
202) Consider the following statements WITH RESPECT TO
PRESIDENTIAL ELECTION:
1. 50 proposers are required for nomination.
2. 50 seconders are required for nomination.
3. A security Deposit of 15000 is required in RBI.
Which of the above is/are correct ?
(a) 1 and 2
(b) 3 only
(c) 1, 2 and 3
(d) None
Correct Answer is C (The security deposit is forfeited in case candidate
fails to secure 1/6th of the polled votes.)

203) Which statement/statements is/are correct ?


1. Nominated members don’t participate in the election of
president.
2. Nominated members do participate in the impeachment
process of president.
3. Elected members of Delhi and Puducherry participate in the
election of the president.
(a) 1 and 2
(b) 2 only
(c) 1, 2 and 3
(d) 3 only
Correct Answer is C ( Elected members of Delhi and Puducherry
participate in the appointment of president but they don’t participate in the
impeachment process of the president.)

204) In case of vacancy of presidential office in how much time the same is
filled?
(a) 6 months
(b) 1 year
(c) 3 months
(d) 1 month
Correct Answer is 6 months (6 months)

205) Which statement/statements is/are correct ?


1. President does not possess Suspensive veto in the case of
money bills.
2. Pocket Veto does not prescribe time limit of withholding of
bill.
3. Absolute Veto can be applied on Private Members Bill.
4. Suspensive veto can be overridden by the legislature with a re-
passage of the bill.
Select the correct answer using the codes given below.
(a) 1, 2 and 4 only
(b) 2 only
(c) 1,3 and 4 only
(d) 1, 2, 3 and 4
Correct Answer is D (All are correct.)

206) Which of the following amendment made it obligatory for the


President to give his assent to a constitutional amendment bill?
(a) 24th amendment
(b) 42nd amendment
(c) 51st amendment
(d) 93rd amendment
Correct Answer is A (24th Constitutional Amendment Act of 1971 made it
obligatory for the President to give his assent to a constitutional amendment
bill.)

207) Which of the following amendment made the president’s satisfaction


final and conclusive in terms of ordinance ?
(a) 22nd amendment
(b) 38th amendment
(c) 42nd amendment
(d) 44th amendment
Correct Answer is 38th amendment (The 38th Constitutional Amendment
Act of 1975 made the President’s satisfaction final and conclusive and
beyond judicial review. But, this provision was deleted by the 44th
Constitutional Amendment Act of 1978. Thus, the President’s satisfaction is
justiciable on the ground of malafide.)

208) Which statement/statements is/are correct?


1. The ordinance may also cease to operate even earlier than the
prescribed 6 weeks.
2. The maximum life of an ordinance is 6 months 6 weeks.
3. An ordinance cannot be issued when one house is in session.
Select the correct answer using the codes given below.
(a) 1 only
(b) 2 and 3 only
(c) 1 and 2 only
(d) 1, 2 and 3
Correct Answer is C (The ordinance may also cease to operate even earlier
than the prescribed 6 weeks, if both the Houses of Parliament pass
resolutions disapproving it. If the Houses of Parliament are summoned to
reassemble on different dates, the period of six weeks is calculated from the
later of those dates. This means that the maximum life of an ordinance can
be 6 months and six weeks, in case of non-approval by the Parliament (six
months being the maximum gap between the two sessions of Parliament.
An ordinance can be issued even when one house is in session.)

209) Which statement/statements is/are correct ?


1. Governor can pardon a death sentence.
2. Governor can pardon a court martial.
3. Governor can commute a death sentence.
Which of the statements given above is are correct?
(a) only 3
(b) 1 and 2 only
(c) 1, 2 and 3
(d) None
Correct Answer is A (Governor Can Suspend, remit or Commute but
cannot pardon a death sentence.)

210) Under which article the Governor does have a pardoning power ?
(a) Article 164
(b) Article 163
(c) Article 162
(d) Article 161
Correct Answer is Article 161 (Article 161)

211) Which of the following is the 2nd highest office of the country ?
(a) Prime Minister
(b) President
(c) Home Minister
(d) Vice President
Correct Answer is D (Vice President is the 2nd highest office of the
country.)

212) Consider the following statements w.r.t. election of Vice-President:


1. Electoral college consists of both houses of the Parliament.
2. Electoral college consists of both elected and nominated
members.
3. Electoral college does not include members of state
legislative assemblies.
Which of the above correctly matched ?
(a) 1 and 2
(b) 3 only
(c) 1, 2 and 3
(d) None
Correct Answer is C (The composition of electoral college for vice
president election is different from electoral college for election of
President, but similar to election of president.)

213) Which statement/statements is/are correct ?


1. In case of the Presidential election, only the elected members
participate.
2. In case of a Presidential Election, the elected members of state
legislative members participate.
3. President is the 2nd highest office of the country after Prime
Minister.
(a) 1 and 2
(b) 2 only
(c) 2 and 3
(d) 3 only
Correct Answer is A (President is the highest office of the country.)

214) How many proposers are required for the nomination of a candidate
for the election of Vice President ?
(a) 10
(b) 20
(c) 30
(d) 40
Correct Answer is 20 (20 proposers and 20 seconders are required and a
security deposit of 15000 also needs to be deposited in RBI.)

215) Which statement/statements is/are correct concerning Vice President ?


1. Vice President is removed by a resolution of Rajya Sabha
passed by a special majority.
2. Formal impeachment is not required in Vice President.
3. 14 days of advance notice is necessary to move the resolution.
4. Vice President qualifies for re-election.
Select the correct answer using the codes given below.
(a) 1, 2 and 4 only
(b) 2 only
(c) 2,3 and 4 only
(d) 1, 2, 3 and 4
Correct Answer is C (Vice President is removed by a resolution of Rajya
Sabha passed by an absolute majority.)

216) Which of the following article says that the Prime Minister is
appointed by the President ?
(a) Article 72
(b) Article 73
(c) Article 74
(d) Article 75
Correct Answer is D (Article 75)

217) For how long the Vice President can act as President of India ?
(a) 1 month
(b) 3 months
(c) 6 months
(d) 12 months
Correct Answer is 6 months (6 months)

218) Which statement/statements is/are correct ?


1. Prime Minister recommends President to dismiss a member.
2. In case of death or resignation of prime minister the council of
minister dissolves automatically.
3. In case of death of a minister the council of minister dissolves
automatically.
Select the correct answer using the codes given below.
(a) 1 only
(b) 2 and 3 only
(c) 1 and 3 only
(d) 1 and 2
Correct Answer is D (In case of death of a minister the council of minister
does not dissolve. The vacancy can either be filled or leave vacant by Prime
Minister.)

219) Which statement/statements is/are correct ?


1. Prime Minister can ask the president to summon or prorogue
the house.
2. Prime Minister can’t recommend the dissolution of Loksabha.
3. Prime Minister can announce the government policies on the
floor of the house.
Which of the statements given above is are correct?
(1) only 3
(b) 1 and 3 only
(c) 1, 2 and 3
(d) None
Correct Answer is B (Prime Minister can also recommend the dissolution
of Loksabha.)

220) Which of the following article says that Council of Ministers can aid
and advice President ?
(a) Article 74
(b) Article 75
(c) Article 76
(d) Article 77
Correct Answer is Article 74 (Article 74)

221) Which of the following article says that there shall be Council of
Ministers?
(a) Article 73
(b) Article 74
(c) Article 75
(d) Article 76
Correct Answer is B (Article 74 says that there shall be council of
ministers with Prime Minister as the head. Council can aid and advice the
president, the advice is not inquired in any court of law.)

222) Consider the following statements:


1. Council of ministers shall not exceed 15% of the total strength
of the Loksabha.
2. Article 77 says that all the executive actions are taken in the
name President.
3. Article 78 says that it is the duty of the prime minister to
communicate all the decisions of the Council of Ministers to the
President.
Which of the above correctly matched ?
(a) 1 and 2
(b) 3 only
(c) 1, 2 and 3
(d) None
Correct Answer is C (All are Correct.)

23) Which statement/statements is/are correct ?


1. Minister of State can join Cabinet meetings if called for.
2. Deputy Ministers don’t hold any independent charge.
3. Ministers of state cannot be given independent charge.
(a) 1 and 2
(b) 2 only
(c) 2 and 3
(d) 3 only
Correct Answer is A (Ministers of state can be given independent charge.)

224) What is the maximum percentage of the council of ministers vis a vis
Loksabha ?
(a) 10%
(b) 12%
(c) 15%
(d) 20%
Correct Answer is 15% (The Council of Ministers shall not exceed 15% of
the strength of Loksabha.)

225) Which statement/statements is/are correct ?


1. Council Of Ministers is relatively bigger than the cabinet.
2. Council of Ministers does not have any provision of meeting.
3. Council of Ministers is a constitutional body.
4. The cabinet was defined in the original constitution.
Select the correct answer using the codes given below.
(a) 1, 2 and 3 only
(b) 2 only
(c) 1,3 and 4 only
(d) 1, 2, 3 and 4
Correct Answer is A (The cabinet was not defined in the original
constitution rather added by article 352 under 44th constitutional
amendment act.)

226) Which of the following amendment curtailed the property rights ?


(a) 1st amendment
(b) 2nd amendment
(c) 3rd amendment
(d) 4th amendment
Correct Answer is A (1st amendment)

227) When do we observe Constitution Day ?


(a) January 26
(b) August 15
(c) November 26
(d) January 1
Correct Answer is C (November 26 is observed as Constitutional Day or
the Samvidhan Divas. If we go back in time, it was 26th day of November
1949 when the constitution of India was adopted.)

228) Which statement/statements is/are correct ?


1. Shankari Prasad case – The word law includes only the
ordinary law.
2. Golaknath Case – Constitutional amendment act does not
come under article 13
3. Waman Rao case – Adhered to basic structure of the
constitution
Select the correct answer using the codes given below.
(a) 1 only
(b) 2 and 3 only
(c) 1 and 3 only
(d) 1, 2 and 3
Correct Answer is C (Golaknath Case – Constitutional amendment act also
comes under article 13.)

229) Which statement/statements is/are correct ?


1. 42nd amendment act makes the aid and advice by the Council
of Ministers binding on the president.
2. The advice given by the council cannot be inquired in court of
law.
3. A person who is not a member of the house cannot be
appointed as minister.
Which of the statements given above is are correct?
(1) only 3
(b) 1 and 2 only
(c) 1, 2 and 3
(d) None
Correct Answer is B (A person who is not a member of the house can be
appointed as minister.)

230) In which case concept of Basic Structure was introduced ?


(a) GolakNath case
(b) Shankari Prasad Case
(c) Keshvananda Bharati case
(d) Minerva Mill case
Correct Answer is C (Keshvananda Bharati case.)

231) Which of the following article describes India as “Union of States” ?


(a) Article 1
(b) Article 2
(c) Article 3
(d) Article 4
Correct Answer is A (Article 1)

232) Consider the following statements:


1. Indian federation system is based on Canadian Model.
2. Indian federation system has centralising tendencies.
3. Indian constitution has 25 parts and 465 articles.
Which of the above correctly matched ?
(a) 1 and 2
(b) 3 only
(c) 1, 2 and 3
(d) None
Correct Answer is C (All are Correct.)

233) Which statement/statements is/are correct ?


1. Part XI of the constitution deals with Centre-State relations.
2. Article 355 requires Centre to protect the states from external
aggression only.
3. Governor of any state is appointed by the President.
(a) 1 and 3
(b) 2 only
(c) 2 and 3
(d) 3 only
Correct Answer is A (Article 355 requires Centre to protect the states from
external aggression and internal disturbances.)

234) Which of the following part of our constitution deals with financial
relations of Centre and State ?
(a) Part XI
(b) Part XII
(c) Part XIII
(d) Part XIV
Correct Answer is XII (Where Part XI deals with the administrative
relations. Part XII deals with financial relations.)

235) Which of the following subject/subjects are covered under Centre-


State relations?
1. Territorial extent of centre-state legislations.
2. Distribution of legislative subjects.
3. Parliamentary legislations which fall in state field.
4. Centre’s control over state legislation.
Select the correct answer using the codes given below.
(a) 1, 2 and 4 only
(b) 2 only
(c) 1,3 and 4 only
(d) 1, 2, 3 and 4
Correct Answer is D (All the four subjects are covered under centre-state
relations.)

236) Constitution has imposed the financial limit on the taxing powers of
the states, what is the limit?
(a) The tax amount should not exceed 4500 per annum
(b) The tax amount should not exceed 3500 per annum
(c) The tax amount should not exceed 1500 per annum
(d) The tax amount should not exceed 2500 per annum
Correct Answer is D (The tax amount should not exceed 2500 per annum.)

237) Which is the following area where the states cannot impose the taxes ?
(a) Profession
(b) Sale of goods
(c) Newspaper
(d) Electricity
Correct Answer is C (The states cannot impose tax on newspapers.)

238) Which statement/statements is/are correct ?


1. President of Assam may direct that an act of parliament does
not apply to the autonomous tribal region or apply with some
modifications.
2. Governor of Mizoram, Tripura and Meghalaya may direct that
an act of parliament may not apply to respective tribal regions.
Select the correct answer using the codes given below.
(a) 1 only
(b) 2 only
(c) 1 and 2 only
(d) None of the above
Correct Answer is D (Governor of Assam may direct that an act of
parliament does not apply to the autonomous tribal region or apply with
some modifications. President of Mizoram, Tripura and Meghalaya may
direct that an act of parliament may not apply to respective tribal regions.)

239) Which statement/statements is/are correct ?


1. When there is a conflict between State list and Concurrent list
then the latter should prevail.
2. Rajya Sabha may ask the parliament to make laws on the
subject in the state list.
3. The resolution created on call of Rajya Sabha will remain in
force for a year and can be renewed once only.
Which of the statements given above is are correct?
(1) only 3
(b) 1 and 2 only
(c) 1, 2 and 3
(d) None
Correct Answer is B (The resolution can be renewed any number of times
but not exceeding one year at a time.)

240) Who can formulate an Inter-state council under Article 263 ?


(a) President
(b) Prime Minister
(c) Union Minister
(d) Chief Minister
Correct Answer is President ( President can create an inter-state council.)

241) Which of the following authority set up the Cabinet Committee?


(a) President
(b) Home Minister
(c) Prime Minister
(d) Cabinet Minister
Correct Answer is C (Prime Minister)

242) Consider the following statements:


1. Cabinet Committee is an extra-constitutional in nature.
2. Cabinet committee could be permanent or ad-hoc.
3. There are no rules for the establishment of Cabinet committee.
Which of the above correctly matched ?
(a) 1 and 2
(b) 3 only
(c) 1, 2 and 3
(d) None
Correct Answer is A (Rules of Business provide for the establishment of
Cabinet committee.)

243) Which statement/statements is/are correct ?


1. Prime Minister usually heads the Cabinet committee.
2. There are generally 3-8 members in cabinet committee.
3. Cabinet committees are good tool to reduce the enormous
workload.
(a) 1 and 2
(b) 2 only
(c) 1, 2 and 3
(d) 3 only
Correct Answer is C (All are Correct.)

244) Which of the following chair the Parliamentary Affair Committee?


(a) Cabinet Minister
(b) Prime Minister
(c) President
(d) Home Minister
Correct Answer is Home Minister (Parliamentary Affair Committee is
headed by Home Minister; However, the Political affairs committee is
headed by Prime minister which is the most powerful committee.)

245) Which statement/statements is/are correct ?


1. Group of Minister (GoM) is an ad-hoc body.
2. Parliament is discussed in part VI of the constitution.
3. In India, we follow the Westminister model of government.
Select the correct answer using the codes given below.
(a) 1 and 3 only
(b) 2 only
(c) 1 and 2 only
(d) 1only
Correct Answer is A (Parliament is discussed in part V of the constitution.)

246) Which of the following authority chair Economic Affairs committee


?
(a) Prime Minister
(b) President
(c) Home Ministers
(d) Governor
Correct Answer is A (Prime Minister)

247) Which amendment has extended the freeze over seat allocation in Lok
Sabha till 2026 ?
(a) 42nd amendment act
(b) 44th amendment act
(c) 82nd amendment act
(d) 84th amendment act
Correct Answer is 84th amendment act (42nd amendment act froze it till
2000. However, 84th amendment act froze it till 2026.)

248) Which statement/statements is/are correct ?


1. Only Delhi union territory has representatives in RajyaSabha.
2. The principle of single transferable vote is practised in
RajyaSabha.
3. Uttar Pradesh has the largest number of members in
Rajyasabha.
Select the correct answer using the codes given below.
(a) 1 only
(b) 2 and 3 only
(c) 1 and 3 only
(d) 1, 2 and 3
Correct Answer is B (Only Delhi and Puducherry UTs have representatives
in RajyaSabha.)

249) Which statement/statements is/are correct ?


1. In India, proportional representation principle is used in Rajya
Sabha.
2. In U.S., equal representation principle is practised.
3. There are 2 nominated members in Rajyasabha.
Which of the statements given above is are correct?
(1) only 3
(b) 1 and 2 only
(c) 1, 2 and 3
(d) None
Correct Answer is B (There are 12 nominated members in Rajyasabha.
There are 2 nominated members in Loksabha from anglo Indian
communities.)

250) Which year is demarcated as the year till which two nominated
members will be taken in Lok Sabha ?
(a) 2018
(b) 2019
(c) 2020
(d) 2025
Correct Answer is 2020 (It was originally decided to take the nominated
members for Lok Sabha till 1960; However, the same was extended till
2020 by the 95th amendment.)
251. Which of the following is/are true?
1. Institution of Ombudsman was first created in Sweden in the year
1908.
2. Ombud is a Swedish term which means the spokesperson or
representative of another person.
a) Only 1
b) None
c) Only 2
d) Both of the Above
D is the Correct Answer (Both are correct.)

252. Who administers the oath to speaker pro tem?


(a) Prime Minister
(b) Speaker
(c) President
(d) Deputy Speaker
Correct Answer is C (President)

253. Which of the following is/are true?


1. Ombudsman does not have power to take suo moto cases.
2. Ombudsman in India is called Lokpal.
a) Only 1
b) None
c) Only 2
d) Both of the Above
C is the Correct Answer (The suo moto power is available with
Ombudsman.)

254. Which of the following is/are true?


1. Administrative Reform Commission (ARC) recommended to
establish Lokpal And Lokayukta for grievance redressal.
2. This system is emulated from Sweden system of grievance
redressal.
a) Only 1
b) None
c) Only 2
d) Both of the Above
A is the Correct Answer (This system is emulated from New Zealand
system of grievance redressal.)

255. Which of the following is/are true?


1. ARC kept the judiciary within the ambit of Lokpal.
2. In Sweden, the judiciary is within the reach of Ombudsman.
a) Only 1
b) None
c) Only 2
d) Both of the Above
C is the Correct Answer (ARC exempted judiciary and excluded the same.)

256. Which of the following is/are true?


1. As per ARC, the Lokpal will be appointed by President after
consultation with the PM.
2. The Functionaries such as Speaker of Loksabha and Chairman of
Rajysabha will also be consulted.
a) Only 1
b) None
c) Only 2
d) Both of the Above
C is the Correct Answer (There is no role of Prime minister in ARC
recommendation.)

257. Which of the following is/are true?


1. As per Lokpal Bill 2011, the Investigation and Prosecution were
kept separate.
2. The constitutional status was also asked to be conferred on
Lokpal and Lokayukta under the Lokpal bill 2011.
a) Only 1
b) None
c) Only 2
d) Both of the Above
D is the Correct Answer (Both are correct.)

258. Which of the following is/are true?


1. As per 2011 Bill, the Prime Minister (with certain exceptions)
has also been included under the jurisdiction of Lokpal.
2. The Group A,B,C,D Government Servants are also included
under the jurisdiction of Lokpal.
a) Only 1
b) None
c) Only 2
d) Both of the Above
D is the Correct Answer (Both are correct.)

259. Which of the following is/are true?


1. When the Central Government is not able to pass the Lokpal bill
due to myriad reasons, various State governments have
established state version of Lokpal institution.
2. Maharashtra was the first state to pass such act.
a) Only 1
b) None
c) Only 2
d) Both of the Above
A is the Correct Answer (Odisha passed the act in 1970 and Maharashtra in
1971; However, Odisha was able to materialise this institution in 1983.)

260. Which of the following is/are true?


1. Recommendation of Lokayukta are binding over the state
government.
2. In U.P., Lokayukta does not have suo moto power.
a) Only 1
b) None
c) Only 2
d) Both of the Above
C is the Correct Answer (Recommendations of Lokayukta are not binding.)

261. Which of the following is/are true?


1. Article 112 talks about Annual Financial statement.
2. Budget shows the estimated figures of the following year.
a) Only 1
b) None
c) Only 2
d) Both of the Above
D is the Correct Answer (Both are correct.)

262. Which of the following is/are true?


1. Budget shows the revised figures of the ensuing year.
2. Budget shows the expenditures of previous year.
a) Only 1
b) None
c) Only 2
d) Both of the Above
C is the Correct Answer (Budget shows the revised figures of the current
year.)

263. Which of the following is/are true?


1. Government Budget consists of capital budget and revenue
budget.
2. Consolidated Fund of India depicts all the revenues received via
provident fund.
a) Only 1
b) None
c) Only 2
d) Both of the Above
A is the Correct Answer (Consolidated Fund of India depicts all the
revenues received via loans and advances.)

264. Which of the following is/are true?


1. In performance budgeting the budget is allocated on the basis of
targets achieved.
2. Zero based budgeting is time consuming.
a) Only 1
b) None
c) Only 2
d) Both of the Above
D is the Correct Answer (Both are correct.)

265. Which of the following is/are true?


1. The Annual Financial Statement is presented before LokSabha
only.
2. The parliament does not have the power to discuss and criticise
the budget.
a) Only 1
b) None
c) Only 2
d) Both of the Above
B is the Correct Answer (The Annual Financial Statement is presented
before both the houses of the parliament. The parliament has the power to
discuss and criticise the budget.)

266. Which of the following is/are true?


1. Emoluments and allowance of President are charged from
Consolidated fund of India.
2. Salary and allowances of Supreme Court judge are also charged
from Consolidated fund of India.
a) Only 1
b) None
c) Only 2
d) Both of the Above
D is the Correct Answer (Both are correct.)

267. Which of the following is/are true?


1. Demand for Grants made to Loksabha are voted upon and Lok
sabha can give its assent and refuse if it feels desirable.
2. President's assent is not required for introducing Demand for
Grant.
a) Only 1
b) None
c) Only 2
d) Both of the Above
A is the Correct Answer (President's assent is required for introducing
Demand for Grant.)

268. Which of the following is/are true?


1. Post Facto Parliament Authorization is required to withdraw
funds from Consolidated fund of India.
2. Article 266 deals with consolidated fund of India.
a) Only 1
b) None
c) Only 2
d) Both of the Above
C is the Correct Answer (Parliament Authorization is required to withdraw
funds from Consolidated fund of India before hand.)

269. Which of the following is/are true?


1. Article 266 deals with Public Accounts of India.
2. Article 267 deals with Contingency fund of India.
a) Only 1
b) None
c) Only 2
d) Both of the Above
D is the Correct Answer (Both are correct.)

270. Which of the following is/are true?


1. Contingency fund of India is imprest of the Parliament.
2. Post Facto approval of parliament is obtained if money is spent
from contingency fund.
a) Only 1
b) None
c) Only 2
d) Both of the Above
C is the Correct Answer (Contingency fund of India is imprest of the
President of India.)

271. Which of the following is/are true?


1. Provident Funds and money in other government trusts form part
of Consolidated Fund of India.
2. The money in public account needs to be paid back.
a) Only 1
b) None
c) Only 2
d) Both of the Above
C is the Correct Answer (Provident Funds and money in other government
trusts form part of Public Accounts of India.)

272. Which of the following is/are true?


1. Parliament Authorization is not required in case money is spent
out of Public Accounts.
2. When money is transferred from consolidated fund of India to
Public accounts for specific expenditure then the parliament's
approval becomes mandatory.
a) Only 1
b) None
c) Only 2
d) Both of the Above
D is the Correct Answer (Both are correct.)

273. Which of the following is/are true?


1. Revenue expenditure may create assets for the government.
2. Loan raised by the government is a part of government receipt.
a) Only 1
b) None
c) Only 2
d) Both of the Above
C is the Correct Answer (Capital expenditure may create assets for the
government.)

274. Which of the following is/are true?


1. Revenue expenditure helps in running day to day affairs of the
government.
2. Interest received by the government forms part of Capital
receipts.
a) Only 1
b) None
c) Only 2
d) Both of the Above
A is the Correct Answer (Interest received by the government forms part of
Revenue receipts)

275. Which of the following is/are true?


1. If an investment is made by government in shares then the same
comes under Capital Revenue.
2. The interest received on the above investment comes under
revenue receipts.
a) Only 1
b) None
c) Only 2
d) Both of the Above
C is the Correct Answer (If an investment is made by government in shares
then the same comes under Capital Expenditure.)

276. Which of the following is/are true?


1. The first part of the budget contains General economic Survey.
2. The second part of the budget contains taxation proposals.
a) Only 1
b) None
c) Only 2
d) Both of the Above
D is the Correct Answer (Both are correct.)

277. Which of the following is/are true?


1. Article 112 deals with Demands for grant.
2. Demand for grant need not to be voted by Lok Sabha
a) Only 1
b) None
c) Only 2
d) Both of the Above
B is the Correct Answer (Article 113 deals with Demands for grant.
Demand for grant needs to be voted by Lok Sabha.)

278. Which of the following is/are true?


1. Each Ministry presents separate demands for grant requests.
2. Demand for grant statement is presented along with the Annual
Financial Statement.
a) Only 1
b) None
c) Only 2
d) Both of the Above
D is the Correct Answer (Both are correct.)

279. Which of the following is/are true?


1. Budget is generally discussed on the day it is presented.
2. The day of discussion is determined by Speaker.
a) Only 1
b) None
c) Only 2
d) Both of the Above
C is the Correct Answer (Budget is generally not discussed on the day it is
presented..)

280. Which of the following is/are true


1. Motions can be moved to reduce demand for grant.
2. A cut motion can reduce the amount of demand to Rs. 1.
a) Only 1
b) None
c) Only 2
d) Both of the Above
D is the Correct Answer (Both are correct.)

281. Which of the following is/are true?


1. Article 164 says that chief minister shall be appointed by
Governor.
2. Procedure for appointment of CM is prescribed in the
constitution.
a) Only 1
b) None
c) Only 2
d) Both of the Above
A is the Correct Answer (No procedure for appointment of CM is
prescribed in the constitution)

282. Which of the following is/are true


1. Governor, generally, appoints the leader of majority party as
Chief Minister.
2. If Chief Minister dies then the ruling party can't elect new leader.
a) Only 1
b) None
c) Only 2
d) Both of the Above
A is the Correct Answer (If Chief minister dies then the ruling party can
elect new leader.)

283. Which of the following is/are true


1. It is possible that the governor first choose a Chief Minister and
then ask to prove his majority.
2. A person who is not a member of state legislature can't be elected
as CM.
a) Only 1
b) None
c) Only 2
d) Both of the Above
A is the Correct Answer (A person who is not a member of state legislature
can be elected as CM. But he/she needs to be elected within 6 months.)

284. Which of the following is/are true


1. CJI administers the oath of allegiance to the Chief Minister.
2. CM holds office during the pleasure of the president.
a) Only 1
b) None
c) Only 2
d) Both of the Above
B is the Correct Answer (Governor administers the oath of allegiance to the
Chief Minister. The office is held at the pleasure of Governor.)

285. Which of the following is/are true?


1. Ministers are appointed by the governor on the recommendation
of CM.
2. CM can ask a minister to resign.
a) Only 1
b) None
c) Only 2
d) Both of the Above
D is the Correct Answer (Both are correct.)

286. Which of the following is/are true?


1. CM can recommend the governor for the dissolution of the
assembly.
2. CM is the Vice chairman of the state planning board.
a) Only 1
b) None
c) Only 2
d) Both of the Above
A is the Correct Answer (CM is the chairman of the state planning board..)

287. Which of the following is/are true?


1. CM is the chief spokesman of the state government.
2. CM is the member of inter-state council
a) Only 1
b) None
c) Only 2
d) Both of the Above
D is the Correct Answer (Both are correct.)

288. Which of the following is/are true?


1. CM is the Chairman of National Development Council.
2. NDC is headed by Prime Minister.
a) Only 1
b) None
c) Only 2
d) Both of the Above
C is the Correct Answer (CM is a member of NDC, not the chairman.)

289. Which of the following is/are true?


1. Article 163 concerns Council of ministers aids and advice
Governor.
2. Article 164 conveys that ministers hold office on the pleasure of
the president.
a) Only 1
b) None
c) Only 2
d) Both of the Above
A is the Correct Answer (Article 164 conveys that ministers hold office on
the pleasure of the Governor.)

290. Which of the following is/are true?


1. Article 167 deals with duties of chief minister with respect to
Council of ministers.
2. Article 166 deals with conduct of business of the government of
state.
a) Only 1
b) None
c) Only 2
d) Both of the Above
C is the Correct Answer (Article 167 deals with duties of chief minister
with respect to Governor.)

291. Which of the following is/are true?


1. Council of ministers (CoM) is headed by Governor.
2. Article 163 dials with status of CoM.
a) Only 1
b) None
c) Only 2
d) Both of the Above
C is the Correct Answer (CoM is headed by Chief Minister.)

292. Which of the following is/are true?


1. Article 164 deals with appointment of Council of Minister.
2. As Per article I64 CoM is appointed by Governor.
a) Only 1
b) None
c) Only 2
d) Both of the Above
D is the Correct Answer (Both are correct.)

293. Which of the following is/are true?


1. Council of Minister can advise Governor.
2. Such advice can be inquired by court.
a) Only 1
b) None
c) Only 2
d) Both of the Above
A is the Correct Answer (Advice given by CoM can't be inquired by
court.)

294. Which of the following is/are true?


1. States such as MP, Odisha, Chhattisgarh and Jharkhand do have
separate ministers for tribal welfare.
2. Odisha is the latest entrant in the list.
a) Only 1
b) None
c) Only 2
d) Both of the Above
A is the Correct Answer (Jharknand is the latest entrant.)

295. Which of the following is/are true?


1. 91st amendment says that strength of CoM should not be
more than 15% of Strength of assembly.
2. Number of ministers including CoM Should not be less than
12.
a) Only 1
b) None
c) Only 2
d) Both of the Above
D is the Correct Answer (Both are correct.)

296. Which of the following is/are true?


1. CoM is collectively responsible to Legislative Assembly
2. Ministers hold office at the pleasure of Governor.
a) Only 1
b) None
c) Only 2
d) Both of the Above
D is the Correct Answer (Both are correct.)

297. Which of the following is/are true?


1. Article 167 mandates CM to Communicate information
concerning administration to CoM.
2. CoM, even when dissolved, shall aid and advice Governor.
a) Only 1
b) None
c) Only 2
d) Both of the Above
C is the Correct Answer (CM should communicate information to
Governor.)

298. Which of the following is/are true?


1. 94th amendment removed Madhya Pradesh from the Obligation
of having a tribal welfare minister.
2. 94th amendment added Jharkhand in the list of states which
should have tribal welfare minister.
a) Only 1
b) None
c) Only 2
d) Both of the Above
C is the Correct Answer (94th amendment removed BIHAR from the
list.)

299. Which of the following is/are true?


1. A person who is not a member of either house can still become
minister.
2. Minister who is not a member of house needs to become a
member of Lower house within 6 months
a) Only 1
b) None
c) Only 2
d) Both of the Above
A is the Correct Answer (He/she can choose to become member of either
house.)

300. Which of the following is/are true?


1. Sumptuary allowance is given to minister.
2. Members of state Legislature too get sumptuary Allowance.
a) Only 1
b) None
c) Only 2
d) Both of the Above
A is the Correct Answer (Sumptuary allowance is given to ministers
only.)
301. Which of the following is/are true?
1. In 1862, first three high courts were setup in Calcutta, Bombay
and Madras.
2. In 1866, 4th high court was setup in Delhi.
a) Only 1
b) None
c) Only 2
d) Both of the Above
A is the Correct Answer (In 1866, 4th high court was setup in Allahabad)
You can also join telegram channel for latest updates

Telegram Exam Group (For Updates on UPSC)

Telegram UGC NET Channel (UGC Material)

UGC NET Group (To Meet Other Aspirants and Free Test Series)

UPSC NET Group (To Meet Other Aspirants and Free Test
Series)

You can also enjoy our Youtube videos on our channel.

Don’t Forget to rate us on Amazon and claim a Free UPSC or UGC


(Any One) test series worth ₹ 1999 on our telegram group given
above.

302. Which of the following is/are true?


1. 7th amendment authorised parliament to demarcate one high
court for two or more states.
2. Territorial jurisdiction of high court is co-terminus with state
boundary.
a) Only 1
b) None
c) Only 2
d) Both of the Above
D is the Correct Answer (Both are correct.)

303. Which of the following is/are true?


1. Delhi is the only union territory which has its own high court
2. The country has 3 common high courts.
a) Only 1
b) None
c) Only 2
d) Both of the Above
D is the Correct Answer (Both are correct.)

304. Which of the following is/are true?


1. The Strength of high court is determined by Governor.
2. The appointment of high court judges is done by president after
consultation with chief justice
a) Only 1
b) None
c) Only 2
d) Both of the Above
C is the Correct Answer (The Strength of high court is determined by
President.)

305. Which of the following is/are true?


1. The appointment of chief Justice of high court is done by
President after consultation with Governor.
2. 10 years judicial service is one of the pre-conditions to become
high court judge.
a) Only 1
b) None
c) Only 2
d) Both of the Above
D is the Correct Answer (Judge of a High Court shall be appointed by the
President by warrant under his hand and seal after consultation with the
Chief Justice of India, the Governor of the State, and, in the case of
appointment of a Judge other than the chief Justice, the chief Justice of the
High court.)

306. Which of the following is/are true


1. Two judge case (1993) said that no high court judge will be
appointed unless there is conformity with Governor.
2. Three judge case said that CJl would consult a collegium of five
senior most supreme court judges to make high court
appointments of judges.
a) Only 1
b) None
c) Only 2
d) Both of the Above
B is the Correct Answer (Two judge case (1993) said that no high court
judge will be appointed unless there is conformity with chief justice of
India. Three judge case said that CJl would consult a collegium of two
senior most supreme court judges to make high court appointments of
judges.)

307. Which of the following is/are true


1. Oath of affirmation to high court judge is given by President.
2. A high court judge can hold an office until he attains the age of
62.
a) Only 1
b) None
c) Only 2
d) Both of the Above
C is the Correct Answer (Oath of affirmation to high court judge is given by
Governor.)

308. Which of the following is/are true?


1. President can remove a judge of high court on the
recommendation of parliament.
2. If a high court judge resigns, he can give his resignation to
president.
a) Only 1
b) None
c) Only 2
d) Both of the Above
D is the Correct Answer (Both are correct.)

309. Which of the following is/are true?


1. A high court judge can be removed by special majority of both
the houses.
2. Special majority implies majority of total membership of house
and not less than 2/3rd of the members present and voting.
a) Only 1
b) None
c) Only 2
d) Both of the Above
D is the Correct Answer (Both are correct.)

310. Which of the following is/are true


1. As A judge can be impeached on two grounds viz. misbehaviour
and Incapacity.
2. Judges Enquiry act of (1968) deals with appointment of high
court judge.
a) Only 1
b) None
c) Only 2
d) Both of the Above
A is the Correct Answer (judges Enquiry act of (1968) deals with removal
of high court judge.)

311) In how many days The Finance Bill must be enacted by the
parliament and assented by the president?
(a) 15 days
(b) 25 days
(c) 45 days
(d) 75 days
Correct Answer is D(According to the Provisional Collection of Taxes Act
of 1931, the Finance Bill must be enacted by the Parliament and assented to
by the president within 75 days)

312) Consider the following statements:


1. The budget consists of two types of expenditure.
2. The charged expenditure is votable.
3. The Charged Expenditure can only be discussed by the
Parliament.
Which of the above are correct?
(a) 1 and 3
(b) 3 only
(c) 1, 2 and 3
(d) None
Correct Answer is A(The charged expenditure is non-votable.)

313) Which of the following is/are charged expenditures?


1. Emoluments and allowances of the President.
2. Salaries and allowances of the Chairman and the Deputy
Chairman of Rajyasabha.
3. Salary, allowances and pension of the Comptroller and Auditor
General.
(a) 1 and 2
(b) 2 only
(c) 1, 2 and 3
(d) 3 only
Correct Answer is C
(All are Correct.)

314) Which motion is introduced to reduce any demand for grant?


(a) Adjournment motion
(b) Policy motion
(c) Token Cut
(d) Cut motion
Correct Answer is D(Cut motion)

315) Which statement/statements is/are correct?


1. Expenditure charged on the Consolidated Fund of India is not
submitted to the vote.
2. Voting of demands for grants is the exclusive privilege of the
Lok Sabha.
3. The demands are presented ministry wise.
4. Grant becomes a Demand after it has been duly voted.
Select the correct answer using the codes given below.
(a) 1 and 4 only
(b) 2 only
(c) 1,2 and 3 only
(d) 1, 2, 3 and 4
Correct Answer is C (Demand becomes a grant after it has been duly
voted.)

316) In which of the following Cut motion, the amount is reduced to Re 1?


(a) Token Cut
(b) Economic Cut
(c) Value CUt
(d) Policy Cut Motion
Correct Answer is D(Policy Cut Motion)

317) How much is the Vote on account with respect to total estimation?
(a) 1/6th of the total estimation
(b) 1/3rd of the total estimation
(c) 1/5th of the total estimation
(d) 1/7th of the total estimation
Correct Answer is 1/6th of the total estimation(It is usually granted for 2
months.)

318) Which statement/statements is/are correct?


1. The Cut motion does not have much utility in practice.
2. The Appropriation Bill becomes the Appropriation Act and
there is no need to seek an assent by the President.
3. The Government cannot withdraw money from the
Consolidated Fund of India till the enactment of the
appropriation bill.
Select the correct answer using the codes given below...
(a) 1 only
(b) 2 and 3 only
(c) 1 and 3 only
(d) 1, 2 and 3
Correct Answer is C (Appropriation Bill becomes the Appropriation Act
after it is assented to by the President.)

319) Which statement/statements is/are correct?


1. Grants are voted by the Lok Sabha.
2. No money shall be withdrawn from the Contingency Fund of
India except under appropriation made by law.
Which of the statements given above is are correct...
(a) only 2
(b) 1 only
(c) Both are Correct
(d) None
Correct Answer is B(No money shall be withdrawn from the Consolidated
Fund of India except under appropriation made by law.)

320) How many days are allotted for the voting of demands?
(a) 25 Days
(b) 26 Days
(c) 30 Days
(d) 10 Days
Correct Answer is B (26 Days)

320. Which of the following is/are true


1. The Appointment of district judge is done by governor with
consultation with High court.
2. A person who already holds seat in central state government can
be appointed as judge.
a) Only 1
b) None
c) Only 2
d) Both of the Above
A is the Correct Answer (He or she should not hold any office in
government.)

321. Which of the following is/are true


1. Minimum age to become a member of Legislative assembly is
30.
2. Minimum age to become a member of legislative council is 25.
a) Only 1
b) None
c) Only 2
d) Both of the Above
B is the Correct Answer (Age for assemble is 25 and for council it is 30.)

322. Which of the following is/are true


1. Person who is to be elected for assembly must be an elector for
the assembly constituency.
2. A member of SC/ST can also contest a non-reserved seat.
a) Only 1
b) None
c) Only 2
d) Both of the Above
D is the Correct Answer (Both are correct.)

323. Which of the following is/are true


1. Representation of People's act 1951 also describes qualifications
& disqualifications to the assembly.
2. RPA disqualifies a member from being chosen in council if the
person has any interest in government contracts.
a) Only 1
b) None
c) Only 2
d) Both of the Above
D is the Correct Answer (Both are correct.)

324. Which of the following is/are true


1. Disqualification is dealt under tenth schedule is subject to
judicial review.
2. The question of disqualification is decided by house in case of
assembly or council.
a) Only 1
b) None
c) Only 2
d) Both of the Above
A is the Correct Answer (The question of disqualification is decided by
speaker in case of assembly and chairman in case of council..)

325. Which of the following is/are true


1. Double membership is possible in Indian setup
2. Resignation will lead to vacation of seat and resignation can be
handed over to speaker or Chairman as the case may be.
a) Only 1
b) None
c) Only 2
d) Both of the Above
C is the Correct Answer (Double membership leads to vacation of seat.)

326. Which of the following is/are true


1. Speaker is the presiding officer for legislative assembly.
2. Deputy speaker is the presiding officer for legislative assembly.
a) Only 1
b) None
c) Only 2
d) Both of the Above
D is the Correct Answer (Dy. Speaker comes into picture when speaker is
not present.)

327. Which of the following is/are true


1. Absence for a period of 60 days could lead to Vacation of seat.
2. Absence from the house is determined by Governor.
a) Only 1
b) None
c) Only 2
d) Both of the Above
A is the Correct Answer (Absence from the house is determined by
speaker.)
328. Which of the following is/are true
1. Speaker can be removed by passing a resolution.
2. 14 days' notice is required to pass such resolution.
a) Only 1
b) None
c) Only 2
d) Both of the Above
D is the Correct Answer (Both are correct.)

329. Which of the following is/are true


1. Speaker cannot allow a secret meeting.
2. Speaker is the final interpreter of provisions of constitution of
India within the house.
a) Only 1
b) None
c) Only 2
d) Both of the Above
C is the Correct Answer (Speaker can allow a secret meeting.)

330. Which of the following is/are true


1. Deputy speaker can be removed by a resolution with an advance
notice of 14 days.
2. When both speaker and deputy speaker are not available then
anyone from the panel can preside.
a) Only 1
b) None
c) Only 2
d) Both of the Above
D is the Correct Answer (Both are correct.)

331. Which of the following is/are true


1. J&K handed over defence, external affairs and communication to
Indian state under instrument of accession.
2. Provisions concerning J&K are permanent in nature.
a) Only 1
b) None
c) Only 2
d) Both of the Above
A is the Correct Answer (These provisions are temporary in nature.)

332. Which of the following is/are true


1. Article 370 of part XXI, provides special state Status to J&K.
2. Part XXI Provides special status to J&K only.
a) Only 1
b) None
c) Only 2
d) Both of the Above
A is the Correct Answer (Part XXI Provides special status to 11 other states
as well.)

333. Which of the following is/are true


1. Judicial magistrate tackles Criminal cases where maximum
punishment is life imprisonment up to 3 years.
2. 20th constitutional amendment added article 233A. This article
retrospectively validated the appointment of certain district
judges.
a) Only 1
b) None
c) Only 2
d) Both of the Above
D is the Correct Answer (Both are correct.)

334. Which of the following is/are true


1. A session judge has the authority to impose life imprisonment but
not the capital punishment.
2. Capital punishment given by a session judge needs to be
confirmed by High court.
a) Only 1
b) None
c) Only 2
d) Both of the Above
C is the Correct Answer (A session judge has the authority to impose life
imprisonment and even capital punishment.)

335. Which of the following is/are true


1. When a judge deals with Civil cases, he is called session judge.
2. When a judge deals with criminal cases, he is called district judge
a) Only 1
b) None
c) Only 2
d) Both of the Above
D is the Correct Answer (Both are correct.)

336. Which of the following is/are true


1. District judge possesses the original and appellate jurisdiction.
2. A district judge cannot deal with Criminal cases.
a) Only 1
b) None
c) Only 2
d) Both of the Above
A is the Correct Answer (A district judge can deal with Criminal cases..)

337. Which of the following is/are true


1. Munsiff court belongs to civil side.
2. Judicial magistrate court belongs to criminal side.
a) Only 1
b) None
c) Only 2
d) Both of the Above
D is the Correct Answer (Both are correct.)

338. Which of the following is/are true


1. Appointment in judicial services, other than district judge, is
done by President in consultation with state public service
commission and High Court.
2. Organisational structure of subordinate judiciary is determined by
Supreme court.
a) Only 1
b) None
c) Only 2
d) Both of the Above
B is the Correct Answer (Appointment in judicial services, other than
district judge, is done by governor in consultation with state public service
commission and High Court. Organizational structure of subordinate
judiciary is determined by high court.)

339. Which of the following is/are true?


1. To be appointed as a district judge a person has to be a pleader or
advocate for 7 years.
2. A high court judge or a jurist is qualified to become judge of
Supreme Court.
a) Only 1
b) None
c) Only 2
d) Both of the Above
D is the Correct Answer (Both are correct.)

340. Which of the following is/are true


1. Appointment of district judge is done by President with
consultation with High court.
2. A person who already holds seat in central or state government
cannot be appointed as judge.
a) Only 1
b) None
c) Only 2
d) Both of the Above
C is the Correct Answer (Appointment of district judge is done by governor
with consultation with High court.)

341. Which of the following is/are true


1. As Per article 164, CoM is Individually responsible to
parliament.
2. In case of no confidence motion against CoM, all of them have to
resign.
a) Only 1
b) None
c) Only 2
d) Both of the Above
C is the Correct Answer (As Per article 164, CoM is collectively
responsible to legislative assembly..)

342. Which of the following is/are true


1. CoM cannot advice Governor to dissolve legislative assembly.
2. Principle of collective responsibility means that ministers need to
align with cabinet decisions.
a) Only 1
b) None
c) Only 2
d) Both of the Above
C is the Correct Answer (CoM can advise governor.)

343. Which of the following is/are true


1. Article 164 contains principle of Individual responsibility.
2. A minister can individually be removed If chief Minister so
desires.
a) Only 1
b) None
c) Only 2
d) Both of the Above
D is the Correct Answer (Both are correct.)

344. Which of the following is/are true


1. Constitution does specify size of the state council.
2. CoM categories viz. cabinet ministers, Ministers of States and
deputy Ministers.
a) Only 1
b) None
c) Only 2
d) Both of the Above
C is the Correct Answer (Constitution does not specify size of state
council..)

345. Which of the following is/are true


1. Cabinet Ministers are given departments such as Education and
Finance.
2. Ministers of state can be attached to Cabinet ministers but they
cannot be given independent charge.
a) Only 1
b) None
c) Only 2
d) Both of the Above
A is the Correct Answer (Ministers of state can be attached to Cabinet
ministers or they can be given independent charge.)

346. Which of the following is/are true


1. Ministers of state attend meetings of cabinet.
2. Cabinet meetings Can be attended by MoS, even If they are not
specifically invited.
a) Only 1
b) None
c) Only 2
d) Both of the Above
B is the Correct Answer (Ministers of state generally dont attend meetings
of cabinet. Cabinet meetings Can be attended by MoS, If they are
specifically invited.)

347. Which of the following is/are true


1. Deputy ministers are not given independent charge.
2. Deputy ministers can assist cabinet ministers.
a) Only 1
b) None
c) Only 2
d) Both of the Above
D is the Correct Answer (Both are correct.)

348. Which of the following is/are true


1. Cabinet can be called the real power Centre.
2. Cabinet can be called a decision making body.
a) Only 1
b) None
c) Only 2
d) Both of the Above
D is the Correct Answer (Both are correct.)

349. Which of the following is/are true


1. Cabinet committee could be of 2 types Standing and ad-hoc.
2. Cabinet does not have the power to review its decision.
a) Only 1
b) None
c) Only 2
d) Both of the Above
A is the Correct Answer (Cabinet has the power to review its decision..)

350. Which of the following is/are true?


1. Resignation of Chief Minister amounts to resignation of council.
2. In states, unlike centre, we don't have ministries but departments.
a) Only 1
b) None
c) Only 2
d) Both of the Above
D is the Correct Answer (Both are correct.)
351. Which of the following is/are true?
1. 97th constitutional amendment act gave constitutional status to
cooperative societies.
2. Right to form cooperative society is a fundamental right.
a) Only 1
b) None
c) Only 2
d) Both of the Above
D is the Correct Answer (Both are correct.)

352. Which of the following pairs are correctly matched?


1 243ZH Definitions
2 243ZI Incorporation of cooperative societies.
3 243ZP Returns
4 243ZQ Offences & Penalties
a) Only 1
b) None
c) Only 2
d) Both of the Above
D is the Correct Answer (All are correct.)

353. Which of the following is/are true?


1. Maximum number of directors in case of cooperative society
should not exceed 10.
2. In cooperative society one seat is reserved for SC or ST.
a) Only 1
b) None
c) Only 2
d) Both of the Above
C is the Correct Answer (the number of director should not exceed 21. In
cooperative society one seat is reserved for SC or ST and 2 seats are
reserved for women.)

354. Which of the following is/are true?


1. In cooperative society, Board can be suspended for a maximum
period of 6 months.
2. In case of suspension, the elections need to be conducted within 6
months.
a) Only 1
b) None
c) Only 2
d) Both of the Above
D is the Correct Answer (Both are correct.)

355. Which of the following is/are true?


1. Every Cooperative society shall be audited by the auditor
appointed by auditing firm.
2. The cooperative societies are exempted from filing returns.
a) Only 1
b) None
c) Only 2
d) Both of the Above
A is the Correct Answer (The cooperative societies are required to file
return within 6 months once the financial year ends.)

356. Which of the following is/are true?


1. The word cooperative society is given in Article 19.
2. Article 43 B talks about the professional management of
cooperative society.
a) Only 1
b) None
c) Only 2
d) Both of the Above
D is the Correct Answer (Both are correct.)
357. Which of the following is/are true?
1. Article 243ZJ refers to the number and terms of members of
cooperative society.
2. Article 243ZL talks about the election of member of board.
a) Only 1
b) None
c) Only 2
d) Both of the Above
A is the Correct Answer (Article 243ZK talks about the election of member
of board. Article 243ZL talks about the Suspension of member of board)

358. Which of the following is/are true?


1. Concerning cooperative societies the rights of members to get
information is given under 243ZO.
2. Under 243 ZM, the accounts of cooperative societies are audited.
a) Only 1
b) None
c) Only 2
d) Both of the Above
D is the Correct Answer (Both are correct.)

359. Which of the following is/are true


1. Cooperative Societies can be booked under the acts of omission
and not commission.
2. If the books of society are not handed over in the custody then
the same tantamount to an offense.
a) Only 1
b) None
c) Only 2
d) Both of the Above
C is the Correct Answer (Cooperative Societies can be booked under the
acts of omission and commission.)
360. Which of the following is/are true?
1. The Cooperative society board can be suspended for not
conducting elections within stipulated period.
2. The tenure of elected members of cooperative society is 4 years.
a) Only 1
b) None
c) Only 2
d) Both of the Above
A is the Correct Answer (The tenure of elected members of cooperative
society is 5 years.)

361. Which of the following is/are true?


1. Constitution amendment bill cannot be returned or rejected.
2. Money bill can be returned.
a) Only 1
b) None
c) Only 2
d) Both of the Above
Correct Answer is A (Money Bill can be rejected but not returned.)

362. Which of the following pairs are correctly matched ?


1 Article 33 Government can restrict the application of
certain rights on government servants.
2 311 (2) Reasonable opportunity is given to the civil
servant.
3 311 (2) Enquiry is not necessary in case of dismissal on
the ground of misconduct.
4 Article 33 The provision of restriction over Fundamental
Rights is taken form US constitution.
a) 1,2 and 3
b) 1,2,3 and 4
c) 2,3 and 4
d) None
A is the Correct Answer
"(The provision of restriction over Fundamental Rights is not taken form
US constitution..) "

363. Consider the following statements. Which statement/s is/are correct?


1. Council of ministers are collectively responsible to loksabha
2. PM is the leader of the lower house only.
3. PM recommends president regarding the summoning of the
house.
a) 1 and 3only
b) 2 only
c) 3 only
d) All of the above
Correct Answer is A

364. Which of the following is/are true?


1. PM is the principal channel of communication between president
and council of minister
2. President is the principal channel of communication between PM
and the Council of minister
a) Only 1
b) Only 2
c) Both of the above
d) Neither of the two
Correct Answer is A (Only 1)

365. Which of the following pairs are correctly matched?


1. Article 52 Executive power of the union
2. Article 54 Election of the president
3. Article 58 Qualification for presidential election
4. Article 61 Impeachment of the president
a) 1,2 and 3
b) 1,2,3 and 4
c) 2,3 and 4
d) None
Correct Answer is C (Article 52 is about that There shall be a President of
India.)

366. Consider the following statements


1. Salary and allowances of prime minister are determined by
Parliament.
2. Salary and allowances of prime minister are determined by
President.
3. A person who is member of neither house can be made prime
minister.
a) 1 and 3
b) 1,2 and 3
c) Only 2
d) None
Correct Answer is A (yes, the member of neither house can be made prime
minister for 6 months only.)

367. Consider the following statements


1. Vice president is the 2nd highest office of the country.
2. Both elected and nominated members of parliament participate in
vice president's elections.
3. State assemblies too participate in vice president's election
a) 1 and 2
b) 1,2 and 3
c) Only 3
d) None
Correct Answer is A (1 and 2)

368. Arrange chronologically the vice presidents of India


1. Krishan Kant
2. KR narayanan
3. BS Shekhawat
4. Hamid Ansari
a) 1, 3, 2, 4
b) 1,2,3, 4
c) 2, 1, 3, 4
d) 2,3,1,4
Correct Answer is C (2, 1, 3, 4)

369. Which of the following pairs are correctly matched?


1. Article 66 Election of vice president of india
2. Article 69 Oath and affirmation by vice president
3. Article 67 Term of office of vice president
4. Article 70 Discharge of presidential functions if need be
a) 1,2 and 3
b) 1,2,3 and 4
c) 2,3 and 4
d) None
Correct Answer is B (1,2,3 and 4)

370. Which of the following is/are true


1. President is the head of the government.
2. Prime minister is the head of the government.
a) Only 2
b) Both of the above
c) Only 1
d) Neither of the above
Correct Answer is A (President is the head of the state while Prime Minister
is the head of Government.)

371. Which of the following is/are true?


1. Regulation act of 1773 was the 1st step in the direction of
controlling east India company.
2. 1773 regulation act did not recognize the political and
administrative functions of company.
a) Only 1
b) None
c) Only 2
d) Both of the Above
Correct Answer is A (It did recognise the political and administrative
functions of the company.)

372. Which of the following is/are true?


1. GoI act 1935 has 321 articles and 12 schedules.
2. Residuary power was given to viceroy as per GoI 1935 act.
a) Only 2
b) Both of the above
c) Only 1
d) Neither of the above
Correct Answer is A (There were only 10 schedules.)

373. Consider the following statements. Which of them is/are correct?


1. Pitts India act is also called act of settlement.
2. Pitts India distinguished the political and economic functions of
the company.
3. A new body Board of control was created.
4. Company's territory for the first time was called British territory.
a) 1,2 and 3
b) 1,2,3 and 4
c) 2,3 and 4
d) None
Correct Answer is B ( 1,2,3 and 4 )

374. Which of the following is/are true?


1. Lord William Bentinck was the 1st Governor General of India.
2. Governor general of Bengal was made Governor general of India
by Pitts India act.
a) 1 and 2
b) Only 2
c) Only 1
d) Neither of the two
Correct Answer is C (Governor general of Bengal was made governor
general of India by charter act 1833.)

375. Which f the following is/are true?


1. 1773 regulation act made the governor of Bombay & Madras
subordinate to Governor of Bengal.
2. 1773 act authorized the establishment of Supreme Court in Delhi.
a) Only 1
b) Only 2
c) Both of the above
d) Neither of the two
Correct Answer is A ( Only 1)

376. Consider the following statements


1. Charter act of 1861 restored the power of madras and Bombay
presidencies.
2. 1861 act also established new legislative council for Bengal.
3. 1861 act recognised the portfolio system.
a) 1 and 2
b) 1,2 and 3
c) Only 3
d) None
Correct Answer is B (1, 2 and 3)

377. Which of the following is/are true?


1. Charter act of 1833 ended activities of east India company as a
commercial entity.
2. Charter act 1833 made an attempt to introduce the open
competition system in civil services.
a) Only 1
b) Only 2
c) Both of the above
d) Neither of the two
Correct Answer is C (Both of the above )

378. Which of the following is/are true?


1. The legislative and executive functions of governor general
council were separated by charter act 1853.
2. An open competition system in civil service got introduced as per
charter act 1853
a) None
b) Only 1
c) Both of the above
d) Only 2
Correct Answer is C (An attempt to introduce the open competition was
made in 1833 but the system finally got introduced in 1853.)

379. Which of the following pairs are correctly matched?


1. 1853 act Dual system of government (Dyarchy) was started
2. 1892 act Increased the non official members in council.
3. 1909 act Increased the size of legislative council.
4. 1919 act Divided the subjects into transferred and reserved
a) 1,2 and 3
b) 1,2,3 and 4
c) 2,3 and 4
d) None
Correct Answer is C (Dyarchy started in 1919 Montague chelmsford
reforms.)

380. Which of the following is/are true?


1. GoI Act 1919 introduced Bicameralism for the first time in the
country.
2. The principle of separate electorates was extended by 1919 act.
a) Only 2
b) Both of the above
c) Only 1
d) Neither of the above
Correct Answer is B (Both of the above)

381. Which of the following is/are true?


1. France constitution is rigid in the sense that Special procedure
(60% majority votes in both the House of Parliament) is required.
2. U.S. constitution is flexible as no special procedure is required
for Constitutional amendment.
a) Only 1
b) None
c) Only 2
d) Both of the Above
Correct Answer is A (In Britain, however, no special procedure is
required.)

382. Which of the following pairs are correctly matched?


1. Article 201 Withholding power of president
2. Article 123 Promulgation of ordinance
3. Article 72 Pardoning power of president
4. Article 161 Pardoning power of Governor
a) 1,2 and 3
b) 1,2,3 and 4
c) 2,3 and 4
d) None
Correct Answer is B ( 1,2,3 and 4 )

383 Consider the following statements. Which of them is/are correct?


1. 44th schedule can be subjected to judicial review if fundamental
rights under article 14,15,19 and 21 are being violated.
2. The same was determined under I.R. Coelho v. State of Tamil
Nadu
a) Only 1
b) None
c) Only 2
d) Both of the Above
Correct Answer is C (9th schedule can be subjected to judicial review if
fundamental rights under article 14,15,19 and 21 are being violated.)

384. Which of the following is/are true?


1. In U.S., The residuary powers are vested in the states
2. In France, local governments are created and abolished by central
government.
a) Only 1
b) Only 2
c) Both of the above
d) Neither of the two
Correct Answer is C (both correct )

385. Which of the following is/are true?


1. Part XI contains Finance, property and suits.
2. Part IX of our constitution consists – The Panchayats.
a) Only 1
b) Only 2
c) Both of the above
d) Neither of the two
Correct Answer is B ( Part XI – Relations between the Union and the
States. Part XII is about finance, property and suits. )

386. Consider the following statements


1. Indian constitution was framed under Cabinet mission plan 1946.
2. Indian Constitution contains a preamble, 22 parts and 12
schedules.
3. Part IX-B of our constitution consists – The Municipalities.
a) 1 and 2
b) 1,2 and 3
c) Only 3
d) None
Correct Answer is A (Part IXB – The Cooperative Societies.)

387. Which of the following is/are true?


1. In Japan, both the houses are directly elected.
2. Just like india, in Japan PM and cabinet are answerable to
Parliament.
a) Only 1
b) Only 2
c) Both of the above
d) Neither of the two
Correct Answer is C (Both Correct.)

388. Which of the following is/are true?


1. The lower house in Germany is called Bundestag.
2. In China, The Legislative Body is called Duma
a) Only 1
b) Only 2
c) Both of the above
d) Neither of the two
Correct Answer is A ( In Russia, The Legislative Body is called Duma)
389. Which of the following is/are true?
1. France's constitution is called Constitution of the Fifth Republic
2. In Russia, President is elected for four year term
a) Only 1
b) None
c) Only 2
d) Both of the Above
Correct Answer is A (In Russia, President is elected for six year term.)

390. Which of the following is/are true?


1. Term constitution comes through French from the Latin word
constitutio.
2. Constitutional monarchy with a parliamentary system is a
prominent feature of American System.
a) Only 2
b) Both of the above
c) Only 1
d) Neither of the above
Correct Answer is C (Constitutional monarchy with a parliamentary system
is a feature of British system.)

391) Which Of the following article talks about Inter-State water disputes?
(a) Article 260
(b) Article 261
(c) Article 262
(d) Article 263
Correct Answer is Article 262 (There has been 8 Inter state water tribunal
which have been established so far.)

392) Vansdhara Water dispute Tribunal involve which of the following


states?
1. Odisha
2. Andhra Pradesh
3. Bihar
Which of the above are correctly matched ?
(a) 1 and 2
(b) 3 only
(c) 1, 2 and 3
(d) None
Correct Answer is A (Bihar is not included.)

393) Which of the following states are included in Mahadayi water


disputes?
1. Goa
2. Karnataka
3. Maharashtra
(a) 1 and 2
(b) 2 only
(c) 1, 2 and 3
(d) 3 only
Correct Answer is C (The inter-state tribunal was established in 2010.)

394) Which of the following is also called Constitutional emergency?


(a) National Emergency
(b) Presidents Rule
(c) Financial Emergency
(d) External Emergency
Correct Answer is B (Presidents rule is also called state emergency or
Constitutional emergency. It has been imposed numerous times.)

395) Which statement/statements is/are correct ?


1. There are five zonal Council.
2. North-east council is in addition to the five zonal counsels.
3. Uttar Pradesh falls in the central zonal Council
4. Uttarakhand falls in the North zonal Council.
Select the correct answer using the codes given below.
(a) 1, 2 and 3 only
(b) 2 only
(c) 1,3 and 4 only
(d) 1, 2, 3 and 4
Correct Answer is A (Uttarakhand falls in the central zonal Council.)

396) Which of the following Article talks about Inter-state Council?


(a) Article 263
(b) Article 264
(c) Article 265
(d) Article 266
Correct Answer is Article 263 (Article 263)

397) Which of the following commission established on the issue of


Centre-State relations?
(a) BN Krishanan Commission
(b) Sarkaria Commission
(c) Bachwat Commission
(d) Dhar Commission
Correct Answer is Sarkaria Commission (Sarkaria Commission)

398) Which statement/statements is/are correct ?


1. Prime Minister is the chairman of Inter-state Council
2. Inter-state Council was first established by Janta Dal
Government.
3. Inter-state Council is the recommendatory body.
Select the correct answer using the codes given below.
(a) 1 only
(b) 2 and 3 only
(c) 1 and 3 only
(d) 1, 2 and 3
Correct Answer is D (All are Correct.)

399) Which statement/statements is/are correct ?


1. Inter-state Council is assisted by a Inter-state Council
secretariat.
2. Standing committee of the Council assists the Interstate
Council.
3. Prime Minister is the chairman of the standing committee of
the Council.
Which of the statements given above is are correct?
(1) only 3
(b) 1 and 2 only
(c) 1, 2 and 3
(d) None
Correct Answer is B (Home Minister is the chairman of the standing
committee of the Council.)

400) which one of the following is the chairman of Zonal Council?


(a) Prime minister
(b) Home Minister
(c) Chief Minister
(d) Finance Minister
Correct Answer is Home Minister (Home Minister)
401. Which of the following is/are true?
1. Fundamental rights are given under part III of the constitution
2. DPSPs are given under part IV of the constitution.
a) Only 1
b) None
c) Only 2
d) Both of the Above
D is the Correct Answer "Both are correct”

You can also join telegram channel for latest updates

Telegram Exam Group (For Updates on UPSC)

Telegram UGC NET Channel (UGC Material)

UGC NET Group (To Meet Other Aspirants and Free Test Series)

UPSC NET Group (To Meet Other Aspirants and Free Test
Series)

You can also enjoy our Youtube videos on our channel.

Don’t Forget to rate us on Amazon and claim a Free UPSC or UGC


(Any One) test series worth ₹ 1999 on our telegram group given
above.

402. which of the following pairs are correctly matched ?


1. Article 38 Welfare
2. Article 41 Right to Work
2. Article 42 Bonded labour
3. Article 43 Adequate wages
a) Only 1
b) Only 2
c) 1, 2 and 3
d) All are correct
D is the Correct Answer "All Options are Correct "

403. Consider the following statements. Which of them is/are correct ?


1. DPSPs are justiciable in nature.
2. Articles 36-51 deal with DPSPs.
a) Only 1
b) None
c) Only 2
d) Both of the Above
C is the Correct Answer “DPSPs are non justiciable. One cannot go to court
to get them enforced”

404. Which of the following is/are true


1. Article 44 talks about rights of women.
2. Article 45 talks about early childhood care.
a) Only 1
b) Only 2
c) Both of the above
d) Neither of the two
B is the Correct Answer "Article 44 talks about uniform civil code."

405. Which of the following is/are true


1. Article 47 concerns with public health.
2. Article 48 deals with animal husbandry.
a) Only 1
b) Only 2
c) Both of the above
d) Neither of the two
C is the Correct Answer "Both are Correct"

406. Consider the following statements


1. Article 45 is a liberal directive principle.
2. Article 40 is a Gandhian principle.
3. Article 47 is a socialist principle.
a) 1 and 2
b) 1,2 and 3
c) Only 3
d) None
A is the Correct Answer "Article 47 can be cited as a Gandhian and liberal
principle"

407. Which of the following is/are true


1. Article 40 demands the establishment of Gram Panchayats.
2. Article 43 demands the formation of cottage industries.
a) Only 1
b) Only 2
c) Both of the above
d) Neither of the two
C is the Correct Answer "Both are Correct"

408. Which of the following is/are true


1. Article 47 deals with banning slaughtering of Cows.
2. Article 48 deals with banning consumption of liquor.
a) Only 1
b) Only 2
c) Both of the above
d) Neither of the two
D is the Correct Answer. (Article 48 bans slaughtering of cow and 47 deals
with banning liquor consumption")

409. Which of the following is/are true ?


1. 86th constitutional amendment act introduced RTE.
2. Article 21B was introduced via 86th amendment.
a) Only 1
b) None
c) Only 2
d) Both of the Above
A is the Correct Answer"Article 21A was introduced"

410. Which of the following is/are true


1. Article 51 encourages international peace.
2. Article 48A protects the environment.
a) Only 2
b) Both of the above
c) Only 1
d) Neither of the above
B is the Correct Answer. "Both are Correct"

411. Which of the following is/are true


1. Part XIV of the constitution deals with civil services rules.
2. Chapter I of part XIV contains articles such as 309, 310, 311
and 312.
a) Only 1
b) None
c) Only 2
d) Both of the Above
D is the Correct Answer " Both are Correct "

412. which of the following pairs are correctly matched ?


1 Article Supreme court can pass orders which are necessary to do
142 complete justice
2 Article S.C. can give special leave for appeal from any court or
136 tribunal.
3 Article No limitation regarding the cases in which the power is
142 exercised.
4 Article An appeal may lie even for interlocutory order.
136
a) 1,2 and 3
b) 1,2,3 and 4
c) 2,3 and 4
d) None
B is the Correct Answer " All options are correct "

413. Consider the following statements. Which of them is/are correct with
respect civil services?
1. Article 309 deals with recruitment and conditions of service
of persons serving the Union or a State Subject.
2. Article 310 deals with conditions of service.
a) Only 1
b) None
c) Only 2
d) Both of the Above
B is the Correct Answer " 309 deals with condition of service and 310
deals with tenure of office. Remember TENure and 310 "

414. Which of the following is/are true


1. Article 311 provides safeguard to the people employed in the
service.
2. Article 312 talks about the creation of new All India Service.
a) Only 1
b) Only 2
c) Both of the above
d) Neither of the two
C is the Correct Answer " parliament can create legislation to create new
all India services under article 312 "

415. Which of the following is/are true


1. Chapter I of part XIV deals with services.
2. Chapter II of part XIV deals with public service commission.
a) Only 1
b) Only 2
c) Both of the above
d) Neither of the two
C is the Correct Answer " chapter I deals with article 308 to 313 and
Chapter II deals with article 315 to 323 "

416. Consider the following statements


1. Article 311(2) which places restriction on the pleasure is
applicable only to the Civil services.
2. The expression civil means an appointment, office or
employment on the civil side of the administration which is
distinguished from military side.
3. IFoS is a central service.
a) 1 and 2
b) 1,2 and 3
c) Only 3
d) None
A is the Correct Answer " IFoS is an all India service "

417. Which of the following is/are true


1. Our civil service model is based on British model.
2. Article 310 of Indian constitution incorporates the common
law doctrine of pleasure.
a) Only 1
b) Only 2
c) Both of the above
d) Neither of the two
C is the Correct Answer " both are correct "

418. Which of the following is/are true


1. Person employed in civil services under article 311(1) shall
not be dismissed by an authority subordinate to the authority by
which they were appointed.
2. Article 311(2) is silent about minor punishment.
a) Only 1
b) Only 2
c) Both of the above
d) Neither of the two
C is the Correct Answer "both are correct "

419. which of the following is/are true ?


1. With respect to all India service penalty, dismissal and removal
or compulsory retirement can be imposed by GoI not by the state
government.
2. A person who is retired can claim right under article 31.
a) Only 1
b) None
c) Only 2
d) Both of the Above
A is the Correct Answer " A person who is retired can't claim right under
article 31 "

420. Which of the following is/are true


1. Dismissal from service disqualifies a person from further
reemployment in government.
2. Removal from service does not disqualify a person from
reemployment in government.
a) Only 2
b) Both of the above
c) Only 1
d) Neither of the above
D is the Correct Answer "Both of the Above"

421. Which of the following is/are true


1. Supreme Court judge does hold office on the pleasure of president.
2. Election commissioner does not require pleasure of the president.
a) Only 1
b) None
c) Only 2
d) Both of the Above
C is the Correct Answer". (Even SC judge does not require the pleasure of
president.) "

422. How many days are allocated for Short Duration Discussion per week?
(a) 2 days
(b) 3 days
(c) 4 days
(d) 5 days
Correct Answer is A (2 days)

423. Consider the following statements. Which of them is/are correct ?


1. In case of discharge from service opportunity to defend is extended.
2. In the case of Dismissal or Removal, no opportunity to defend is
extended
a) Only 1
b) None of the above.
c) Only 2
d) Both of the Above
B is the Correct Answer"(discharge does not give opportunity to defend but
dismissal and removal give the same.) "

424. Which of the following is/are true


1. As per Article 311(2), enquiry for dismissal is not necessary when the
interest of the security of state is involved.
2. As per Article 311(2) enquiry is not necessary when it is impracticable to
give civil servant an opportunity to defend himself.
a) Only 1
b) Only 2
c) Both of the above
d) Neither of the two
C is the Correct Answer"(both are correct.) "

425. Which of the following is/are true


1. In disciplinary proceedings where doctrine of proof beyond reasonable
doubt does not apply, we apply principle of preponderance of probabilities.
2. Article 311 does not apply on probationers when removed on ground of
unsuitability.
a) Only 1
b) Only 2
c) Both of the above
d) Neither of the two
C is the Correct Answer"(both options are correct.) "

426. Consider the following statements


1. Article 148 provides for independent office of CAG.
2. Article 149 authorises parliament to describe the power and duties of
CAG.
3. Article 150 prescribes the forms of account.
a) 1 and 2
b) 1,2 and 3
c) Only 3
d) None
B is the Correct Answer"(All options are correct. There is article 151 which
is about submission of audit report.) "

427. Which of the following is/are true


1. CAG can be removed by both the houses with simple majority.
2. CAG can hold office for 6 years or up to the age of 65. whichever is
earlier.
a) Only 1
b) Only 2
c) Both of the above
d) Neither of the two
B is the Correct Answer"(CAG is removed by special majority.) "

428. Which of the following is/are true


1. The salary of CAG is equal to the judge of supreme court.
2. CAG salary cannot be altered in his disadvantage.
a) Only 1
b) Only 2
c) Both of the above
d) Neither of the two
C is the Correct Answer"(both options are correct.) "

429. which of the following is/are true ?


1. CAG audits the accounts of union and state governments.
2. The accounts of local bodies are not audited by CAG in any situation.
a) Only 1
b) None
c) Only 2
d) Both of the Above
A is the Correct Answer"(CAG has to audit the accounts of local bodies if
requested by the president.) "

430. Which of the following is/are true


1. CAG is not responsible for maintenance and compilation of accounts.
2. CAG submits the audit report to the president.
a) Only 2
b) Both of the above
c) Only 1
d) Neither of the above
B is the Correct Answer "(CAG was relieved from the responsibilities of
maintenance in 1976.) "

431. Which of the following is/are true


1. Appropriation accounts compare the actual expenditure with the
sanctioned ones.
2. Apart from appropriation account, CAG also submits report on
finance accounts and public accounts to the president.
a) Only 1
b) None
c) Only 2
d) Both of the Above
D is the Correct Answer"(Both the options are correct.) "

432. which of the following pairs are correctly matched ?


1 ONGC CAG does the audit
2 LIC Audit is done by private Audit is done by private auditors
auditors.
3 IFC Audit is done by private auditors and
CAG together
4 SBI Audit is done by CAG
a) 1,2 and 3
b) 1,2,3 and 4
c) 2,3 and 4
d) None
A is the Correct Answer"(SBI, LIC and RBI's audits are done by private
auditors. ) "

433. Consider the following statements. Which of them is/are correct ?


1. CAG can also go for Propriety audit.
2. Propriety audit is discretionary.
a) Only 1
b) None
c) Only 2
d) Both of the Above
D is the Correct Answer"(both options are correct.) "

434. Which of the following is/are true


1. The CAG office in India is similar to that of in Britain.
2. CAG is responsible for both comptrolling and auditing
a) Only 1
b) Only 2
c) Both of the above
d) Neither of the two
D is the Correct Answer "(CAG in India is different from britain. The
reason being in India, comptrolling function is not under the ambit of CAG,
but in britain it is. In India, if money needs to be withdrawn from
consolidated fund of India, the permission of CAG is not required.) "

435. Which of the following is/are true


1. Indian forest service was created in 1966.
2. IFoS is the coveted central service.
a) Only 1
b) Only 2
c) Both of the above
d) Neither of the two
A is the Correct Answer "(IFoS is an all India service.) "

436. Consider the following statements


1. AIS (All India Service) is controlled by Centre and State.
2. Central service is under the exclusive jurisdiction of centre government.
3. The central services are divided into Group A,B,C and D.
a) 1 and 2
b) 1,2 and 3
c) Only 3
d) None
B is the Correct Answer "(All options are correct.) "

437. Which of the following is/are true


1. Sardar Vallabh Bhai Patel is regarded as father of all India services.
2. The officers of AIS and central services are Gazetted officers.
a) Only 1
b) Only 2
c) Both of the above
d) Neither of the two
C is the Correct Answer "(Group A and B belong to the gazetted class.) "

438. Which of the following is/are true


1. In Government, Gazetted class are called officers.
2. Non-Gazetted class called employees.
a) Only 1
b) Only 2
c) Both of the above
d) Neither of the two
C is the Correct Answer"(Both are correct.) "

439. which of the following is/are true ?


1. Principle of pleasure applies to Civil and Military posts.
2. No compensation is given when a government post is abolished.
a) Only 1
b) None
c) Only 2
d) Both of the Above
A is the Correct Answer"(The compensation provision exists given the fact
that no misconduct is involved.) "

440. Which of the following is/are true


1. Article 311 provides safeguard to the civil servants.
2. Article 311 puts restrictions on the principle of pleasure.
a) Only 2
b) Both of the above
c) Only 1
d) Neither of the above
B is the Correct Answer"(Both are correct.) "

441. Which of the following first suggested setting up of Lokpal and


Lokyukta?
(a) Administrative Reforms Commission (ARC)
(b) Government of India act 1947
(c) Constitution of India
(d) None of the above
Correct Answer is A
(Administrative Reforms Commission (ARC))

442. Which of the following Institute or institutes work under Central


Vigilance Commission?
1. CVC Secretariat
2. Chief Technical Examiners Wing
3. Commissioners for Departmental Enquiries
Which of the above correctly matched?
(a) 1 and 3
(b) 3 only
(c) 1, 2 and 3
(d) None
Correct Answer is C
(All work under Central Vigilance Commission.)

443. Which of the following is not included in the motto of CBI?


1. Industry
2. Impartiality
3. Intelligence
(a) 1 and 2
(b) 2 only
(c) 1, 2 and 3
(d) 3 only
Correct Answer is A
(It is Integrity, not Intelligence.)

444. Who is the head of the committee which recommends the


appointment of director of CBI?
(a) Prime Minister
(b) Home Minister
(c) President
(d) Chief Vigilance Commissioner
Correct Answer is D
(Chief Vigilance Commissioner)

445. Which statement/statements is/are correct?


1. Special police Establishment is a division of CBI
2. CBI Academy is located at Ghaziabad.
3. CBI comes under the administrative control of Ministry of
Home Affairs.
Select the correct answer using the codes given below.
(a) 1 only
(b) 2 only
(c) 3 only
(d) 1 and 2
Correct Answer is D
(CBI comes under the administrative control of Department of personnel
and training (DoPT).)

446In which country the institution of ombudsman was first created?


(a) US
(b) Singapore
(c) Sweden
(d) Switzerland
Correct Answer is Sweden
(In 1809, the first institution of ombudsman was created in Sweden. The
word Ombud in Swedish means a person who acts as a spokesman all
representative of another person.)

447. Where does the CBI derive its power?


(a) Constitution
(b) CBI act
(c) From People
(d) Delhi Special Police Establishment Act 1946
Correct Answer is Delhi Special Police Establishment Act 1946
(CBI is not a statutory body. It derives its power from Delhi Special Police
Establishment Act 1946.)

448. Which statement/statements is/are correct?


1. CVC is a statutory body.
2. CBI was set up by a resolution of Ministry of home affairs.
3. CBI was established as per the recommendation of Santhanam
committee
Select the correct answer using the codes given below...
(a) 1 only
(b) 2 and 3 only
(c) 1 and 3 only
(d) 1, 2 and 3
Correct Answer is D
(All are correct.)

449. Which statement/statements is/are correct?


1. CVC has the power of a civil court.
2. CVC presents annually report of its performance to the
Parliament.
3. All the ministries and departments of the union government
have a Chief vigilance officer.
Which of the statements given above is are correct...
(a) only 2
(b) 1 and 2 only
(c) 1 and 3
(d) None
Correct Answer is C
(CVC presents annually report of its performance to the President.)

450. Where the headquarters of CVC is located?


(a) Kolkata
(b) Mumbai
(c) Chennai
(d) New Delhi
Correct Answer is D
(New Delhi)
451) how many sub rights are given under article
19?
(a) 5
(b) 6
(c) 7
(d) 8
Correct Answer is 6 (Six sub rights are there under Article 19.)

452) consider the following statements...


1. When a national emergency is declared on the grounds of
armed rebellion that is called internal emergency.
2. 44th Amendment act curtail the power of Prime Minister in
imposing emergency.
3. As per 38th Amendment act 1975, Judicial review is not
permissible with respect to national emergency
Which of the above correctly matched?
(a) 1 and 2
(b) 3 only
(c) 1, 2 and 3
(d) None
Correct Answer is C (However, 44th Amendment act Deleted the
provision of judicial review given in 13th Amendment act. Now, after
Minerva Mills case the emergency can be questioned in the court on the
basis of malafide.)

453) Which statement/statements is/are correct ?


1. Proclamation of emergency must be approved by both the
houses viz. Lok Sabha and Rajya Sabha.
2. It must be passed in two months of proclamation.
3. If an emergency is passed By both the houses it continues for
6 Months.
(a) 1 and 2
(b) 2 only
(c) 1 and 3
(d) 3 only
Correct Answer is C (It must be passed in one month of proclamation.The
emergency can be extended up to an indefinite period but has to be
approved every six months. The 44th Constitution Amendment act added
the provision of periodic Approval.)

454) Which of the following majority is required for proclamation or


continuation of emergency?
(a) Either house of Parliament by a Simple majority
(b) Either house of Parliament by a Special majority
(c) Upper house of Parliament by a special majority
(d) Lower house of Parliament by a special majority
Correct Answer is B (A majority of total membership of that house along
with Not less than 2/3rd of the member of the house present voting.)

455) Which statement/statements is/are correct ?


1. President can revoke the emergency at any time.
2. 1/10th of the total member of Lok Sabha can also give notice
to Speaker to consider a resolution which disapproves the
continuation of proclamation.
3. Resolution of approval of continuation has to be passed by
both the Houses of Parliament with special majority
4. Resolution of disapproval needs to be passed by the Lok
Sabha only and that too with simple majority.
Select the correct answer using the codes given below.
(a) 1, 2 and 4 only
(b) 2 only
(c) 1,3 and 4 only
(d) 1, 2, 3 and 4
Correct Answer is D (All are correct)
456) The life of the Lok Sabha can be extended beyond the usual five years
for ______ year at a time?
(a) 6 Months
(b) 1 year
(c) 3 Months
(d) 2 years
Correct Answer is B(This extension can go any length of time. But, once
the emergency ceases to operate then Lok Sabha cannot continue for a
period beyond 6 months.)

457) Which of the following article deals with suspension of fundamental


rights given in article 19?
(a) Article 355
(b) Article 356
(c) Article 357
(d) Article 358
Correct Answer is D (Article 359, on the other hand, deals with other
fundamental rights which can be suspended except article 19 which is
suspended by article 358, and article 20 and 21 which are not suspended.)

458) Which statement/statements is/are correct ?


1. Article 19 cannot be suspended only on the grounds of
external aggression or War.
2. Article 19 can be suspended in the case of armed rebellion.
3. Laws which are related to emergency and the executive
actions taken related to such laws are protected.
Select the correct answer using the codes given below.
(a) 1 and 3 only
(b) 2 and 3 only
(c) 1 and 3 only
(d) 1, 2 and 3
Correct Answer is A (Article 19 can be suspended only on the grounds of
external aggression or War. Article 19 cannot be suspended in the case of
armed rebellion.)

459) Which statement/statements is/are correct?


1. Article 358 uses with article 19, and article 359 deals with rest
of the fundamental rights.
2. Article 358 automatically suspends article 19 while other
rights are not automatically suspended by article 359
3. Article 358 extends to entire country while 359 may or may
not extend to the entire country.
Which of the statements given above is are correct?
(a) only 3
(b) 1 and 2 only
(c) 1, 2 and 3
(d) None
Correct Answer is (All are Correct.)

460) How many times The National Emergency has been declared?
(a) One Time
(b) Two times
(c) Four Times
(d) Three times
Correct Answer is Three times. (1962, 1971 and 1975 were the years when
the national emergency was proclaimed. Whereas the first two one on the
grounds of conflict with China and Pakistan, the third one was because of
internal disturbances.)

461) Which of the following article makes it a duty of centre that every
State shall be carried out as per the Constitution ?
(a) Article 355
(b) Article 358
(c) Article 357
(d) Article 356
Correct Answer is A (Article 355)

462) Consider the following statements:


1. The President's rule should be approved by both Loksabha
and Rajysabha within 2 months.
2. Once approved, the President's rule continues for 1 year.
3. The Presidents rule can be extended up to 3 years, and not
indefinitely as in the case of LokSabha.
Which of the above correctly matched ?
(a) 1 and 2
(b) 3 only
(c) 1 and 3
(d) None
Correct Answer is C (The 44th Amendment act of 1978 impose restrictions
on extension of proclamation of Presidents rule beyond One year. The
provision was added that it has to be extended by six months at a time given
the two conditions. First, national emergency is in existence in the entire
country and election commission must certify that the legislative assemblies
election cannot be executed.)

463) Which statement/statements is/are correct with respect to the


President's rule?
1. The functions of state government can be acquired by the
President.
2. President runs the State's administration directly.
3. Parliament can be given the power of State's legislature
(a) 1 and 3
(b) 2 only
(c) 2 and 3
(d) 3 only
Correct Answer is A (President runs the administration through the
Governor of the State.)
464) What is the maximum period for which a national emergency can stay
the operation?
(a) 1 Year
(b) 3 Years
(c) 6 Months
(d) Indefinitely
Correct Answer is D (President's rule can stay in operation for maximum
three years, it can be extended in exceptional circumstances. But National
emergency can run indefinitely.)

465) Which statement/statements is/are correct ?


1. 38th constitutional amendment act of 1975 made article 356
unchallengeable in any court.
2. 44th Constitutional Amendment act deleted the provision
introduced by 38th Amendment act.
3. The President's rule is subjective to judicial review is also
determined in Bommai case.
Select the correct answer using the codes given below.
(a) 1, 2 and 3 only
(b) 2 only
(c) 1,2 and 4 only
(d) 1, 2, 3 and 4
Correct Answer is D (All are Correct.)

466) Which of the following is proper use of imposition of President's Rule


?
(a) Not allowing ministry to prove its majority
(b) Internal disturbances not amounting to physical breakdown
(c) Allegations of corruption against the ministry
(d) Hung Parliament
Correct Answer is D (Hung Parliament)

467) Financial Emergency proclamation must be approved by ..... ?


(a) Loksabha
(b) Rajyasabha
(c) Both the Houses
(d) President Only
Correct Answer is Both the Houses (Financial Emergency can run
indefinitely, there is no maximum period. It can be revoked by President at
any time he/she thinks is suitable.)

468) Which statement/statements is/are correct ?


1. Resolution for Financial Emergency can be passed by simple
majority.
2. During financial emergency salaries of judges can be
reduced.
3. In the financial emergency, repeated parliamentary approval is
required.
Select the correct answer using the codes given below.
(a) 1 only
(b) 2 and 3 only
(c) 1 and 3 only
(d) 1and 2
Correct Answer is D (In financial emergency, repeated parliamentary
approval is not required.)

469) Which statement/statements is/are correct with respect to President's


Rule ?
1. 42nd Amendment Act of 1976 raised the period of 6 months
to one year.
2. 44th Amendment Act of 1978 again reduced the period to 6
months.
3. President’s Rule imposed in Punjab (1987) was allowed to
continue for 5 years under the 68th Amendment Act of 1991.
Which of the statements given above is are correct?
(a) only 3
(b) 1 and 2 only
(c) 1, 2 and 3
(d) None
Correct Answer is C (All are Correct)

470) Which of the following amendment introduced the terminology Armed


Rebellion replacing Internal Disturbances?
(a) 46th Constitutional Amendment Act
(b) 41th Constitutional Amendment Act
(c) 44th Constitutional Amendment Act
(d) 45th Constitutional Amendment Act

Correct Answer is 44th Constitutional Amendment Act


(44th Constitutional Amendment Act)

471) Which of the following does not participate in the presidential


elections?
(a) Loksabha
(b) Rajyasabha
(c) Legislative Assemblies of states
(d) Legislative Council of states
Correct Answer is D (Only Legislative Assemblies of states participate.)

472) Consider the following statements:


1. All disputes in connection with the election of the President
are decided by the Supreme Court whose decision is final.
2. President is a representative of the states only.
3. In India, a President can be appointed any number of times.
Which of the above correctly matched?
(a) 1 and 3
(b) 3 only
(c) 1, 2 and 3
(d) None
Correct Answer is A (President a representative of the Union and the states
equally.)

473) Which statement/statements is/are correct ?


1. A 14 days notice is required to start the impeachment charges
against the president.
2. These charges for resolution of impeachment should be signed
by 1/4th of the members.
3. 2/3rd of the total members should pass the resolution to make
it move to the other house.
(a) 1 and 2
(b) 2 only
(c) 1, 2 and 3
(d) 3 only
Correct Answer is C (Once the charges are moved to the other house, there
the charges are investigated. The impeachment process of the president is a
quasi-judicial process.)

474) Which of the following does not participate in the impeachment of the
president?
(a) Loksabha
(b) Rajya sabha
(c) Nominated members of Parliament
(d) Legislative Assemblies
Correct Answer is Legislative Assemblies (All the members of the
parliament can participate in the impeachment process of the president.)

475) Which statement/statements is/are correct with respect to president's


legislative power?
1. President nominates 12 members in Rajya Sabha.
2. President may give his assent or may withhold one with
respect to ordinary bills.
3. The prior recommendation of the President is not required in
the case of the money bill.
Select the correct answer using the codes given below.
(a) 1 and 2 only
(b) 2 only
(c) 3 only
(d) 1, 2 and 3
Correct Answer is A ( The prior recommendation of the President is
required in the case of the money bill. With respect to financial powers, the
money bill can be introduced with a prior recommendation of the President.
Furthermore, it is the president who lays down the budget before the
Parliament and he is the one who recommends for demand for grant.)

476) Which of the following authority constitutes financial commission?


(a) President
(b) Home Minister
(c) Prime Minister
(d) Finance Minister
Correct Answer is President (President constitutes financial commission
once in 5 years. The commission's task is to distribute the revenue between
Centre and State.)

477) Which of the following authority can appoint Inter-state Council so


that inter-state relations, as well as the relations between Centre and
State could be promoted?
(a) Prime Minister
(b) Home Minister
(c) President
(d) Vice President
Correct Answer is President (President)

478) Which statement/statements is/are correct ?


1. The ordinance is given under article 123.
2. The ordinance has to be approved by the Parliament within 6
weeks of its reassembly.
3. The maximum life of an ordinance is 6 months.
Select the correct answer using the codes given below.
(a) 1 only
(b) 2 and 3 only
(c) 1 and 2 only
(d) 1, 2 and 3
Correct Answer is C (The maximum life of an ordinance is 6 months and 6
weeks.)

479) Which statement/statements is/are correct ?


1. President has the power to pardon, commute,
remission, respite and reprieve.
2. President has the power with respect to court-martial.
3. President is the Supreme Commander of defence forces.
Which of the statements given above is are correct?
(a) only 3
(b) 1 and 2 only
(c) 1, 2 and 3
(d) None
Correct Answer is C (The President is the authority which appoints the
chiefs of Army, Navy and air force. If Parliament approves, he can also
declare the war or peace.)

480) Which of the following veto is not available with President of India?
(a) Absolute veto
(b) Qualified veto
(c) Suspense veto
(d) Pocket veto

Correct Answer is Qualified veto


(In Indian system, there is no qualified veto. however, the same is given to
American President.)

481) In which of the following cases absolute veto can be exercised?


(a) Private members Bill
(b) Government Bills
(c) Money Bills
(d) Constitutional Amendment Bills

Correct Answer is Private members Bill


(Absolute veto can also be exercised in the case of when the Cabinet resigns
after the passage of the bill but before the president's assent. Similarly, the
other case could be when a new cabinet advises not to give the assent to the
President.)

482) Consider the following statements:


1. Suspensive veto is exercised in the form when the bill is
returned for reconsideration.
2. If the bill comes back to the President, he is supposed to give
his assent.
3. Concerning money bills, suspensive veto is not available with
the President.
Which of the above correctly matched?
(a) 1 and 2
(b) 3 only
(c) 1, 2 and 3
(d) None
Correct Answer is C(After reconsideration when the bill comes to the
President and he has to give his assent therefore we can say that the re-
passaging of the bill has an overriding power over the presidential veto.)

483) Which statement/statements is/are correct?


1.The pocket veto may keep the bill pending for an indefinite
period.
2.There is no such time limit provided by the Constitution in the
case of pocket veto.
3. With respect to the constitutional amendment bills, President
has veto power.
(a) 1 and 2
(b) 2 only
(c) 2 and 3
(d) 3 only
Correct Answer is A
(With respect to the constitutional amendment bills, President does not have
any veto power.)

484) Which of the following amendment made the President's satisfaction


final and conclusive with respect to ordinance?
(a) 44th constitutional amendment act
(b) 42nd constitutional amendment act
(c) 38th constitutional amendment act
(d) 39th constitutional amendment act
Correct Answer is 38th constitutional amendment act
(This Constitution amendment put the President's decision out of the
purview of judicial review. However, the 44th amendment changed it all.)

485) Which statement/statements is/are correct?


1. Money bill can be rejected.
2. Money bill cannot be returned.
3. A Constitutional amendment bill can be rejected and can be
returned.
Select the correct answer using the codes given below.
(a) 1 and 2 only
(b) 2 only
(c) 1 only
(d) 1, 2 and 3
Correct Answer is A
(A money bill can be rejected but cannot be returned. Constitutional
amendment bill neither can be rejected nor can be returned)

486) When an ordinance cannot be promulgated?


(a) When Both the houses are in session
(b) When Loksabha is not in session
(c) When Rajyasabha is not in session
(d) When Both the houses are not in session
Correct Answer is A
(When Both the houses are in session.)

487) In which among the following pardoning power the punishment is


reduced without changing its character?
(a) Pardon
(b) Remission
(c) Respite
(d) Commute
Correct Answer is Remission
(However, In Respite, the lesser form of punishment is given all special
grounds like pregnancy or physical disability. Similarly, in commutation
lighter form of punishment is given for instance imprisonment can be given
in place of death sentence.)

488) Which statement/statements is/are correct?


1. An ordinance can be retrospective.
2. Ordinance cannot be issued to amend the Constitution.
3. The maximum gap between the two parliamentary sessions
can be six months.
Select the correct answer using the codes given below...
(a) 1 only
(b) 2 and 3 only
(c) 1 and 3 only
(d) 1, 2 and 3
Correct Answer is D
(All are Correct.)

489) Which statement/statements is/are correct?


1. Article 161 confers the pardoning power to the Governor.
2. Like President, the Governor can pardon the death sentence.
3. Governor can only suspend, remit or commute the death
sentence.
Which of the statements given above is are correct...
(a) only 3
(b) 1 and 3 only
(c) 1, 2 and 3
(d) None
Correct Answer is B
(Unlike President, the governor cannot pardon the death sentence.)

490) Which of the following advices the President when there is a question
of exercising pardoning power?
(a) Prime Minister
(b) Home Minister
(c) Cabinet
(d) Council of Minister

Correct Answer is Cabinet


(In case of pardoning, the President can take a different view than the court
but he needs to listen Cabinet.)
491) Article 358 works in which of the following case?
(a) Internal Emergency
(b) External emergency
(c) President's Rule
(d) None of the above
Correct Answer is B
(External emergency)

492) Consider the following statements:


1. Article 358 suspends the fundamental right given under
article 19.
2. Article 359 suspends the fundamental rights other than article
19, 20 and 21.
3. In India, the National emergency has been imposed twice so
far.
Which of the above correctly matched?
(a) 1 and 2
(b) 3 only
(c) 1, 2 and 3
(d) None
Correct Answer is A
(In India the National emergency has been imposed thrice so far. The first
emergency was imposed during India-China war in 1962 then in 1971 and
finally in 1975. The third and final emergency was proclaimed on the
grounds of internal disturbances.)

493) Which statement/statements is/are correct?


1. Article 355 mandates the centre to ensure that the State
government should run as per the Constitution.
2. National emergency must be passed by special majority.
3. The President's rule must be passed by simple majority.
(a) 1 and 2
(b) 2 only
(c) 1, 2 and 3
(d) 3 only
Correct Answer is C
(The president rule can run for a maximum 3 years. it has been imposed
several times. The condition of its extension is when the election
commission agrees that the situation is not germane for conducting an
election in the state.)

494) Which of the following Constitutional Amendment act made the


president bound by the advise of Council of ministers which is headed by
the Prime Minister?
(a) 42nd constitutional amendment act
(b) 38th constitutional amendment act
(c) 24th constitutional amendment act
(d) 45th constitutional amendment act
Correct Answer is A
(42nd constitutional amendment act)

495) Which statement/statements is/are correct?


1. President enjoys situational discretion
2. President enjoys constitutional discretion
Select the correct answer using the codes given below.
(a) 1 only
(b) 2 only
(c) Both
(d) None of the Above
Correct Answer is A
(President does not enjoy constitutional discretion.)
496) How many proposers and seconders are required for the nomination of
a candidate in the presidential election?
(a) 10 proposers 10 seconders
(b) 20 proposers 20 seconders
(c) 50 proposers 50 seconders
(d) 100 proposers 50 seconders
Correct Answer is C
(50 proposers, 50 seconders)

497) On which lines the Vice President's office is modeled in India ?


(a) British Vice President
(b) Canadian Vice President
(c) American Vice President
(d) Australian Vice President
Correct Answer is American Vice President
(Vice President is accorded the rank just next to the President. However,
when compared with American Vice President, he does not enjoy much
power. In America, the Vice President succeeds President if the presidential
post is vacant however in India the Vice President can act as a president for
a maximum period of six months and then the presidential elections are
organised.)

498) Which statement/statements is/are correct with respect to situational


discretion of President?
1. President can appoint Prime Minister when nobody has a clear
majority
2. President cannot dismiss the council even when it fails to
prove the confidence in the Lok Sabha
3. When the council loses its majority in the Lok Sabha then the
President can dissolve the Lok Sabha.
Select the correct answer using the codes given below...
(a) 1 only
(b) 1 and 3 only
(c) 1 and 3 only
(d) 1, 2 and 3
Correct Answer is B (President can also dismiss the council when it fails to
prove the confidence in the Lok Sabha.)

499) Which statement/statements is/are correct concerning election of the


Vice President?
1. He should be a citizen of India
2. He should not hold any office of profit under local
government, state government or union government
3. He should be qualified for election as a member of Rajya
Sabha
Which of the statements given above is are correct...
(a) only 3
(b) 1 and 2 only
(c) 1, 2 and 3
(d) None
Correct Answer is C (All are Correct.)
500) How many proposers and seconders are
required for the nomination of a candidate in Vice
presidential election?
(a) 20 proposers 20 seconders
(b) 10 proposers 10 seconders
(c) 50 proposers 50 seconders
(d) 100 proposers 50 seconders
Correct Answer is A (20 proposers 20 seconders)

You can also join telegram channel for latest updates

Telegram Exam Group (For Updates on UPSC)

Telegram UGC NET Channel (UGC Material)

UGC NET Group (To Meet Other Aspirants and Free Test Series)

UPSC NET Group (To Meet Other Aspirants and Free Test
Series)

You can also enjoy our Youtube videos on our channel.

Don’t Forget to rate us on Amazon and claim a Free UPSC or UGC


(Any One) test series worth ₹ 1999 on our telegram group given
above.
INDIAN POLITY
PART -II
1) How many members Committee on Papers
Laid on the Table consists?
(a) 22
(b) 25
(c) 9
(d) 12
Correct Answer is 25
(Library committee consists 9 members. While Committee on Papers
Laid on the Table consists 25 members, Lok Sabha Committee has 15
members, while the Rajya Sabha Committee has 10 members )

2) Consider the following statements:


1. Business Advisory Committee has 26 members.
2. Committee on Private Members' Bills and Resolutions
exists only in Loksabha.
3. Committee on Private Members' Bill does not exist in
Rajyasabha.
Which of the above correctly matched?
(a) 1 and 2
(b) 3 only
(c) 1, 2 and 3
(d) None
Correct Answer is A
(Committee on Private Members' Bill does not exist in Rajyasabha. This
job is performed by the Business Advisory Committee of Rajya Sabha.)

3) Which statement/statements is/are correct with respect to Rules


Committee?
1. Rules Committee has 31 members. 15 members including
the Speaker as its ex-officio chairman.
2. There are 16 members who come from Rajyasabha in
Rules committee.
3. There are 15 members from Loksabha in Rules
committee.
(a) 1 and 2
(b) 2 only
(c) 1, 2 and 3
(d) 3 only
Correct Answer is C
(All are Correct.)

4) How many members are there in Committee on Welfare of SCs and


STs?
(a) 30
(b) 25
(c) 31
(d) 12
Correct Answer is 30
(This committee consists of 30 members (20 from Lok Sabha and 10
from Rajya Sabha).)

5) Which statement/statements is/are correct?


1. House Committee In the Lok Sabha, it consists of 12
members.
2. Library committee consists of 9 members, six from Lok
Sabha and three from Rajya Sabha.
3. Joint Committee on Salaries and Allowances of Members
consist of 15 members.
4. Consultative Committees are constituted by Ministry of
Parliamentary Affairs.
Select the correct answer using the codes given below.
(a) 1 and 4 only
(b) 2 only
(c) 1,3 and 4 only
(d) 1, 2, 3 and 4
Correct Answer is D
(The membership of Consultative committees is voluntary and is left to
the choice of the members and the leaders of their parties. The
maximum membership of a committee is 30 and the minimum is 10.)

6) How many members are there in Committee on Empowerment of


Women?
(a) 30
(b) 20
(c) 25
(d) 31
Correct Answer is 30 members
(This committee was constituted in 1997 and consists of 30 members (20
from Lok Sabha and 10 from Rajya Sabha).)

7) How many members are there in Joint Committee on Offices of Profit?


(a) 10
(b) 15
(c) 20
(d) 25
Correct Answer is 15 members
(Joint Committee on Offices of Profit has 10 from Lok Sabha and 5 from
Rajya Sabha.)

8) Which statement/statements is/are correct?


1. Other judges, after consultation with the chief justice, are
appointed by president.
2. The judges of the Supreme Court are appointed by the
president.
3. A person should have been a judge of a High Court for
five years to be eligibile for the judge of supreme court.
Select the correct answer using the codes given below...
(a) 1 only
(b) 2 and 3 only
(c) 1 and 3 only
(d) 1, 2 and 3
Correct Answer is D
(All are Correct.)

9) Which statement/statements is/are correct?


1. Parliamentary Fora does not interfere with or encroach
upon the jurisdiction of the DRSC- Departmentally Related
Standing Committees.
2. In Forum on Population and Public Health, the Chairman
of Rajya Sabha is the President and the Speaker is the Co-
President.
3. Each Forum consists of not more than 20 members
(excluding the President and ex-officio Vice-Presidents) out
of whom not more than 10 are from the Lok Sabha and not
more than 10 are from the Rajya Sabha.
Which of the statements given above is are correct...
(a) only 2
(b) 1 and 2 only
(c) 1, 2 and 3
(d) None
Correct Answer is B
( Speaker of Lok Sabha is the President of all the Forums except the
Parliamentary Forum on Population and Public Health wherein the
Chairman of Rajya Sabha is the President and the Speaker is the Co-
President. Each Forum consists of not more than 31 members
(excluding the President and ex-officio Vice-Presidents) out of whom not
more than 21 are from the Lok Sabha and not more than 10 are from
the Rajya Sabha.)
10) How many members DRSC consists?
(a) 10
(b) 20
(c) 21
(d) 31
Correct Answer is 31
(Departmentally Related Standing Committees consists of 31 members –
21 from Lok Sabha and 10 from Rajya Sabha.)

11) Which of the following fund is used to meet unforeseen expenditure?


(a) State Fund
(b) Public Accounts of India
(c) Consolidated Fund Of India
(d) Contingency Fund of India
Correct Answer is Contingency Fund of India
(Contingency Fund of India)

12) Consider the following statements:


1. Supplementary Grant is authorised by the Parliament
through the appropriation act.
2. Excess Grant is approved by the Estimate Committee of
Parliament.
3. Vote of Credit is granted for meeting an unexpected
demand upon the resources of India.
Which of the above correctly matched?
(a) 1 and 3
(b) 3 only
(c) 1, 2 and 3
(d) None
Correct Answer is A
(Excess Grant is approved by the Public Accounts Committee of
Parliament.)
13) Which statement/statements is/are correct?
1. Token Grant demand for the grant of a token sum of
Rupee 100 is submitted to the vote of the Lok Sabha.
2. Public Accounts of India consists Provident fund deposits,
judicial deposits, savings bank deposits, departmental
deposits, remittances.
(a) 1 only
(b) 2 only
(c) None of the above
(d) Both are correct
Correct Answer is B
(Token Grant demand for the grant of a token sum of Rupee 1 is
submitted to the vote of the Lok Sabha.)

14) Who is the final authority to decide whether a bill is money bill or
not?
(a) President
(b) Chief Minister
(c) Speaker
(d) CJI
Correct Answer is Speaker
(Speaker has the final say on determining whether a bill is money bill or
not.)

15) Which statement/statements is/are correct?


1. Ministers are collectively responsible to the Parliament in
the Lok Sabha.
2. Council of ministers can be removed from office by the
Lok Sabha by passing a no-confidence motion.
3. The major part of the Constitution can be amended by the
Parliament with the simple majority.
Select the correct answer using the codes given below.
(a) 1 only
(b) 2 only
(c) 1 and 2 only
(d) All are correct
Correct Answer is C
(The major part of the Constitution can be amended by the Parliament
with the special majority.)

16) Under which article, Financial bill can be introduced only in the
Lok Sabha and not in the Rajya Sabha?
(a) Article 110
(b) Article 112
(c) Article 113
(d) Article 111
Correct Answer is A
(Article 110.)

17) In how many days, money bill should be returned by Rajyasabha to


Loksabha?
(a) 2 months
(b) 14 days
(c) 7 Days
(d) 1 month
Correct Answer is B
(The Rajyasabha should return the bill to Loksabha within 14 days.)

18) Which statement/statements is/are correct?


1. The constitution can be amended in 3 different ways
which are by Simple Majority, Special a Majority
and special majority but with the consent of half of all the
state legislatures.
3. State legislature can pass a resolution requesting the
parliament for the creation or abolition of the legislative
council.
Select the correct answer using the codes given below...
(a) 1 only
(b) 2 only
(c) None of the above
(d) Both are correct
Correct Answer is D
(Both are correct.)

19) Which statement/statements is/are correct?


1. Rajya Sabha elects its Deputy Speaker.
2. Lok Sabha elects its Speaker and Deputy Speaker.
3. The Parliament participates in the election of the
President along with the state legislative assemblies.
Which of the statements given above is are correct...
(a) only 2
(b) 1 and 2 only
(c) 2 and 3 only
(d) None
Correct Answer is C
(Rajya Sabha elects its Deputy Chairman.)

20) Which of the following authority can create or abolish the state
legislative councils on the recommendation of the concerned state
legislative assemblies?
(a) Speaker
(b) President
(c) Parliament
(d) Prime Minister
Correct Answer is C
(Parliament.)
21) Which amendment says that the Governor can be appointed for two
or more states?
(a) 7th Constitutional Amendment
(b) 5th Constitutional Amendment
(c) 6th Constitutional Amendment
(d) 8th Constitutional Amendment
Correct Answer is A
(7th Constitutional Amendment Act of 1956 refers that the appointment
of the same person as a governor for two or more states.)

22) Consider the following statements:


1. Original jurisdiction of the Supreme Court with regard to
federal disputes is different from its original jurisdiction
with regard to disputes relating to fundamental rights
2. The Supreme Court can issue writs only for the
enforcement of the Fundamental Rights and others.
3. Writ jurisdiction of the Supreme court is narrower than
that of the High Court.
Which of the above correctly matched?
(a) 1 and 3
(b) 3 only
(c) 1, 2 and 3
(d) None
Correct Answer is A
(The Supreme Court can issue writs only for the enforcement of the
Fundamental Rights.)

23) Which statement/statements is/are correct?


1. Supreme Court is authorised to grant in its discretion
special leave to appeal from any judgement in any matter
passed by any court or tribunal in the country
2. Constitution authorises the president to seek the opinion
of the Supreme Court.
3. The Scope of judicial review in India is narrower than
that of what exists in USA.
(a) 1 and 2
(b) 2 only
(c) 1, 2 and 3
(d) 3 only
Correct Answer is C
(All are correct.)

24) What was the method of Governor's election recommended in


Draft constitution?
(a) Universal adult suffrage
(b) List System
(c) Appointment by President
(d) Appointment by Parliament
Correct Answer is A
(Draft Constitution provided for the direct election of the governor on
the basis of universal adult suffrage.)

25) In which among the following case/cases, Supreme Court enjoys a


wide appellate jurisdiction?
1. Appeals in constitutional matters.
2. Appeals in Civil matters.
3. Appeals in criminal matters.
4. Appeals in Special leave matters.
Select the correct answer using the codes given below.
(a) 1 and 4 only
(b) 2 only
(c) 1,3 and 4 only
(d) 1, 2, 3 and 4
Correct Answer is D
(Supreme Court has not only succeeded the Federal Court of India but
also replaced the British Privy Council as the highest court of appeal.)

26) Who does the President consult while appointing Governor of any
state?
(a) Council of Minister
(b) Prime Minister
(c) Chief Minister
(d) Speaker
Correct Answer is C
(While appointing the governor, the president is required to consult
the chief minister of the state concerned, so that the smooth functioning.)

27) How many months notice is given while initiating a civil


proceedings?
(a) 4 Months
(b) 2 Months
(c) 1 Months
(d) 3 Months
Correct Answer is 2 Months
(Governor is immune from any criminal proceedings, even in respect of
his personal acts. He cannot be arrested or imprisoned. However, civil
proceedings can be instituted against him after giving 2 months' notice,
during his term of office in respect of his personal acts.)

28) Which statement/statements is/are correct?


1. The Supreme Court ‘must' tender its opinion to the
president when there is a question of fact.
2. The Supreme Court may tender or may refuse to tender its
opinion to the president in the question of Law.
Select the correct answer using the codes given below...
(a) 1 only
(b) 2 only
(c) Both are correct
(d) None of the above
Correct Answer is C
(The opinion of the Supreme Court is not binding on the president.)

29) As a Court of Record, which of the following powers the Supreme


Court has?
1. Judgements, proceedings and acts of the Supreme Court
are recorded for perpetual memory and testimony.
2. Power to punish for contempt of court
Which of the statements given above is are correct...
(a) only 2
(b) 1 and 2 only
(c) 1,only
(d) None
Correct Answer is B
(Both are correct.)

30) Which of the following does not come under The Original
jurisdiction of court?
(a) Dispute between Centre and State
(b) Dispute between One state and other
(c) Dispute among states
(d) Dispute between India and Foreign territory
Correct Answer is D
(Its exclusive original jurisdiction extends to any dispute between the
Government of India and one or more States or between the
Government of India and any State or States on one side and one or
more States on the other or between two or more States)

31) Which of the following institute is regarded as the watchdog of


merit system by Constitution?
(a) CEC
(b) CAG
(c) CVC
(d) UPSC
Correct Answer is D
(Constitution visualises the UPSC to be the ‘watch-dog of merit system'
in India.)

32) Consider the following statements:


1.The entry of Central Vigilance Commission (CVC) has
affected the role of UPSC in disciplinary matters.
2.UPSC is an independent constitutional body.
3. CVC is a constitution body.
Which of the above statement is/are correct
(a) 1 and 2
(b) 3 only
(c) 1, 2 and 3
(d) None
Correct Answer is A
(UPSC is a constitution body which has an edge over the CVC which is
a statutory body.)

33) In 1985, a new full-fledged Ministry of Personnel, Public Grievances


and Pensions was created with three separate departments, which of the
following is or are the departments included?
1. Department of Personnel and Training
2. Department of Administrative Reforms and Public
Grievances
3. Department of Pensions and Pensioners' Welfare
(a) 1 and 2
(b) 2 only
(c) 1, 2 and 3
(d) 3 only
Correct Answer is C

(All are correct.)


34) Which of the following authority can extend the jurisdiction of
SPSC?
(a) State legislature
(b) High Court
(c) Supreme Court
(d) President
Correct Answer is State legislature
(The jurisdiction of SPSC can be extended by an Act made by the state
legislature.)

35) Which statement/statements is/are correct?


1.A State Public Service Commission consists of a chairman
and other members appointed by the governor of the state.
2. Constitution also authorises the governor to determine the
conditions of service of the chairman and members of the
Commission.
3. Chairman and members of a SPSC are appointed by the
governor And can be removed by governir.
4. Chairman and members of the Commission hold office
for a term of six years or until they attain the age of 62
years.
Select the correct answer using the codes given below.
(a) 1, 2 and 4 only
(b) 2 only
(c) 1,3 and 4 only
(d) 1, 2, 3 and 4
Correct Answer is A

(Chairman and members of a SPSC are appointed by the governor and


can be removed only by the president.)
36) Which of the following authority is described as watchdog of merit
system by constitution?
(a) SPSC
(b) CAG
(c) CVC
(d) CEC
Correct Answer is SPSC
(Constitution visualises the SPSC to be the ‘watchdog of merit system'
in the state.)

37) Which of the following state pair had JSPSC?


(a) Punjab Haryana
(b) Punjab Himachal
(c) Punjab Uttarakhand
(d) Punjab RAJASTHAN
Correct Answer is A
(Punjab and Haryana had a JSPSC for a short period, However, after
the creation of Haryana out of Punjab in 1966 the setup got changed.)

38) Which statement/statements is/are correct?


1. Article 280 of the Constitution of India provides for a
Finance Commission.
2. Finance Commission consists of a chairman and four
other members to be appointed by the president.
3. Chairman of Finance a commission should be a person
having experience in public affairs.
Select the correct answer using the codes given below...
(a) 1 only
(b) 2 and 3 only
(c) 1 and 3 only
(d) 1, 2 and 3
Correct Answer is D
(All are correct.)

39) Finance Commission is required to make recommendations to the


president of India on the following matters?
1. Distribution between the Union and the States of the net
proceeds of taxes
2. Recommend measures needed to augment the
Consolidated Fund of a State to supplement the resources of
the Panchayats.
3. Recommend principles which should govern the grants-
in-aid of the revenues of the States out of the Consolidated
Fund of India.
Which of the statements given above is are correct...
(a) only 3
(b) 1 and 2 only
(c) 1, 2 and 3
(d) None
Correct Answer is C
(All are correct.)

40) Who appoints the Finance Commission?


(a) President
(b) Prime Minister
(c) Speaker
(d) Finance Minister
Correct Answer is President
(The Finance Commission is appointed by the President under Article
280 of the Constitution.)

41) How many members are nominated by Governor in state council?


(a) 1/6th
(b) 5/6th
(c) 1 member
(d) 2 member
Correct Answer is B
(Governor nominates one-sixth of the members of the state legislative
council from amongst persons having special knowledge.)

42) Governor is obligatory to Reserve the bill for the consideration of


the president in which of the following cases....
1. Where the bill passed by the state legislature endangers
the position of the state high court.
2. Opposed to the Directive Principles of State Policy.
Which of the above statement is/are correct
(a) 1 and 2
(b) 2 only
(c) 1 only
(d) None
Correct Answer is A
(Both are correct.)

43) Which statement/statements is/are correct?


1. Governor decides on the question of disqualification of
members of the state legislature in consultation with the
Election Commission.
2. Governor lays the reports of the State Finance
Commission.
3. Governor cannot promulgate an ordinances.
(a) 1 and 2
(b) 2 only
(c) 2 and 3
(d) 3 only
Correct Answer is A
(Governor can promulgate an ordinances.)

44) Which of the following does not come under the domain
of Governor?
(a) Pardon
(b) Reprieve
(c) Respite
(d) Exemption

Correct Answer is D
(Governor can pardon, reprieve, respite, remit, suspend or commute.)

45) Which statement/statements is/are correct?


1.Money bills can be introduced in the state legislature only
with prior recommendation of Governor.
2. Governor can grant pardons, reprives, respites and
remissions of punishment or suspend, remit and commute
3. The governor cannot return a money bill for the
reconsideration as far as money bill is concerned.
4. Governor's ordinance-making power is co-extensive with
the legislative power of the state legislature.
Select the correct answer using the codes given below.
(a) 1, 2 and 4 only
(b) 2 only
(c) 1,3 and 4 only
(d) 1, 2, 3 and 4
Correct Answer is D
(All are correct.)

46) Which of the following amendment makes ministerial advice


binding on the president?
(a) 41st Constitutional Amendment
(b) 42nd Constitutional Amendment
(c) 43rd Constitutional Amendment
(d) 44th Constitutional Amendment

Correct Answer is 42nd Constitutional Amendment


(42nd Constitutional Amendment (1976) made ministerial advice
binding on the President.)

47) Which of the following article says that the Chief Minister should be
appointed by Governor?
(a) Article 161
(b) Article 162
(c) Article 164
(d) Article 163
Correct Answer is C
(Article 164 only says that the Chief Minister shall be appointed by the
governor.)

48) Which statement/statements is/are correct with respect to


Situational Discretion of Governor
1. Appointment of chief minister when no party has a clear-
cut majority
2. Dismissal of the council of ministers when it cannot
prove the confidence of the state legislative assembly.
3. Dissolution of the state legislative assembly if the council
of ministers has lost its majority.
Select the correct answer using the codes given below...
(a) 1 only
(b) 2 and 3 only
(c) 1 and 3 only
(d) 1, 2 and 3

Correct Answer is D
(All are correct.)
49) Which statement/statements is/are correct?
1.Sixth Schedule says If any dispute arises as to the share of
such royalties to be made over to a district council, it shall
be referred to the governor.
2. Amount payable to the district council and the decision of
the governor President shall be final.
3. 6th schedule encircles Assam, Meghalaya, Tripura and
Mizoram.
Which of the statements given above is are correct...
(a) only 3
(b) 1 and 3 only
(c) 1, 2 and 3
(d) None
Correct Answer is B
(Amount payable to the district council and the decision of the governor
shall be final.)

50) Constitution does not have Which of the following?


(a) Procedure for the selection and appointment of the Chief
Minister
(b) Imoeachment Process
(c) Procedure of electing president
(d) Procedure of electing Vice President

Correct Answer is A
(Constitution does not contain any specific procedure for the selection
and appointment of the Chief Minister.)
51) Which of the following term is used in state
government?
(a) Ministry
(b) Department
(c) Houses
(d) Branch
Correct Answer is B
(The term ‘ministry' or ‘ministries' is used only in the centre. In
states, the state government is divided into departments and not
ministries.)

52) Consider the following statements:


1. Governor may not oblige the council of ministers which
has lost the confidence of the legislative assembly.
2. Courts can enquiry into the nature of advice rendered by
the ministers to the governor.
Which of the above statement is/are correct
(a) 1 Only
(b) 2 only
(c) Both are correct
(d) None
Correct Answer is A
(Courts are barred from enquiring into the nature of advice rendered by
the ministers to the governor.)

53) Which statement/statements is/are correct?


1. For constituency, population' means, the population as
ascertained at the last preceding census of which the relevant
figures have been published.
2. Parliament has enacted the Delimitation Commission Act
in 1952.
3. This ban on readjustment of constituency has been
extended for another 25 years by the 42nd Amendment Act
of 2001 till 2026.
(a) 1 and 2
(b) 2 only
(c) 2 and 3
(d) 3 only
Correct Answer is A
(This ban on readjustment of constituency has been extended for
another 25 years by the 84th Amendment Act of 2001 till 2026.)

54) Which of the following amendment has led to the formulation of


legislative council in Madhya Pradesh?
(a) 4th Amendment Act
(b) 5th Amendment Act
(c) 6th Amendment Act
(d) 7th Amendment Act

Correct Answer is D
(7th Amendment Act of 1956 provided for a Legislative Council in
Madhya Pradesh.)

55) Which statement/statements is/are correct?


1. Constitution provides for the creation of legislative councils in
states.
2. The council is created with a Majority of the total membership of
the assembly and a majority of not less than two-thirds of the
members of the assembly present and voting.
3. Constitution provides for the abolition of legislative councils in
states.
4. A simple majority is required in the parliament to create a
council.
Select the correct answer using the codes given below.
(a) 1, 2 and 4 only
(b) 2 only
(c) 1,3 and 4 only
(d) 1, 2, 3 and 4
Correct Answer is D
(All are correct.)

56) Which of the following is the maximum and minimum strength of


the state legislature?
(a) 500 and minimum strength at 40
(b) 500 and minimum strength at 30
(c) 500 and minimum strength at 60
(d) 500 and minimum strength at 42
Correct Answer is 500 and minimum strength at 60
(500 and minimum strength at 60.)

57) How many members governor can nominate from the Anglo-Indian
community? In assembly?
(a) Two Member
(b) Three Members
(c) Four Members
(d) One Member
Correct Answer is D
(1 Member)

58) Which statement/statements is/are correct?


1. The maximum strength of the council is fixed at one-third of the
total strength of the assembly.
2. The minimum strength of the council is fixed at 30.
3. 1/6 of the total number of members of a legislative council are
indirectly elected and 5/6 are nominated by the governor.
Select the correct answer using the codes given below...
(a) 1 only
(b) 2 and 3 only
(c) 1 and 3 only
(d) 1, 2 and 3
Correct Answer is A

(The minimum strength of the council is fixed at 40. 5/6 of the total
number of members of a legislative council are indirectly elected and 1/6
are nominated by the governor.)
59) Which statement/statements is/are correct with respect to legislative
council members' elections?
1. 1/3 are elected by the members of local bodies.
2. 1/12 are elected by graduates of three years standing and residing
within the state
3. 1/6 are elected by the members of the legislative assembly of the
state from amongst persons who are not members of the
assembly.
Which of the statements given above is are correct...
(a) only 3
(b) 1 and 2 only
(c) 1, 2 and 3
(d) None
Correct Answer is B
(1/3 are elected by the members of the legislative assembly of the state
from amongst persons who are not members of the assembly.)

60) How many members retire once in 2 years of State Council?


(a) 1/3rd
(b) 1/2nd
(c) 1/4th
(d) 1/5th
Correct Answer is B
(one-third of its members retire once every two years.)

61) What is the usual tenure of Governor's office?


(a) 5 Years
(b) 4 years
(c) 6 Years
(d) Not fixed
Correct Answer is A
(Governor usually holds office for a term of five years from the date on
which he enters upon his office. However, this term of five years is
subject to the pleasure of the President)

62) Consider the following statements with respect to Governor...


1. Governor Summon or prorogue the state legislature and dissolve
the state legislative assembly.
2. Governor nominates one-sixth of the members of the state
legislative council from amongst persons having special
knowledge
3. Governor nominates two member to the state legislature
assembly from the Anglo-Indian Community
Which of the above correctly matched?
(a) 1 and 2
(b) 3 only
(c) 1, 2 and 3
(d) None
Correct Answer is A
(Governor nominates one member to the state legislature assembly from
the Anglo-Indian Community.)

63) Which statement/statements is/are correct with respect to


Governor?
1. Governor lays the reports of the State Finance Commission.
2. Governor is consulted by the president while appointing the
judges of the concerned District courts.
3. Governor appoints persons to the judicial service of the state
other than district judges.
(a) 1 and 3
(b) 2 only
(c) 1, 2 and 3
(d) 3 only
Correct Answer is A
(Governor is consulted by the president while appointing the judges of
the concerned state high court.)

64) Which of the following article says If any dispute arises as to the
share of such royalties to be made over to a district council, it shall be
referred to the governor for determination and the amount determined
by the governor in his discretion shall be deemed to be the amount
payable to the district council and the decision of the governor shall be
final'.
(a) 5th Schedule
(b) 6th Schedule
(c) 7th Schedule
(d) 8th Schedule
(Sixth Schedule says: ‘If any dispute arises as to the share of such
royalties to be made over to a district council, it shall be referred to the
governor for determination and the amount determined by the governor
in his discretion shall be deemed to be the amount payable to the district
council and the decision of the governor shall be final'. The Sixth
Schedule contains the provisions as to the administration of tribal areas
in the States of Assam, Meghalaya, Tripura and Mizoram)

65) Which statement/statements is/are correct?


1. Governor can return a money bill for the reconsideration.
2. Governor cannot pardon a death sentence.
3. Governor determines the amount payable by the Government of
Assam, Meghalaya, Tripura and Mizoram to an autonomous
Tribal District Council as royalty accruing from licenses.
4. Appointment of chief minister when no party has a clear-cut
majority Comes under situational discretion of Governor.
Select the correct answer using the codes given below.
(a) 1 and 4 only
(b) 2 only
(c) 2,3 and 4 only
(d) 1, 2, 3 and 4
Correct Answer is C
(Governor cannot return a money bill for the reconsideration.)

66) Which of the following article says that there shall be a Council of
Minister?
(a) Article 163
(b) Article 166
(c) Article 165
(d) Article 164
Correct Answer is
(Article 163: There shall be a council of ministers with the Chief
Minister as the head to aid and advise the governor.)

67) Which committee said that the village panchayat should be


constituted with directly elected representatives?
(a) LM Singhvi Committee
(b) GVK Rao Committee
(c) Balwant Rai Mehta
(d) Ashok Mehta Committee
Correct Answer is C
(Balwant Rai Mehta.)
68) Which statement/statements is/are correct With respect to
recommendation of Balwant Rai Mehta committee?
1. District collector should be the chairman of the zila parishad.
2. There should be genuine transfer of power
3. Panchayat samiti should be the advisory body while the zila
parishad should be the executive body.
Select the correct answer using the codes given below...
(a) 1 only
(b) 2 and 3 only
(c) 1 and2 only
(d) 1, 2 and 3
Correct Answer is C
(Panchayat samiti should be the executive body while the zila parishad
should be the advisory body.)

69) What was the recommendations of Balwant Rai Mehta committee?


1. gram panchayat at the village level
2. panchayat samiti at the block level
3. zila parishad at the district level
Which of the statements given above is are correct...
(a) only 2
(b) 1 and 2 only
(c) 1, 2 and 3
(d) None
Correct Answer is C
(All are correct.)

70) Which state first establish the panchayati Raj?


(a) RAJASTHAN
(b) Orissa
(c) Haryana
(d) Uttar Pradesh
Correct Answer is A
(Rajasthan was the first state to establish Panchayati Raj.)

71) How many other election commissioners are there along with Chief
Election Commissioner?
(a) 2
(b) 5
(c) 4
(d) President decides
Correct Answer is President decides
(Election Commission consists of the chief election commissioners and
such number of other election commissioners as the president may from
time to time fix.)

72) Consider the following statements:


1. Appointment of the chief election commissioner is done by the
president.
2. President determines the conditions of service and tenure of
office of the election commissioners
3. President determines the conditions of service for the regional
commissioners.
Which of the above statement is/are correct
(a) 1 and 2
(b) 3 only
(c) 1, 2 and 3
(d) None
Correct Answer is C
(All are correct.)

73) Which statement/statements is/are correct?


1. The Constitution has prescribed the qualifications for the
members of the Election Commission.
2. Constitution does not restrict the retiring election commissioners
from further appointment by the government.
3. After the appointment, Service conditions of the chief election
commissioner cannot be varied to his disadvantage.
(a) 1 and 2
(b) 2 only
(c) 2 and 3
(d) 3 only
Correct Answer is C
(The Constitution has not prescribed the qualifications for the members
of the Election Commission.)

74) Which of the following article of the Constitution has ensured the
independent and impartial functioning of the Election Commission?
(a) Article 322
(b) Article 323
(c) Article 324
(d) Article 325
Correct Answer is C
(Article 324)

75) Which statement/statements is/are correct?


1. Election Commissioner advises the president whether elections
can be held in a state under president's rule.
2. Election Commissioner prepares a roster for publicity of the
policies.
3. Election Commissioner can cancel polls in the event of booth
capturing, rigging, violence and other irregularities.
4. Election Commissioner determines the territorial areas of the
electoral constituencies.
Select the correct answer using the codes given below.
(a) 1, 2 and 4 only
(b) 2 only
(c) 1,3 and 4 only
(d) 1, 2, 3 and 4
Correct Answer is D
(All are correct.)

76) Which of the following is the tenure of election commission of India?


(a) Six years or until they attain the age of 65 years
(b) Five years or until they attain the age of 65 years
(c) six years or until they attain the age of 62 years
(d) Five years or until they attain the age of 62 years
Correct Answer is six years or until they attain the age of 65 years

(Chairman and members of the Election Commission hold office for a


term of six years or until they attain the age of 65 years, whichever is
earlier.)
77) Who can remove any other member of UPSC for misbehaviour. ?
(a) President
(b) Chairman of UPSC
(c) Prime Minister
(d) Parliament
Correct Answer is President
(President can remove the chairman or any other member of UPSC for
misbehaviour. However the president has to refer the matter to the
Supreme Court for an enquiry. If the Supreme Court upholds the cause
of removal and advises so, the president can remove the chairman or a
member. The advise tendered by the Supreme Court in this regard is
binding on the president.)
78) Which statement/statements is/are correct?
1. Chairman or a member or UPSC is eligible for reappointment for
second term.
2. The member of UPSC is eligible for appointment as the
chairman of UPSC or a State Public Service Commission
(SPSC) however not for any other employment in the
Government of India or a state.
Select the correct answer using the codes given below...
(a) 1 only
(b) 2 only
(c) Both are correct
(d) None
Correct Answer is B
(Chairman or a member or UPSC is not eligible for reappointment for
second term.)

79) Which statement/statements is/are correct?


1. The jurisdiction of UPSC cannot be extended by an act made by
the Parliament.
2. UPSC presents a report to the parliament annually on its
performance.
3. Individual ministry or department has no power to reject the
advice of the UPSC.
Which of the statements given above is are correct...
(a) only 3
(b) 1 and 2 only
(c) 1, 2 and 3
(d) None
Correct Answer is A
(UPSC presents a report to the president annually on its performance.
The jurisdiction of UPSC can be extended by an act made by the
Parliament.)
80) Which of the following authority deals with all disciplinary matters
affecting a person serving under the Government of India in a civil
capacity including memorials or petitions relating to such matters?
(a) Prime Minister
(b) President
(c) UPSC
(d) High Court
Correct Answer is C
(UPSC)

81) Which of the following state does not come under the ambit of 11
states which are under the part of XXI of the constitution?
(a) Maharashtra
(b) Gujarat
(c) Haryana
(d) Karnataka
Correct Answer is Haryana
(Articles 371 to 371-J in Part XXI of the constitution contain special
provisions for 1 states which are Assam, Manipur, Andhra Pradesh,
Sikkim, Maharashtra, Gujarat, Nagaland, Mizoram, Arunachal
Pradesh, Goa and Karnataka.)

82) Consider the following statements:


1. Governor of Nagaland has special responsibility for law and
order because of internal disturbances caused by the hostile
Nagas continue.
2. Article 371-A concerns the special provisions for Assam.
Which of the above statement is/are correct
(a) 1 and 2
(b) 1 only
(c) 2 only
(d) None
Correct Answer is B
(Article 371-A concerns the special provisions for Nagaland.)

83) Which statement/statements is/are correct?


1. Governor administers the Tuensang district of Nagaland.
2. The Governor directs on the recommendation of the regional
council to apply The Act of Nagaland Legislature on Tuensang
district
3.There shall be a Minister for Tuensang affairs in the State Council
of Ministers.
(a) 1 and 2
(b) 2 only
(c) 1, 2 and 3
(d) 3 only
Correct Answer is C
(All are correct.)

84) ?
(a)
(b)
(c)
(d)

Correct Answer is B
(Article 371-B6, the President is empowered to provide for the creation
of a committee of the Assam Legislative Assembly consisting of the
members elected from the Tribal Areas of the state and such other
members as he may specify7)

85) Which statement/statements is/are correct?


1. Article 371-B makes the following special provisions for
Manipur.
2. Articles 371-D and 371-E contain the special provisions for
Andhra Pradesh
3. Article 371-E empowers the Parliament to provide for the
establishment of a Central University in the state.
Select the correct answer using the codes given below.
(a) 1 and 2 only
(b) 2 only
(c) 1,2 and 3
(d) 2 and 3 only
Correct Answer is D
(Article 371-B makes the following special provisions for Manipur.)

86) Which of the following Constitutional Amendment Act made Sikkim


a full-fledged state of Indian Union?
(a) 36th Constitutional Amendment Act
(b) 42nd Constitutional Amendment Act
(c) 84th Constitutional Amendment Act
(d) 61st Constitutional Amendment Act

Correct Answer is 36th Constitutional Amendment Act


(The 36th Constitutional Amendment Act of 1975 made Sikkim a full-
fledged state of the Indian Union. It included a new Article 371-F
containing special provisions with respect to Sikkim.)

87) With which of the following state 371-G is related to?


(a) Mizoram
(b) Assam
(c) Manipur
(d) Andhra Pradesh
Correct Answer is Mizoram
(Article 371-G specifies the special provisions for Mizoram. Mizoram
Legislative Assembly shall consist of not less than 40 members.)

88) Which statement/statements is/are correct?


1. Town area committee is set up for the small town administration.
2. Town area committee is a semi-municipal authority.
3. Town area committee is a wholly nominated body only.

Select the correct answer using the codes given below...


(a) 1 only
(b) 2 and 3 only
(c) 1 and 2 only
(d) 1, 2 and 3

Correct Answer is C
(Town area committee is a wholly nominated body by a state
government or a wholly elected body or partly nominated and partly
elected.)

89) Which statement/statements is/are correct?


1. Article 371-J provides for special provisions for the Hyderabad-
Karnataka region of the state of Karnataka
2. Under Article 371-J was inserted in the Constitution by the 98th
Constitutional Amendment Act of 2012.
3. The special provision aims for development of the most
backward region of the state and promote inclusive growth with
a view to reducing inter-district and inter-regional disparities in
the state.
Which of the statements given above is are correct...
(a) only 3
(b) 1 and 2 only
(c) 1, 2 and 3
(d) None
Correct Answer is C
(All are Correct.)

90) Which of the following state Article 371-H provides special


provisions for?
(a) Assam
(b) Manipur
(c) Mizoram
(d) Arunachal Pradesh
Correct Answer is D
(Article 371-H provides special provisions for Arunachal Pradesh.)

91) Which of the following committee has more members from


Rajyasabha than Loksabha?
(a) Ethics Committee
(b) Committee on Petitions
(c) Committee of Privileges
(d) Rules Committee
Correct Answer is Rules Committee.
(Rajya sabha has 16 members, and Loksabha has 15 members.)

92) Consider the following statements:


1. The Parliamentary committee is appointed or elected by the
House or nominated by the Speaker / Chairman.
2. Parliamentary committee presents its to Council of Minister.
3. Ad hoc committees can be Inquiry Committees or Advisory
Committees.
Which of the above correctly matched?
(a) 1 and 3
(b) 3 only
(c) 1, 2 and 3
(d) None
Correct Answer is A
(Parliamentary committee presents its report to the House or to the
Speaker / Chairman.)

93) Which statement/statements is/are correct?


1. Public Accounts Committee has 22 members (15 from the Lok
Sabha and 7).
2. Public Accounts Committee examine the appropriation accounts
and the finance accounts of the Union government.
3. Public Accounts Committee examine the accounts of stores and
stocks.
(a) 1 and 2
(b) 2 only
(c) 1, 2 and 3
(d) 3 only
Correct Answer is C
(All are Correct)

94) How many members are there in Public Accounts Committee?


(a) 31
(b) 15
(c) 30
(d) 22
Correct Answer is 22

(Public Accounts Committee had 15 members (10 from the Lok Sabha
and 5 from the Rajya Sabha). However, in 1974, The membership was
increased to 22 (15 from the Lok Sabha and 7 from the Rajya Sabha)).
95) Which statement/statements is/are correct?
1. Estimates Committee has 22 members.
2. The Rajya Sabha has no representation in Estimates Committee.
3. Estimates Committee suggests alternative policies in order to
bring about efficiency and economy in administration.
4. The chairman of the Estimates Committee is appointed by the
Speaker.
Select the correct answer using the codes given below.
(a) 1 and 4 only
(b) 2 only
(c) 2,3 and 4 only
(d) 1, 2, 3 and 4
Correct Answer is C
(Estimates Committee has 30 members.)

96) How many members are there in Departmental Standing


Committees?
(a) 22
(b) 24
(c) 30
(d) 31
Correct Answer is 31
(There are 31 members, 21 come from Loksabha and 10 from
Rajyasabha.)

97) What is the term of office of each standing committee from the date
of its constitution?
(a) 1 Year
(b) 2 Years
(c) 3 Years
(d) 5 Years
Correct Answer is 1 Year
(The term is generally 1 year.)

98) Which of the following report is submitted by CAG to the


President?
1. Audit report on public undertakings
2. Audit report on finance accounts
3. Audit report on appropriation accounts
Select the correct answer using the codes given below...
(a) 1 only
(b) 2 and 3 only
(c) 1 and 3 only
(d) 1, 2 and 3
Correct Answer is D
(All of the above are submitted by CAG.)

99) Which statement/statements is/are correct?


1. Committee on petition consists of 25 members, Lok Sabha
committee consists of 15 members, while the Rajya Sabha
committee consists of 10 members
2. Committee on privilege has 25 members, Lok Sabha committee
has 15 members, while the Rajya Sabha committee has 10
members.
3. Committee on Government Assurances Lok Sabha consists 30
members, Loksabha has 15 members and in the Rajya Sabha has
15.
Which of the statements given above is are correct...
(a) only 2
(b) 1 and 2 only
(c) 1, 2 and 3
(d) None
Correct Answer is B
(Committee on Government Assurances Lok Sabha consists 25
members, Loksabha has 15 members and in the Rajya Sabha has 10.)

100) How many members Committee on Subordinate Legislation


consists?
(a) 22 members
(b) 31 members
(c) 20 members
(d) 30 members
Correct Answer is 30 members
(In both the Houses, the committee consists of 15 members.)
101) In which one of the following cases Judicial
Review is not used?
(a) Golaknath case
(b) Shankari Prasad case
(c) Bank Nationalisation case
(d) Kesavananda Bharati case
Correct Answer is Shankari Prasad case
(Supreme Court used the power of judicial review in cases such as
Golaknath case (1967), the Bank Nationalisation case (1970), the Privy
Purses Abolition case (1971), the Kesavananda Bharati case (1973), the
Minerva Mills case (1980).)

You can also join telegram channel for latest updates

Telegram Exam Group (For Updates on UPSC)

Telegram UGC NET Channel (UGC Material)

UGC NET Group (To Meet Other Aspirants and Free Test Series)

UPSC NET Group (To Meet Other Aspirants and Free Test Series)

You can also enjoy our Youtube videos on our channel.

Don’t Forget to rate us on Amazon and claim a Free UPSC or UGC


(Any One) test series worth ₹ 1999 on our telegram group given above.

102) Consider the following statements:


1. In First Judges case the Court held that consultation does not
mean concurrence.
2. In Second Judges case the Court reversed and consultation
means concurrence.
3. In the third judge case the Court opined th concept of
‘consultation of plurality judges
Which of the above is/are correct?
(a) 1 and 3
(b) 3 only
(c) 1, 2 and 3
(d) None
Correct Answer is C

(This discretion of the government was restricted by the Supreme Court


in the Second Judges Case (1993).)
103) Which statement/statements is/are correct?
1. To remove a judge, a motion is signed by 100 members in the
case of Lok Sabha.
2. The committee should consist of the chief justice or a judge of
the Supreme Court, chief justice of a high court, and a
distinguished jurist.
3. Finally, the president passes an order removing the judge.
(a) 1 and 2
(b) 2 only
(c) 1, 2 and 3
(d) 3 only
Correct Answer is C
(All are Correct.)

104) What is the power to hear disputes in the first instance not by way
of appeal is called?
(a) Original Jurisdiction
(b) Appellate Jurisdiction
(c) Advisory Jurisdiction
(d) Writ Jurisdiction
Correct Answer is Original Jurisdiction
(It deals with disputed between Centre and State or between two or
more states.)

105) Which statement/statements is/are correct?


1. The Chief Justice of India, for a temporary period, can appoint a
judge of a High Court as an ad hoc judge of the Supreme Court.
2. Chief justice of India can request a retired judge of the Supreme
Court, not a retired judge of a high court to act as a judge of the
Supreme Court for a temporary period.
3. Constitution declares Delhi as the seat of the Supreme Court.
4. President appoints judges of the Supreme Court in consultation
with the members of the judiciary.
Select the correct answer using the codes given below.
(a) 1 and 4 only
(b) 2 only
(c) 1,3 and 4 only
(d) 1, 2, 3 and 4
Correct Answer is C
(Chief justice of India can request a retired judge of the Supreme Court
or a retired judge of a high court to act as a judge of the Supreme Court
for a temporary period.)

106) Which article provides for Establishment and Constitution of


Supreme Court?
(a) Article 121
(b) Article 124
(c) Article 123
(d) Article 125
Correct Answer is Article 124
(Article 124.)

107) Jurisdiction of the Supreme Court extends to the following?


(a) Dispute arising out of any pre-Constitution treaty
(b) Inter-state water disputes
(c) Matters referred to the Finance Commission
(d) Dispute between two or more states
Correct Answer is Dispute between two or more states
(The Supreme Court does not have jurisdiction over Adjustment of
certain expenses and pensions between the Centre and the states,
Ordinary dispute of Commercial nature between the Centre and the
states, Recovery of damages by a state against the Centre.)

108) Which statement/statements is/are correct?


1. Judges of Supreme Court do not hold their office during the
pleasure of the President even though they are appointed by him.
2. Salaries and allowances of Supreme Court judges are non-votable
by the Parliament.
3. Salary of judges cannot be changed to their disadvantage after
their appointment in any case.
Select the correct answer using the codes given below...
(a) 1 only
(b) 2 and 3 only
(c) 1 and 2 only
(d) 1, 2 and 3
Correct Answer is C

(Salary of judges cannot be changed to their disadvantage after their


appointment except during a financial emergency.)
109) Which statement/statements is/are correct?
1. Except when an impeachment motion is under consideration of
the Parliament, Conduct of Judges cannot be discussed.
2. The retired judges of the Supreme Court are prohibited from
pleading or acting in any Court.
3. Parliament is not authorised to curtail the jurisdiction and powers
of the Supreme Court.
Which of the statements given above is are correct...
(a) only 2
(b) 1 and 2 only
(c) 1, 2 and 3
(d) None
Correct Answer is C
(All are correct.)

110) How many categories of Advocates are entitled to practice law


before the Supreme Court?
(a) Two
(b) Four
(c) Three
(d) Six
Correct Answer is Three.
(There are 3 types of advocates who can preactice before the supreme
court.)

111.) How many members of members of a metropolitan planning


committee should be elected?
(a) 1/3rd
(b) 4/5th
(c) 2/3rd
(d) 1/4th
Correct Answer is 2/3rd
(2/3rd of the members of a metropolitan planning committee should be
elected by the elected members of the municipalities and chairpersons of
the panchayats in the metropolitan area from amongst themselves.)

112) Consider the following statements:


1. Municipal corporations are created for the administration of big
cities like Delhi, Hyderabad, Bangalore and others.
2. Municipal corporation has three authorities viz. The council, the
standing committees and the commissioner.
3. Municipal commissioner is responsible for the implementation
of the decisions taken by the council and its standing
committees.
Which of the above correctly matched?
(a) 1 and 3
(b) 3 only
(c) 1, 2 and 3
(d) None
Correct Answer is C
(All are correct.)

113) Which statement/statements is/are correct?


1. Notified area committee is created for the administration
of schedule areas.
2. Notified area committee is created for the administration of a
town which does not yet fulfil all the conditions necessary for
the constitution of a municipality.
3. All the members of a notified area committee including the
chairman are nominated by the state government.
(a) 1 and 2
(b) 2 only
(c) 2 and 3
(d) 3 only
Correct Answer is C
(Notified area committee is created for the administration of a fast
developing town due to industrialisation. )

114) Which of the following is not a committee constituted for


Panchayati Raj?
(a) V.T.Krishnammachari : 1960
(b) Takhatmal Jain Study Group: 1966
(c) Ashok Mehta Committee : 1977
(d) Meen Committee : 1980
Correct Answer is Meen Committee:1980
(Meen Committee:1980)

115) Which of the following is the voluntary provision of the Panchayat


act?
1. The organisation of Gram Sabha in a village or group of villages.
2. Direct elections to all seats in panchayats at the village,
intermediate and district levels.
3. 25 years to be the minimum age for contesting elections to
panchayats.
4. Providing reservation of seats for the backward classes in
panchayats at any level.
Select the correct answer using the codes given below.
(a) 4 only
(b) 2 only
(c) 1,2 and 4 only
(d) 1, 2, 3 and 4
Correct Answer is C
(21 years to be the minimum age for contesting elections to panchayats.)
116) Part IX of the constitution does not apply to schedule areas, where
do these schedule areas mention?
(a) V Schedule
(b) VI Schedule
(c) IX Schedule
(d) X Schedule
Correct Answer is Schedule V

(The provisions of Part IX of the constitution relating to the Panchayats


are not applicable to the Fifth Schedule areas.)
117) Where does the Schedule V area not present?
(a) Andhra Pradesh
(b) Gujarat
(c) Maharashtra
(d) Haryana
Correct Answer is Haryana
(Fifth schedule areas are present in Andhra Pradesh, Chhatisgarh,
Gujarat, Himachal Pradesh, Jharkhand, Madhya Pradesh,
Maharashtra, Odisha and Rajasthan.)

118) Which statement/statements is/are correct?


1. Lord Ripon's Resolution of 1882 has been referred as the ‘Magna
Carta' of local self-government.
2. In Government of India Act of 1935, local self-government was
declared a provincial subject.
3.Lord Mayo's Resolution of 1870 mentoined financial
decentralisation for the development of local self-government
institutions.
Select the correct answer using the codes given below...
(a) 1 only
(b) 2 and 3 only
(c) 1 and 3 only
(d) 1, 2 and 3
Correct Answer is D
(All are Correct.)

119) Which statement/statements is/are correct?


1. Nagar panchayat is for a transitional area, an area in transition
from a rural area to an urban area.
2. A municipal council for a larger urban area.
3. A municipal corporation for a smaller urban area.
Which of the statements given above is are correct...
(a) only 1
(b) 1 and 2 only
(c) 1, 2 and 3
(d) None
Correct Answer is A
(A municipal council for a smaller urban area. A municipal corporation
for a larger urban area.)

120) How many members of a district planning committee should be


elected ?
(a) 3/4th
(b) 1/3rd
(c) 2/3rd
(d) 4/5th
Correct Answer is 4/5th
(4/5th of the members of a district planning committee should be elected
by the elected members of the district panchayat and municipalities in
the district from amongst themselves. )
121) K Santhanam committee was formulated for?
(a) President election
(b) Panchayati Raj Elections
(c) Gram Sabha
(d) Municipalities
Correct Answer is Panchayati Raj Elections
(Committee on Panchayati Raj Elections -K Santhanam.)

122) Consider the following statements:


1. Tamil Nadu : Three-tier system
2. Rajasthan : Three-tier system
3. West Bengal : Four-tier system
Which of the above correctly matched?
(a) 2 and 3
(b) 3 only
(c) 1, 2 and 3
(d) None
Correct Answer is A
(Tamil Nadu : Two-tier system)

123) Which statement/statements is/are correct?


1. Balwant Rai Mehta Committee was followed by the Santhanam
Committee.
2. Santhanam Committee’s scope of study was the financial matters
of the PRIs.
3. Santhanam Committee recommended that Panchayati Raj
Finance Corporations should be established.
(a) 1 and 2
(b) 2 only
(c) 1, 2 and 3
(d) 3 only
Correct Answer is C
(All are correct.)

124) Which of the following committee suggested that the 3 tier


structure of panchayat should be replaced with the 2 tier structure?
(a) LM Singhvi committee
(b) GVK Rao committee
(c) Balwant Rai Mehta committee
(d) Ashok Mehta committee
Correct Answer is Ashok Mehta committee
(Three-tier system of panchayati raj should be replaced by the two-tier
system is recommended by Ashok Mehta committee.)

125) Which of the follwing recommendation was given by the Balwant


Rai Committee. ?
1. Panchayati raj institutions should have compulsory powers of
taxation.
2. Zila parishad should be the executive body.
3. The zila parishad should be given development functions.
4. Minister for panchayati raj should be appointed in the state
council of ministers to look after the affairs of the panchayati raj
institutions.
Select the correct answer using the codes given below.
(a) 1 and 4 only
(b) 2 only
(c) 1,3 and 4 only
(d) 1, 2, 3 and 4
Correct Answer is D
(All are correct.)

126) What was 64th constitutional amendment bill was all about?
(a) Panchayati Raj
(b) Reservation
(c) Delimitation
(d) Ordinance
Correct Answer is Panchayati Raj
(Rajiv Gandhi Government introduced the 64th Constitutional
Amendment Bill in the Lok Sabha in 1989 to confer constitutional status
to panchayati raj institutions.)

127) Which of the following PM succeeded in making Panchayti raj a


constitutional institution?
(a) Rajiv Gandhi
(b) VP Singh
(c) Indira gandhi
(d) PV Narsimhan Rao
Correct Answer is PV Narsimhan Rao
(73rd Constitutional Amendment Act, 1992 and came into force on 24
April, 1932.)

128) Which of the following is/are the recommendations of GVK Rao?


1. District level body or Zila Parishad should be of pivotal
importance.
2. A post of District Development Commissioner should be
created.
3. Elections to the Panchayati Raj institutions should be held
regularly.
Select the correct answer using the codes given below...
(a) 1 only
(b) 2 and 3 only
(c) 1 and 3 only
(d) 1, 2 and 3
Correct Answer is D
(All are correct.)
129) Which of the following is advocated by the Ashok Mehta
Committee?
1. Reduction in the developmental role of the District Collector.
2. Assign a major role to the Panchayati Raj.
Which of the statements given above is are correct...
(a) only 2
(b) 1 only
(c) Both of the above
(d) None
Correct Answer is C
(Both of the above.)

130) Which of the following committee advocated separate district


planning bodies under either the District Collector or a minister.?
(a) GVK Rao
(b) LM Singhvi
(c) Ashok Mehta
(d) Hanmantha rao
Correct Answer is D
(Hanumantha Rao Committee advocated separate district planning
bodies under either the District Collector or a minister.)

131) Under which part of constitution The Panchayat comes?


(a) Part-IX
(b) Part-XI
(c) Part-X
(d) Part-III
Correct Answer is Part-IX
(The Panchayats consists of provisions from Articles 243 to 243 O.)

132) Consider the following statements:


1. Gram Sabha consisting of all the registered voters in panchayat
area.
2. The act provides for the reservation of not less than one-third of
the total number of seats for women, including the number of
seats reserved for women belonging the SCs and STs.
3. Tthe chairperson of panchayats at the intermediate and district
levels shall be elected indirectly.
Which of the above is/are correct?
(a) 1 and 3
(b) 3 only
(c) 1, 2 and 3
(d) None
Correct Answer is C
(All are Correct)

133) Which statement/statements is/are correct?


1. The town area committee is set up for the administration of a
small town.
2. It is a semi-municipal authority.
3. The members of the town area committee may be wholly elected
or wholly nominated by the state government or partly elected
and partly nominated.
(a) 1 and 2
(b) 2 only
(c) 1, 2 and 3
(d) 3 only
Correct Answer is C
(Town Area Committee is given a limited number of civic functions like
drainage, roads, street lighting, and conservancy.)

134) Under which article it is given that Council of Ministers is


supposed to aid and advise Governor?
(a) Article 163
(b) Article 164
(c) Article 165
(d) Article 166
Correct Answer is Article 163
(Article 163 says thatCouncil of Ministers is supposed to aid and advise
Governor.)

135) Which statement/statements is/are correct?


1. The strength of council of minister shall not exceed 15% of the
strength of the assembly excluding the Prime Minister.
2. The number of ministers (including chief minister) in a state
shall not be less than 12.
3. Prime Minister has the rank of cabinet minister.
Select the correct answer using the codes given below.
(a) 1 and 2 only
(b) 2 only
(c) 2 and 3 only
(d) 1 only
Correct Answer is C
(The total number of ministers (including chief minister) in the council
of ministers in a state shall not exceed 15 % of the total strength of the
legislative assembly. The number of ministers, including the chief
minister, in a state shall not be less than 12.)

136) Which of the following authority appoints state election


commissioner?
(a) Governor
(b) President
(c) Chief Minister
(d) Chief Justice of High Court
Correct Answer is Governor
(State election commissioner to be appointed by the governor.)
137) Where does the grant in aid for panchayat come from?
(a) Consolidated fund of the India
(b) Consolidated fund of the state
(c) Public Accounts of India
(d) Contingency fund of the state
Correct Answer is Consolidated fund of the state

(grants-in-aid to the panchayats from the consolidated fund of the


state.)
138) Which statement/statements is/are correct?
1. All executive actions of the Government of a State are taken in
the name of the Governor.
2. Governor shall make rules for the more convenient transaction of
the business of the government of the state.
3. Article 163 provides for a council of ministers with the chief
minister at the head to aid and advise the governor.
Select the correct answer using the codes given below...
(a) 1 only
(b) 2 and 3 only
(c) 1 and 3 only
(d) 1, 2 and 3
Correct Answer is D
(All are correct.)

139) Which statement/statements is/are correct?


1. The advice tendered by Ministers to the Governor can be
inquired into in any court.
2. There shall be a Minister in charge of tribal welfare
Chhattisgarh, Jharkhand, Madhya Pradesh, Bihar and Odisha.
Which of the statements given above is are correct...
(a) only 2
(b) 1 and 2 only
(c) 1, 2 and 3
(d) None
Correct Answer is D
(The advice tendered by Ministers to the Governor shall not be inquired
into in any court. There shall be a Minister in charge of tribal welfare
Chhattisgarh, Jharkhand, Madhya Pradesh and Odisha.)

140) Who was appointed as Constitutional Advisor of the Assembly?


(a) K.Khetan
(b) Krishnamachari
(c) BR Ambedkar
(d) B N Rao
Correct Answer is B N Rao
(B N Rao was appointed the Constitutional Advisor of the Assembly.)

141) Which of the following does not form part of union territories
under article 1?
(a) Adjacent territories
(b) Territories that may be acquired by the Government of India
(c) Union territories
(d) Territories of the states
Correct Answer is A
(Under Article 1 of the Constitution, the territory of India comprises
three categories of territories which are... (a) territories of the states (b)
union territories (c) territories that may be acquired by the Government
of India at any time.)

142) Consider the following statements:


1. Certain areas were constituted as ‘scheduled districts' in 1874.
2. Scheduled districts are known as ‘chief commissioners
provinces'.
3. After independence, Scheduled district were placed in the
category of Part ‘A' and Part ‘B' states.
Which of the above statement is/are correct
(a) 1 and 2
(b) 3 only
(c) 1, 2 and 3
(d) None
Correct Answer is A
(After independence, Scheduled district were placed in the category of
Part ‘C' and Part ‘D' states.)

143) Which statement/statements is/are correct?


1. Election Commission is a permanent and an independent body.
2. Election Commission is a constitution body.
3. The power of superintendence of Panchayat Election is given to
election commission.
(a) 1 and 2
(b) 2 only
(c) 2 and 3
(d) 3 only
Correct Answer is A
(Election Commission is a permanent and an independent body
established by the Constitution of India directly to ensure free and fair
elections in the country. Article 324 of the Constitution provides that the
power of superintendence, direction and control of elections to
parliament, state legislatures, the office of president of India and the
office of vice-president of India.)

144) Which of the following schedule contains special provision


for Assam, Meghalaya, Tripura and Mizoram?
(a) Schedule 3
(b) Schedule 4
(c) Schedule 5
(d) Schedule 6
Correct Answer is Schedule 6
(6th schedule contains special provisions for the administration of tribal
areas in the four north-eastern states of Assam, Meghalaya, Tripura and
Mizoram)

145) Which of the following are states today but were formerly union
territories?
1. Himachal Pradesh
2. Manipur
3. Tripura
4. Mizoram
Select the correct answer using the codes given below.
(a) 1, 2 and 4 only
(b) 2 only
(c) 1,3 and 4 only
(d) 1, 2, 3 and 4
Correct Answer is D
(Arunachal Pradesh and Goa were also the UTs earlier.)

146) Which of the following Areas U N Dhebar commission and Dilip


Singh Bhuria Commission formulated for?
(a) Constitutional structure
(b) Money Laundering
(c) Schedule areas
(d) Poverty
Correct Answer is Schedule areas
(A commission was appointed in 1960. It was headed by U N Dhebar
and submitted its report in 1961. After four decades, the second
commission was appointed in 2002 under the chairmanship of Dilip
Singh Bhuria.)

147) Requires the President to appoint a commission to report on the


administration of the Schedule areas ?
(a) Prime Minister
(b) Speaker
(c) Prerogative of the president
(d) Constitution
Correct Answer is Constitution

(Constitution requires the president to appoint a commission to report


on the administration of the scheduled areas and the welfare of the
scheduled tribes in the states. He can appoint such a commission at any
time but compulsorily after ten years of the commencement of the
Constitution.)
148) Which statement/statements is/are correct?
1. Lieutenant Governor Administers Delhi, Puducherry and
Andaman and Nicobar Islands
2. Administrator Administers in the case of Chandigarh, Dadra and
Nagar Haveli, Daman and Diu and Lakshadweep.
Select the correct answer using the codes given below...
(a) Both are correct
(b) None
(c) 2 only
(d) 1 only
Correct Answer is A
(Both are correct.)

149) Which statement/statements is/are correct?


1. 69th Constitutional Amendment Act Conferred a special status
on the Union Territory of Delhi.
2. Lt. governor is empowered to promulgate ordinances during
recess of the assembly.
3. Ordinance can be promulgated or withdrawn without the prior
permission of the President.
Which of the statements given above is are correct...
(a) only 3
(b) 1 and 2 only
(c) 1, 2 and 3
(d) None
Correct Answer is B
(No ordinance can be promulgated or withdrawn without the prior
permission of the President.)

150) Which of the following Union Territory has a High Court of its
own?
(a) Puducherry
(b) Dadra and Nagar Haveli
(c) Chandigarh
(d) New Delhi
Correct Answer is New Delhi
(Delhi is the only union territory that has a high court of its own )
151) Which of the following appoints Auditor
General?
(a) Prime Minister
(b) President
(c) CJI
(d) Speaker
Correct Answer is President
(AG is appointed by the president.)

152) Consider the following statements:


1. AG must be a person who is qualified to be appointed a judge of
the Supreme Court.
2. AG should have been either Judge of high court for five years or
an advocate of some high court for ten years.
3. As per Article 76, Constitution has provided for the office of the
Attorney General for India.
Which of the above statement is/are correct
(a) 1 and 2
(b) 3 only
(c) 1, 2 and 3
(d) None
Correct Answer is C
(All are correct.)

153) Which statement/statements is/are correct?


1. The remneuration of the AG is not fixed by the Constitution.
2. President determines the remuneration of Auditor General.
(a) 1 and 2
(b) 2 only
(c) 1 only
(d) None of the above
Correct Answer is B
(The remneuration of the AG is not fixed by the Constitution.)

154) Under which article AG represent the Government of India in any


reference made by the president to the Supreme Court?
(a) Article 141
(b) Article 142
(c) Article 143
(d) Article 144
Correct Answer is Article 143
(AG represents the Government of India in any reference made by the
president to the Supreme Court under Article 143 of the Constitution)

155) Which statement/statements is/are correct with respect AG?


1. Attorney General should not advise or hold a brief against the
Government of India.
2. Attorney Generalshould not advise or hold a brief in cases in
which he is called upon to advise or appear for the Government
of India
3. He is not a full-time counsel for the Government.
4. AG does not fall in the category of government servants.
Select the correct answer using the codes given below.
(a) 1, 2 and 4 only
(b) 2 only
(c) 1,3 and 4 only
(d) 1, 2, 3 and 4
Correct Answer is D
(AG is not debarred from private legal practice.)

156) Which of the following is true about the NDC?


(a) Constitutional Body
(b) Executive resolution
(c) Statutory Body
(d) Cabinet body
Correct Answer is Executive resolution
(NDC was established in August 1952 by an executive resolution. It is
neither a constitutional body nor a statutory.)

157) Where the Solicitor General has the right to speak in ?


(a) State Assembly
(b) State Council
(c) Both the Houses
(d) None of the houses
Correct Answer is C
(Solicitor General has the right to speak and to take part in the
proceedings of both the Houses of the state legislature or any committee
of the state legislature.)

158) Which statement/statements is/are correct?


1. AG is a member of the Central cabinet.
2. Solicitor general of India and additional solicitor general of India
assist the AG.
3. AG is created by the Constitution.
Select the correct answer using the codes given below...
(a) 1 only
(b) 2 and 3 only
(c) 1 and 3 only
(d) 1, 2 and 3
Correct Answer is B
(AG is not a member of the Central cabinet.)

159) Which statement/statements is/are correct?


1. Article 165 has provided for the office of the advocate general for
the states.
2. Solicitor General is the highest law officer in the country.
3. Solicitor General appointed by the governor.
Which of the statements given above is are correct...
(a) only 3
(b) 1 and 2 only
(c) 1, 2 and 3
(d) 1 and 3
Correct Answer is D
(Solicitor General is the highest law officer in the state.)

160) Where can the Solicitor General participate ?


(a) State committees only
(b) Proceddings of the state houses
(c) Both of the above
(d) None of the above
Correct Answer is C

(Solicitor General has the right to speak and to take part in the
proceedings of both the Houses of the state legislature or any committee
of the state legislature.)

161) How many members are there in Public Accounts Committee of


India?
(a) 15 members
(b) 20 members
(c) 21 members
(d) 22 members
Correct Answer is 22 members
(There are 22 members are there in PAC, 15 from the Lok Sabha and 7
Rajyasabha.)

162) In Which among the following areas Loksabha and Rajyasabha


have equal power?
1. Approval of ordinances issued by the President.
2. Approval of proclamation of all three types of emergencies by the
President.
3. Introduction and passage of Constitutional amendment bills.
Which of the above correctly matched?
(a) 1 and 3
(b) 3 only
(c) 1, 2 and 3
(d) None
Correct Answer is C
(All are Correct.)

163) Which statement/statements is/are correct?


1. RajyaSabha can discuss the budget.
2. RajyaSabha can vote on the budget.
3. RajyaSabha cannot remove the council of ministers by passing a
no-confidence motion
(a) 1 and 2
(b) 2 only
(c) 1, 2 and 3
(d) 3 only
Correct Answer is A
(RajyaSabha cannot vote on the budget, it can only discuss the budget.)

164) Which of the following is the oldest of all panels?


(a) Public Accounts Committee
(b) Estimate Committee
(c) Parliamentary committee
(d) Rules Committee
Correct Answer is Public Accounts Committee
(PAC is sometimes called the mother of all parliamentary
committees which keeps a supervision over use of public money by
executives. The responsibility of the committee is to keep an eye on
the spending and performance of the government and bring forth the
inefficiencies and financial wastage. PAC is the oldest of all the house
panels which has its origin in the Raj.)

165) Which of the following areas the power of Loksabha and


Rajyasabha are unequal?
1. Money Bill
2. Financial bill (not containing solely the matters of Article 110)
3. Joint sitting
Select the correct answer using the codes given below.
(a) 1 only
(b) 2 only
(c) 3 only
(d) 1, 2, 3
Correct Answer is D
(All are Correct.)

166) With the following authority appoints the chairperson of state


human rights commission?
(a) Governor
(b) President
(c) Chief Minister
(d) Speaker of State legislative assembly
Correct Answer is Governor
(Governor Opponents the chairperson of State human rights
commission on the recommendation of the committee which consists of
Chief Minister of the State, leader of the opposition, speaker of the
state Legislative assembly and the State Home Minister.)

167) In how many months, the government shall take action on the
recommendation of the Human rights commission?
(a) 1 month
(b) 2 months
(c) 3 months
(d) 4 months
Correct Answer is 3 months
(3 months.)

168) Which statement/statements is/are correct?


1. Rajya Sabha in our constitutional system is not as weak as that of
the House of Lords in the British constitutional system
2. Rajya Sabha in our constitutional system is not as strong as that
of the Senate in the American constitutional system.
3. The powers and status of the Rajya Sabha in all other spheres are
equal.
Select the correct answer using the codes given below...
(a) 1 only
(b) 2 and 3 only
(c) 1 and 3 only
(d) 1, 2 and 3
Correct Answer is D
(All are Correct.)

169) Which statement/statements is/are correct?


1. Committee on Public Accounts was first set up in Morley-Minto
Reforms.
2. W M Hailey was its first president, and Bhupendra Nath Mitra its
first Indian president of PAC.
3. PAC keeps a vigil over executive spending.
Which of the statements given above is are correct...
(a) only 2
(b) 2 and 3 only
(c) 1, 2 and 3
(d) None
Correct Answer is B
(Committee on Public Accounts was first set up in 1921 in the wake of
the Montague-Chelmsford Reforms.)

170) Where does the human rights commission lay its annual or special
report?
(a) Central government
(b) State government
(c) None of the above
(d) Both of the above
Correct Answer is D
( Both of the above)

171) Which of the following is the head of NDC?


(a) President
(b) Speaker
(c) Home Minister
(d) Prime Minister
Correct Answer is D

(Prime Minister)

172) Consider the following statements:


1. The five-year draft used to be prepared by the planning
commission.
2. The draft will be submitted to the union Cabinet and then would
move to NDC.
3. Finally, it would be published in the Gazette.
Which of the above statement is/are correct
(a) 1 and 2
(b) 3 only
(c) 1, 2 and 3
(d) None
Correct Answer is C
(Draft Five-Year Plan prepared by the Planning Commission is first
submitted to the Union cabinet. It is placed before the NDC, for its
acceptance. The Plan then is presented to the Parliament. Once
approved, it emerges as the official Plan and published in the official
gazette.)

173) Which statement/statements is/are correct?


1. NDC Is also called super cabinet.
2. NDC recommendations are binding.
3. NDC is also described as rubber stamp.
(a) 1 and 2
(b) 2 only
(c) 2 and 3
(d) 3 only

Correct Answer is B
(described as a ‘Super Cabinet' due to its wide and powerful
composition, though its recommendations are only advisory and not
binding, and can hardly be ignored as they are backed by a national
mandate. On the other hand, it has been described as a mere ‘rubber
stamp' .)

174) Which of the following commission recommended that NDC should


be given the constitutional status?
(a) Sarkaria Commission
(b) Punchi Commission
(c) JVP Commission
(d) Ashok Mehta Commission
Correct Answer is Sarkaria Commission
(Sarkaria Commission on Centre–State Relations recommended that the
NDC should be given a constitutional status under Article 263 of the
Constitution and should be renamed as National Economic and
Development Council.)

175) Which statement/statements is/are correct?


1. Human Right commission (HRC) is a multi-member body
consisting of a chairman and four members.
2. The members of HRC are either serving or retired judges.
3. HRC has four ex-officio members viz. chairmen of the National
Commission for Minorities, the National Commission for SCs,
the National Commission for STs and the National Commission
for Women.

Select the correct answer using the codes given below.


(a) 1 only
(b) 2 only
(c) 3 only
(d) 1, 2and 3
Correct Answer is D
(All are correct.)

176) till what age, Chairman and members of the human rights
commission hold office?
(a) 65 years
(b) 62 years
(c) 70 years
(d) 68 years
Correct Answer is 70 Years
(Chairman and members hold office for a term of five years or until
they attain the age of 70 years, whichever is earlier.)

177) for how many years, chairperson and members hold office in the
case of State human rights commission?
(a) 4 Years
(b) 5 Years
(c) 6 Years
(d) Not Prescribed
Correct Answer is B

(5 Years)
178) Which statement/statements is/are correct?
1. Human Right Commission cannot take a case suo motu.
2. Human rights commission's headquarters is located at Mumbai.
3. Human rights commission has powers of a civil court.
Select the correct answer using the codes given below...
(a) 3 only
(b) 2 and 3 only
(c) 1 and 3 only
(d) 1, 2 and 3
Correct Answer is A
(Human Right Commission either suo motu or on a petition can take a
case. Human rights commission's headquarters is located at Delhi.)

179) Which statement/statements is/are correct With respect to State


human rights commission?
1. The chairperson and the members are not eligible for further
employment under the state government.
2. The chairperson, as well as, the members are not eligible for
further employment under the Central government.
3. The chairperson and the members of the state human rights
commission are removed by Governor.
Which of the statements given above is are correct...
(a) only 3
(b) 1 and 2 only
(c) 1, 2 and 3
(d) None
Correct Answer is B
(They are appointed by the governor, but they are removed by the
President.)

180) Which of the following authority determines the salary and


allowances of the chairman of State human rights commission?
(a) Constitution
(b) Central government
(c) State government
(d) President
Correct Answer is C
(State government.)

181) What is the maximum number of information commissioners can


be there apart from Chief Information Commissioner?
(a) 5
(b) 4
(c) 3
(d) 10
Correct Answer is 10
(In the Central information commission, the Chief information
Commissioner and not more than 10 information commissioners are
there.)

182) Consider the following statements:


1. State human rights commission has all the powers of civil court.
2. The proceedings of State human rights commission are of judicial
nature.
3. The State human rights commission can look into the violation of
human rights within one year of its occurrence.
Which of the above statement is/are correct
(a) 1 and 2
(b) 3 only
(c) 1, 2 and 3
(d) None
Correct Answer is C
(The State human rights commission Cannot enquire into the matters If
the one-year is already passed when the violation actually occurred.)

183) Which statement/statements is/are correct?


1. The State human rights commission can recommend the State
government for compensation.
2. The State human rights commission can recommend the State
government for Interim relief.
3. The State human rights commission cannot recommend the State
government to take action against the guilty.
(a) 1 and 2
(b) 2 only
(c) 2 and 3
(d) 3 only

Correct Answer is A
(The State human rights commission can recommend the State
government to take action against the guilty.)

184) Which of the following authority appoints information


commissioners?
(a) Prime Minister
(b) President
(c) Leader of Opposition
(d) Information Commissioner
Correct Answer is B
(President Appoints information commissioners on the recommendation
of the committee which is headed by the Prime Minister. the committee
consist of leader of opposition and the union Cabinet Minister who is
nominated by Prime Minister.)

185) Which statement/statements is/are correct?


1. State human rights commission does not have any power to
punish guilty.
2. State human rights commission does not have any power to
award any relief.
3. The recommendations of State human rights commission are
binding
Select the correct answer using the codes given below.
(a) 1 only
(b) 2 only
(c) 1 and 2 only
(d) 1, 2and 3
Correct Answer is C
(The recommendations of State human rights commission are not
binding.)

186) When The Central information commission got established?


(a) 2005
(b) 2004
(c) 2003
(d) 2001
Correct Answer is A
(2005)
187) What is the tenure of Chief information Commissioner?
(a) Five years or 65 years whichever is earlier
(b) Five years or 65 years whichever is later
(c) Six years or 65 years whichever is earlier
(d) Six years or 65 years whichever is later
Correct Answer is A
(Five years or 65 years whichever is earlier.)

188) Which statement/statements is/are correct?


1. Central Information Commission falls under Ministry of
personnel.
2. UPSC falls under Ministry of personnel.
3. Starts Selection Commission falls under Ministry of personnel
Select the correct answer using the codes given below...
(a) 1 only
(b) 2 and 3 only
(c) 1 and 3 only
(d) 1, 2 and 3
Correct Answer is D
(Even Central Vigilance Commission, falls under Ministry of personnel.
Apart from this, Central bureau of investigation and Central
administrative Tribunal fall under Ministry of personnel.)

189) Which statement/statements is/are correct?


1. Central Information Commission does not have suo moto power
to order an enquiry.
2. Central information commission has the power of civil court in
certain matters.
3. Central information commission can direct a public authority to
appoint Public Information Officer.
Which of the statements given above is are correct...
(a) only 3
(b) 1 and 2 only
(c) 2 and 3
(d) None
Correct Answer is C
(Central Information Commission suo moto power to order an enquiry.)

190) Which of the following authority and remove The Chief


Information Commissioner?
(a) Governor
(b) Prime Minister
(c) Home Minister
(d) President
Correct Answer is D
(President can remove the Chief information Commissioner or any
other Commissioner in the case of insolvency, conviction of offence,
paid employment, being unfit, Proved misbehaviour and capacity .)

191) What is the tenure of state Information Commissioner?


(a) 6 years or until they attain the age of 65 years whichever is
earlier
(b) 5 years or until they attain the age of 70 years whichever is
earlier
(c) 5 years or until they attain the age of 65 years whichever is
earlier
(d) 5 years or until they attain the age of 62 years whichever is
earlier
Correct Answer is C

(5 years or until they attain the age of 65 years whichever is earlier.)


192) Consider the following statements:
1.The Information Commissioner can become a chief information
Commissioner.
2.The total tenure of Commissioner is five years totalling his tenure
as Commissioner and Chief information Commissioner.
3.The commission can impose a penalty of Rs. 25 per day up to a
maximum of 25,000 on Public information officer.
Which of the above statement is/are correct
(a) 1 and 2
(b) 3 only
(c) 1, 2 and 3
(d) None
Correct Answer is A
(The commission can impose a penalty of Rs. 250 per day up to a
maximum of 25,000 on Public information officer.)

193) Which statement/statements is/are correct?


1. State Information Commissioner is appointed by Gov
2. State Information Commissioner can be removed on the grounds
of proved misbehaviour and incapacity.
3. Chief information Commissioner gets a salary and allowances
similar to election Commissioner.
(a) 1 and 2
(b) 2 only
(c) 2 and 3
(d) 3 only
Correct Answer is B
(on the same lines, the salary and allowances of state information
Commissioner are similar to Chief Sec of the State government.)

194) How many state information commissioners can be appointed?


(a) 4
(b) 5
(c) 6
(d) It varies state to state
Correct Answer is D
(It varies state to state.)

195) Which statement/statements is/are correct?


1. State information commission has the power of a civil court.
2. The commission can examine the witnesses or the documents.
3. The commission can also inspect documents.
4. The commission can also direct a public authority to appoint a
public information officer.
Select the correct answer using the codes given below.
(a) 1, 2 and 4 only
(b) 2 only
(c) 1,3 and 4 only
(d) 1, 2, 3 and 4
Correct Answer is D

(All are correct.)


196) Which of the following committee recommended the establishment
of Central Vigilance Commission?
(a) HanumathRao committee
(b) Punchi Committee
(c) Rajmannar committee
(d) Santhanam Committee
Correct Answer is D
(Santhanam Committee)

197) Which of the following is not the part of the committee which
recommends President on the appointment of CVC?
(a) Prime Minister
(b) Leader of Opposition
(c) Union Minister of Home Affairs
(d) Chief Justice of India
Correct Answer is D
(Chief Justice of India)

198) Which statement/statements is/are correct?


1. CVC is a multi-member body.
2. CVC can have a chairperson and not more than two vigilance
commissioners.
3. CVC is appointed by the President.
Select the correct answer using the codes given below...
(a) 1 only
(b) 2 and 3 only
(c) 1 and 3 only
(d) 1, 2 and 3
Correct Answer is D
(All are correct.)

199) Which statement/statements is/are correct?


1. President can remove CVC on the grounds of insolvency.
2. President and remove CVC on the grounds of proved
misbehaviour and incapacity but in such circumstances
president has to refer the matter to the Supreme Court.
3. The salary and allowances of Central vigilance Commissioner are
similar to The member of union public service commission.
Which of the statements given above is are correct...
(a) only 3
(b) 1 and 2 only
(c) 1, 2 and 3
(d) None
Correct Answer is B
(The salary and allowances of Central vigilance Commissioner are
similar to The chairman of union public service commission.)

200) Who of the following can remove Chief Vigilance Commissioner?


(a) Prime Minister
(b) Home Minister
(c) Governor
(d) President
Correct Answer is D
(President)
201) Who acts as a vice-chairman of the
concerned zonal council by rotation?
(a) Home Minister
(b) Prime Minister
(c) Governor
(d) Chief Minister
Correct Answer is D
(Chief Minister)

You can also join telegram channel for latest updates

Telegram Exam Group (For Updates on UPSC)

Telegram UGC NET Channel (UGC Material)

UGC NET Group (To Meet Other Aspirants and Free Test Series)

UPSC NET Group (To Meet Other Aspirants and Free Test Series)

You can also enjoy our Youtube videos on our channel.

Don’t Forget to rate us on Amazon and claim a Free UPSC or UGC


(Any One) test series worth ₹ 1999 on our telegram group given above.

202) Consider the following statements with respect to Chief


minister is/are correct?
1. CM can collapse the council.
2. CM can ask a minister to resign.
3. CM can allocate and reshuffle the portfolio.
Which of the above statement is/are correct
(a) 1 and 2
(b) 3 only
(c) 1, 2 and 3
(d) None
Correct Answer is C

(All are correct.)


203) Which of the following state or states have Minister in charge of
tribal welfare?
1. Chhattisgarh
2. Jharkhand
3. Punjab
(a) 1 and 2
(b) 2 only
(c) 2 and 3
(d) 3 only
Correct Answer is A
(In the states of Chhattisgarh, Jharkhand, Madhya Pradesh and
Odisha, there shall be a Minister in charge of tribal welfare who may be
in charge of the welfare of the scheduled castes and backward classes.)

204) Who is the chairman of the State Planning Board?


(a) Deputy Chief Minister
(b) Speaker
(c) Governor
(d) Chief Minister
Correct Answer is D
(Chief Minister )
205) Which statement/statements is/are correct?
1. The total number of ministers in the council of ministers in a
state shall not exceed 15 % of the total strength of the legislative
assembly of that state.
2. The number of ministers, including the chief minister, in a state
shall not be less than 12. 4
Select the correct answer using the codes given below.
(a) 1 only
(b) 2 only
(c) Both are correct
(d) None of the above
Correct Answer is C
(Both are correct.)

206) Which of the following does not form part of Council of Minister in
State legislature?
(a) Cabinet Minister
(b) Deputy Minister
(c) Minister of state
(d) Principle Secretaries
Correct Answer is D
(Principle Secretaries)

207) Who is the chief spokesman of the state government?


(a) Governor
(b) Deputy Chief Minister
(c) Chief Minister
(d) There is no such person
Correct Answer is C
(Chief Minister)

208) Which statement/statements is/are correct?


1. Council of ministers with the Chief Minister as the head aids and
advise the governor.
2. Ministers shall hold office during the pleasure of the governor.
3. Council of ministers shall be collectively responsible to the
legislative assembly.
Select the correct answer using the codes given below...
(a) 1 only
(b) 2 and 3 only
(c) 1 and 3 only
(d) 1, 2 and 3
Correct Answer is D
(All this is given under Article 164 of the constitution.)

209) Which statement/statements is/are correct?


1. There are tribal welfare minister in Chhattisgarh, Jharkhand,
Madhya Pradesh, Bihar and Odisha.
2. The 94th Amendment Act of 2006 included Bihar from the
obligation of having a tribal welfare minister.
3. There is no schedule area in Bihar.
Which of the statements given above is are correct...
(a) only 3
(b) 1 and 2 only
(c) 1, 2 and 3
(d) None
Correct Answer is A
(There are tribal welfare minister in Chhattisgarh, Jharkhand, Madhya
Pradesh and Odisha. The 94th Amendment Act of 2006 freed Bihar
from the obligation of having a tribal welfare minister.)
210) Which of the following may be included in State Council of
Minister?
(a) Speaker
(b) Governor
(c) Leader of Opposition
(d) Deputy Chief Minister
Correct Answer is B
(At times, the council of ministers may also include a deputy chief
minister. Andhra Pradesh had the office of deputy chief minister till
1956. West Bengal, Rajasthan, Madhya Pradesh and Karnataka have
created this office. The deputy chief ministers are appointed mostly for
local political reasons.)

211) Which of the following first suggested setting up of Lokpal and


Lokyukta?
(a) Administrative Reforms Commission (ARC)
(b) Government of India act 1947
(c) Constitution of India
(d) None of the above
Correct Answer is A
(Administrative Reforms Commission (ARC))

212) Which of the following Institute or institutes work under Central


Vigilance Commission?
1. CVC Secretariat
2. Chief Technical Examiners Wing
3. Commissioners for Departmental Enquiries
Which of the above correctly matched?
(a) 1 and 3
(b) 3 only
(c) 1, 2 and 3
(d) None
Correct Answer is C
(All work under Central Vigilance Commission.)

213) Which of the following is not included in the motto of CBI?


1. Industry
2. Impartiality
3. Intelligence
(a) 1 and 2
(b) 2 only
(c) 1, 2 and 3
(d) 3 only
Correct Answer is A
(It is Integrity, not Intelligence.)

214) Who is the head of the committee which recommends the


appointment of director of CBI?
(a) Prime Minister
(b) Home Minister
(c) President
(d) Chief Vigilance Commissioner
Correct Answer is D
(Chief Vigilance Commissioner)

215) Which statement/statements is/are correct?


1. Special police Establishment is a division of CBI
2. CBI Academy is located at Ghaziabad.
3. CBI comes under the administrative control of Ministry of
Home Affairs.
Select the correct answer using the codes given below.
(a) 1 only
(b) 2 only
(c) 3 only
(d) 1 and 2
Correct Answer is D
(CBI comes under the administrative control of Department of
personnel and training (DoPT).)

216) In which country the institution of ombudsman was first created?


(a) US
(b) Singapore
(c) Sweden
(d) Switzerland
Correct Answer is Sweden
(In 1809, the first institution of ombudsman was created in Sweden. The
word Ombud in Swedish means a person who acts as a spokesman all
representative of another person.)

217) Where does the CBI derive its power?


(a) Constitution
(b) CBI act
(c) From People
(d) Delhi Special Police Establishment Act 1946
Correct Answer is Delhi Special Police Establishment Act 1946

(CBI is not a statutory body. It derives its power from Delhi Special
Police Establishment Act 1946.)

218) Which statement/statements is/are correct?


1. CVC is a statutory body.
2. CBI was set up by a resolution of Ministry of home affairs.
3. CBI was established as per the recommendation of Santhanam
committee
Select the correct answer using the codes given below...
(a) 1 only
(b) 2 and 3 only
(c) 1 and 3 only
(d) 1, 2 and 3
Correct Answer is D
(All are correct.)

219) Which statement/statements is/are correct?


1. CVC has the power of a civil court.
2. CVC presents annually report of its performance to the
Parliament.
3. All the ministries and departments of the union government have
a Chief vigilance officer.
Which of the statements given above is are correct...
(a) only 2
(b) 1 and 2 only
(c) 1 and 3
(d) None
Correct Answer is C

(CVC presents annually report of its performance to the President.)


220) Where the headquarters of CVC is located?
(a) Kolkata
(b) Mumbai
(c) Chennai
(d) New Delhi
Correct Answer is D
(New Delhi)
221) Which of the following constitutional amendment act gave
constitutional status and protection to cooperative societies?
(a) 97th constitutional amendment act 2011
(b) 96th constitutional amendment act 2011
(c) 95th constitutional amendment act 2011
(d) 94th constitutional amendment act 2011
Correct Answer is 97th constitutional amendment act 2011
(97th constitutional amendment act 2011)

222) Consider the following statements:


1. Administrative reform commission kept judiciary out of the
purview of Lokpal.
2. In Sweden, judiciary is also under the purview of Lokpal
3. Administrative reform commission wanted Lokpal to be
appointed by the Prime Minister.
Which of the above correctly matched?
(a) 1 and 2
(b) 3 only
(c) 1, 2 and 3
(d) None
Correct Answer is A
(ARC recommended that the Lokpal will be appointed by President
after the consultation Chief Justice of India, the Speaker of the Lok
Sabha as was the chairman of Rajya Sabha.)

223) Which statement/statements is/are correct?


1. Part XVII deals with the official language.
2. After independence, the President appointed an official language
commission under BG KHER.
3. States are allowed to adopt the major regional language as their
official language.
(a) 1 and 2
(b) 2 only
(c) 1, 2 and 3
(d) 3 only
Correct Answer is C
(It is not mandatory for the states to adopt the language of the eighth
schedule of the Constitution.)

224) Under which Fundamental Right the cooperative society is given?


(a) Article 18
(b) Article 19
(c) Article 21
(d) Article 16
Correct Answer is B
(Article 19)

225) Which statement/statements is/are correct?


1. The cooperative society has one seat reserved for scheduled
castes or scheduled tribe.
2. The cooperative society reserves 2 seats for women on the board
3. The board members of the cooperative Society has a term of
office of five years.
4. The cooperative societies are supposed to file a return within 6
months of completion of financial year.
Select the correct answer using the codes given below.
(a) 1 and 4 only
(b) 2 only
(c) 1,3 and 4 only
(d) 1, 2, 3 and 4
Correct Answer is D
(All are correct.)
226) Under which part of the Constitution cooperative societies are
given ?
(a) Part IX A
(b) Part X
(c) Part IX B
(d) Part X B
Correct Answer is Part IX B
(The cooperative societies are given under part IXB of the Constitution.)

227) What is the maximum number of directors a cooperative society


can have?
(a) 7
(b) 10
(c) 21
(d) 25
Correct Answer is
(A cooperative society can have a maximum of 21 directors.)

228) Which statement/statements is/are correct?


1. VIIIth schedule of the Constitution consist of 25 languages.
2. Originally, there were 14 languages in the VIIIth schedule.
3. Santhali is one of the languages given under VIIIth schedule
Select the correct answer using the codes given below...
(a) 1 only
(b) 2 and 3 only
(c) 1 and 3 only
(d) 1, 2 and 3
Correct Answer is B
(At present (2013), the Eighth Schedule of the Constitution specifies 22
languages, originally there were 14 languages. These are Assamese,
Bengali, Bodo, Dogri (Dongri), Gujarati, Hindi, Kannada, Kashmiri,
Konkani, Maithili, Malayalam, Manipuri, Punjabi, Marathi, Nepali,
Odia, Sindhi, Sanskrit, Santhali, , Tamil, Telugu and Urdu. Sindhi was
added by the 21st Amendment Act of 1967; Konkani, Manipuri and
Nepali were added by the 71st Amendment Act of 1992. Bodo, Dongri,
Maithili and Santhali were added by the 92nd Amendment Act of 2003.)

229) Which statement/statements is/are correct?


1. 96th constitutional amendment act of 2011 substituted Odia for
Oriya.
2. Article 343 talks about the official language of the union.
3. Article 345 refers to the official language of state.
Which of the statements given above is are correct...
(a) only 2
(b) 1 and 2 only
(c) 1, 2 and 3
(d) None
Correct Answer is C
(All are correct.)

230) Under which directive principle of state policies, the cooperative


societies are mentioned?
(a) Article 43-A
(b) Article 43-D
(c) Article 43-B
(d) Article 43-E
Correct Answer is Article 43-B
(Article 43-B says that the State shall provide autonomous functioning
and professional management of cooperative Society.)

231) Under which amendment, The translation of the Constitution in


Hindi language is caused to publish?
(a) 55th Constitutional Amendment Act
(b) 56th Constitutional Amendment Act
(c) 58th Constitutional Amendment Act
(d) 57th Constitutional Amendment Act
Correct Answer is C
(58th Constitutional Amendment Act )

232) Consider the following statements:


1. Administrative Tribunals Act of 1985 empowers the Central
government to establish the State Administrative Tribunals.
2. The chairman of the SATs are appointed by the president after
consultation with the Chief Justice of high court of the state.
3. Article 323 A establishes tribunals for public service matters
only,
Which of the above correctly matched?
(a) 1 and 3
(b) 3 only
(c) 1, 2 and 3
(d) None
Correct Answer is A
(The chairman of the SATs are appointed by the president after
consultation with the governor of the state.)

233) Which statement/statements is/are correct?


1. Union or a state can acquire, hold and dispose property under the
exercise of its executive power.
2. Article 300 declares that the Union of India or states can sue or
be sued.
3. President and the governors cannot be sued during the term of
their office for official act.
(a) 1 and 2
(b) 2 only
(c) 1, 2 and 3
(d) 3 only
Correct Answer is C
( The aggrieved person can bring appropriate proceedings against the
Union of India instead of the president and the state instead of the
Governor of that state.)

234)Under which part of the constitution, special provisions for SC, ST


and other backward classes are contained?
(a) Part XVI
(b) Part XV
(c) Part XX
(d) Part XIV
Correct Answer is Part XVI

(Part XVI)
235) Which statement/statements is/are correct?
1. No criminal proceedings can be started against the president in
respect of their personal.
2. Criminal proceedings can be started against the governors in
respect of their personal acts.
3. President cannot be arrested or imprisoned in respect of their
personal acts.
4. Governor cannot be arrested or imprisoned in respect of their
personal acts.
Select the correct answer using the codes given below.
(a) 1 and 4 only
(b) 2 only
(c) 1,3 and 4 only
(d) 1, 2, 3 and 4
Correct Answer is C
(No criminal proceedings can be started against the governors in respect
of their personal acts.)
236) Under which amendment act, relaxation for qualifying marks is
provided?
(a) 80th Amendment Act
(b) 81st Amendment Act
(c) 82nd Amendment Act
(d) 83rd Amendment Act
Correct Answer is D
(82nd Amendment Act)

237) Which of the following amendment separated the combined


commission of SC and ST?
(a) 89th Amendment Act
(b) 90th Amendment Act
(c) 87th Amendment Act
(d) 88th Amendment Act
Correct Answer is 89th Amendment Act
(The 89th Amendment Act of 2003 separated the combined commission
into two separate bodies.)

238) Which statement/statements is/are correct?


1. Constitution grants any immunity for official acts of the
ministers.
2. Ministers are not liable for any official acts which are done by
president or president on their advice.
3. The ministers do not enjoy any immunity for their personal acts
hence they can be sued for crimes.
Select the correct answer using the codes given below...
(a) 1 only
(b) 2 and 3 only
(c) 1 and 3 only
(d) 1, 2 and 3
Correct Answer is B
(Constitution does not grant any immunity for official acts of the
ministers.)

239) Which statement/statements is/are correct?


1. The judicial officers enjoy immunity from any liability in respect
of their official acts.
2. The Civil servant who has made a contract in his official capacity
is not personally liable.
3. The liability of the civil servants for illegal acts is more than an
ordinary citizen.
Which of the statements given above is are correct...
(a) only 2
(b) 1 and 2 only
(c) 1, 2 and 3
(d) None
Correct Answer is B
(The liability of the civil servants for illegal acts is the same as of any
ordinary citizen.)

240) Under which article, Protection or Immunity is given to Governors


and President?
(a) Article 361
(b) Article 365
(c) Article 360
(d) Article 355
Correct Answer is A
(Article 361)

241) Who of the following was the chief protagonist of all India services?
(a) Mahatma Gandhi
(b) JL Nehru
(c) Lal Bahadur Shastri
(d) Sardar Vallabh bhai patel
Correct Answer is Sardar Vallabh bhai patel
(Sardar Vallabh bhai patel sometimes is also referred as father of all
India services.)

242) Consider the following statements:


1. The members of gazetted class are called officers while the non-
gazetted are called employees.
2. The Persons holding the position the central services are at the
pleasure of the President.
3. Article 311 imposes restrictions on the doctrine of pleasure of
President.
Which of the above correctly matched?
(a) 1 and 3
(b) 3 only
(c) 1, 2 and 3
(d) None
Correct Answer is C
(All are correct.)

243) Which statement/statements is/are correct?


1. A civil servant cannot be dismissed or removed by the authority
subordinate to his or her appointing authority.
2. A civil servant can be removed or dismissed or reduced in rank
without an enquiry.
3. The safeguards in terms of restrictions on doctrine of pleasure
which are provided to civil servants are also extended to defence
services.
(a) 1 only
(b) 2 only
(c) 1, 2 and 3
(d) 3 only
Correct Answer is A
(A civil servant cannot be removed or dismissed or reduced in rank
without an enquiry. The safeguards in terms of restrictions on doctrine
of pleasure which are provided to civil servants are not extended to
defence services.)

244) Which of the following article referenced creation of all India


services?
(a) Article 309
(b) Article 310
(c) Article 311
(d) Article 312
Correct Answer is Article 312
(Article 312)

245) Which statement/statements is/are correct?


1. Parliament has the power to create new all India services.
2.Rajya sabha has to pass a resolution to create new all India
services.
3. Rajya sabha has to pass the resolution supported by two-thirds
the members total members to create a new all India service.
4.Article 314 protected the officers of civil services were appointed
before 1950.
Select the correct answer using the codes given below.
(a) 1 and 4 only
(b) 2 only
(c) 1,2 and 4 only
(d) 1, 2, 3 and 4
Correct Answer is C
(Rajya sabha has to pass the resolution supported by two-thirds the
members present and voting to create a new all India service.)

246) Which of the following article deals with administrative Tribunal?


(a) Article 323 A
(b) Article 313 A
(c) Article 322 A
(d) Article 321 A
Correct Answer is Article 323 A
(We do have article 323 B which deals with tribunal for other matters.)

247) How much fee an applicant has to pay in CAT?


(a) 100
(b) 500
(c) 50
(d) 20
Correct Answer is C
(Rupees 50 is the amount of fee.)

248) Which statement/statements is/are correct?


1. The Chairman of CAT has a tenure up to the age of 70 years or 5
years, whichever is earlier.
2. The Members of CAT has a tenure of 5 years or the age up to 65
years, whichever is earlier.
3. CAT is guided by the principle of natural justice.
Select the correct answer using the codes given below...
(a) 1 only
(b) 2 and 3 only
(c) 1 and 3 only
(d) Only 3
Correct Answer is D
(The Members of CAT has a tenure of 5 years or the age up to 62 years,
whichever is earlier. The Chairman of CAT has a tenure up to the age of
65 years or 5 years, whichever is earlier.)

249) Which statement/statements is/are correct?


1. CAT is a multi-member body.
2. CAT has a Chairman and Vice-Chairman as well.
3. CAT members have the status of judges of the supreme court.
Which of the statements given above is are correct...
(a) only 2
(b) 1 and 2 only
(c) 1, 2 and 3
(d) None
Correct Answer is B
(CAT members have the status of judges of the high court.)

250) Where the principal bench of CAT is located?


(a) Mumbai
(b) Delhi
(c) Kolkata
(d) Chennai
Correct Answer is Delhi
(The first bench of CAT emerged in 1985 in with the principal bench at
Delhi.)
251) To be recognised as a national party, in how
many states it has to be recognised?
(a) 2
(b) 3
(c) 4
(d) 5
Correct Answer is C
(It has to be recognised in four states.)

252) What are the conditions to be recognised as a national party?


1. It wins two per cent of seats in the Lok Sabha at a general
election and these candidates are elected from three states
2. If it secures six per cent of valid votes polled in any four or more
states at a general election to the Lok Sabha.
3. it is recognised as a state party in four states.
Which of the above correctly matched?
(a) 1 and 3
(b) 3 only
(c) 1, 2 and 3
(d) None
Correct Answer is C
(All are correct.)

253) Which of the following is dealt under RPA (Representation of


People Act 1951)?
1. Actual conduct of elections
2. Deals with administrative machinery for conducting elections
3. Election disputes
(a) 1 and 2
(b) 2 only
(c) 1, 2 and 3
(d) 3 only
Correct Answer is C
(All are correct.)

254) Which article provides for an independent Election Commission?


(a) Article 322
(b) Article 323
(c) Article 324
(d) Article 325
Correct Answer is C
(Article 324)

255) Which of the following is provided under RPA (Representation of


People Act 1950)?
1. qualifications of voters
2. preparation of electoral rolls
3. delimitation of constituencies
4. allocation of seats
Select the correct answer using the codes given below.
(a) 1 and 4 only
(b) 2 only
(c) 1,3 and 4 only
(d) 1, 2, 3 and 4
Correct Answer is D
(All are correct.)

256) Which of the following committee is formulated on Nexus between


crime and politics?
(a) Vohra Committee
(b) Tarkunde Committee
(c) Dinesh Goswami Committee
(d) Jaya Prakash Narayan (JP) committee
Correct Answer is Vohra Committee
(Vohra Committee)

257) Which of the following committee is formulated on state funding of


eleciton?
(a) Tarkunde Committee
(b) Indrajit Gupta Committee
(c) Dinesh Goswami Committee
(d) M.N. Venkatachaliah
Correct Answer is B
(Indrajit Gupta Committee.)

258) Which of the following committee is formulated on Electoral


Reforms?
1. Tarkunde Committee
2. Jaya Prakash Narayan (JP) committee
3. Dinesh Goswami Committee
Select the correct answer using the codes given below...
(a) 1 only
(b) 2 and 3 only
(c) 1 and 3 only
(d) 1, 2 and 3
Correct Answer is D
(All are correct.)

259) Which statement/statements is/are correct?


1. EVMs were used for the first time in the elections to the
Assemblies of Rajasthan, Madhya Pradesh and Delhi.
2.EVMs were used for the first time in the general elections to the
Assembly of Goa.
Which of the statements given above is are correct...
(a) only 2
(b) 1 only
(c) Both of the above
(d) None
Correct Answer is C
(Both of the above.)

260) Which of the following committee is formulated on Reviewing of


working of the constitution?
(a) Dinesh Goswami Committee
(b) Vohra Committee
(c) Indrajit Gupta Committee
(d) M.N. Venkatachaliah
Correct Answer is D
(M.N. Venkatachaliah)

261.) The only instance when the President of India exercised his power
of veto related to
the Hindu Code Bill
the PEPSU Appropriation Bill
the Indian Post Office (Amendment) Bill
the Dowry Prohibition Bill
Correct Answer is B
( the PEPSU Appropriation Bill )

262.) The Chief Minister of a State in India is NOT eligible to vote in the
Presidential election if
he himself is a candidate
he is yet to prove his majority on the floor of the Lower House of the
State legislature
he is a member of the Upper House of the State Legislature
he is a caretaker Chief Minister
Correct Answer is C
( he is a member of the Upper House of the State Legislature )

263.) The abolition of the I.A.S. and the I.P.S. has been recommended by
the
Dhebar Commission
Kalekar Commission
Kher Commission
Rajamannar Commission
Correct Answer is D
( Rajamannar Commission )

264.) Among the four pairs given below which one consists of a correct
combination of dignitaries who became Vice-Presidents after having
held diplomatic posts like Ambassadors and High Commissioners ?
Dr. S. Radhakrishnan and G.S. Pathak
Dr. S. Radhakrishnan and V.V. Giri
Dr. Zakir Hussain and K.R. Narayanan
B.D. Jatti and K.R. Narayanan
Correct Answer is C
( Dr. Zakir Hussain and K.R. Narayanan )

265.) Which one of the following statements is correct?


Neither the Finance Commission nor the Planning Commission is a
constitutional body
The scope of the Finance Commission is limited to a review of the
revenue segment of the budget while the Planning Commission takes
an overall review embracing both capital and revenue requirements of
the States
No one can be a member of both the Finance Commission and the
Planning Commission at the same time
There is no overlapping of work and responsibility of the Finance
Commission and those of the Planning Commission.
Correct Answer is B
( The scope of the Finance Commission is limited to a review of the
revenue segment of the budget while the Planning Commission takes an
overall review embracing both capital and revenue requirements of the
States )

266.) The Swaran Singh Committee considered the question of


more autonomy to Punjab on the model of Jammu and Kashmir
the suitability of the Presidential form of government for India
the precedence of the Directive Principles over Fundamental Rights
administrative reforms
Correct Answer is B
( the suitability of the Presidential form of government for India )

267.) Which one of the following is a feature common to both the Indian
Federation and the American Federation ?
A single citizenship
Three lists in the Constitution
Dual judiciary
A federal supreme court to interpret the Constitution
Correct Answer is D
( federal supreme court to interpret the Constitution )

268.) Which one of the following is in the Concurrent List in the


Constitution of India ?
Population control and family planning
Public health and sanitation
Capitation taxes
Treasure trove
Correct Answer is A
( Population control and family planning )

269.) Which one of the following statements regarding the office of the
Speaker is correct ?
He holds office during the pleasure of the President
He need not be a member of the House at the time of his election but has
to become a member of the House within six months from the date of
his election.
He loses his office if the House is dissolved before the end of its normal
tenure
If he intends to resign the letter of his resignation is to be addressed to the
Deputy Speaker
Correct Answer is D
( If he intends to resign the letter of his resignation is to be addressed to
the Deputy Speaker)

270.) Which one of the following comes under the jurisdiction of both
the High Court and the Supreme Court ?
Disputes between the Centre and the States
Disputes between the States inter se
Protection of the Fundamental Rights
Protection against the violation of the Constitution
Correct Answer is C
( Protection of the Fundamental Rights )

271) The primary function of the Finance Commission in India is to


(a) distribute revenue between the Centre and the States
(b) prepare the Annual Budget
(c) advise the President on financial matters
(d) allocate funds to various ministries of the Union and State
Governments

Correct Answer is A
( distribute revenue between the Centre and the States )

272) The state which has the largest number of seats reserved for the
Scheduled Tribes in the Lok Sabha is
(a) Bihar
(b) Gujarat
(c) Uttar Pradesh
(d) Madhya Pradesh
Correct Answer is D
( Madhya Pradesh )

273) 6. A college student desires to get elected to the Municipal Council


of his city. The validity of his nomination would depend on the
important condition, among others, that
(a) he obtains permission from the Principal of his college
(b) he is a member of a political party
(c) his name figures in the Voters’ List
(d) he files a declaration owing allegiance to the Constitution of
India

Correct Answer is C
( his name figures in the Voters’ List )

274) Which one of the following statements is incorrect ?


(a) Goa attained full statehood in 1987
(b) Diu is an island in the Gulf of Khambhat
(c) Daman and Diu were separated from Goa by the 56th
Amendment of the Constitution of India
(d) Dadra and Nagar Haweli were under French colonial rule till
1954.

Correct Answer is D
( Dadra and Nagar Haweli were under French colonial rule till 1954. )

275) The Parliament can make any law for the whole or any part of
India for implementing International treaties
(a) with the consent of all the States
(b) with the consent of the majority of States
(c) with the consent of the States concerned
(d) without the consent of any State

Correct Answer is D
( without the consent of any State )

276) Which one of the following is NOT a feature of the Government of


India Act of 1935 ?
(a) Diarchy at the Centre as well as in the provinces
(b) A bicameral legislature
(c) Provincial autonomy
(d) An All-India Federation

Correct Answer is A
( Diarchy at the Centre as well as in the provinces )

277) Which one of the following statements about a Money Bill is not
correct ?
(a) A Money Bill can be tabled in either House of Parliament
(b) The Speaker of Lok Sabha is the final authority to decide
whether a Bill is a Money Bill or not
(c) The Rajya Sabha must return a Money Bill passed by the Lok
Sabha and send it for consideration within 14 days
(d) The President cannot return a Money Bill to the Lok Sabha for
reconsideration
Correct Answer is A
( A Money Bill can be tabled in either House of Parliament )

278) The 73rd Constitution Amendment Act 1992 refers to the


(a) generation of gainful employment for the unemployed and the
under-employed men and women in rural area.
(b) generation of employment for the able bodies adults who are in
need and desirous of work during the lean agricultural
season.
(c) laying the foundation for strong and vibrant Panchayati Raj
Institutions in the country.
(d) guarantee of right to life, liberty and security of person, equality
before law and equal protection without discrimination.

Correct Answer is C
( laying the foundation for strong and vibrant Panchayati Raj
Institutions in the country. )

279) Which Article of the Constitution provides that it shall be the


endeavour of every state to provide adequate facility for instruction in
the mother tongue at the primary stage of education ?
(a) Article 349
(b) Articld 350
(c) Article 350 A
(d) Article 351

Correct Answer is C
( Article 350 A )
280) The Supreme Court of India tenders advice to the President on a
matter of law or fact
(a) on its own initiative
(b) only if he seeks such advice
(c) only if the matter relates to the Fundamental Rights of citizens
(d) only if the issue poses a threat to the unity and integrity of the
country

Correct Answer is B
( only if he seeks such advice )

281.) The concept of Public Interest Litigation originated in


(a) the United Kingdom
(b) Australia
(c) the United States
(d) Canada
Correct Answer is C
( the United States )

282.) If the Prime Minister of India belonged to the Upper House of


Parliament
(a) he will not be able to vote in his favour in the event of a no-
confidence motion
(b) he will not be able to speak on the budget in the Lower House
(c) he can make statements only in the Upper House
(d) he was to become a member of the Lower House within six
months after being sworn in as the Prime Minister

Correct Answer is A
( he will not be able to vote in his favour in the event of a no-confidence
motion )
283.) Given below are two statements, one labelled as Assertion (A) and
the other labelled as Reason (R):
Assertion (A): The reservation of thirty-three per cent of seats for women in
Parliament and State legislatures does not require Constitutional amendment.
Reason (R): Political parties contesting elections can allocate thirty-three per
cent of seats they contest to women candidates without any Constitutional
amendment.
In the context of the above two statements, which one of the following is
correct ?
(a) Both A and R are true and R is the correct explanation of A
(b) Both A and R are true, but R is not a correct explanation of A
(c) A is true, but R is false
(d) A is false, but R is true
Correct Answer is D
( A is false, but R is true )

284.) Which one of the following schedules of the Constitution of India


contains provisions regarding anti-defection Act ?
(a) Second Schedule
(b) Fifth Schedule
(c) Eighth Schedule
(d) Tenth Schedule
Correct Answer is D
( Tenth Schedule )

285.) The Indian parliamentary system is different from the British


parliamentary system in that India has
(a) both a real and a nominal executive
(b) a system of collective responsibility
(c) bicameral legislature
(d) the system of judicial review
Correct Answer is D
( the system of judicial review )

286.) Economic Survey in India is published officially, every year by the


(a) Reserve Bank of India
(b) Planning Commission of India
(c) Ministry of Finance, Govt. of India
(d) Ministry of Industries, Govt. of India
Correct Answer is C
( Ministry of Finance, Govt. of India )

287.) Which of the following Parties were not a part of the United Front
which was in power during 1996-97 ?
1. Bahujana Samaj Party.
2. Samata Party
3. Haryana Vikas Party
4. Asom Gana Parishad
Select the correct answer using the codes given below:
Codes:
(a) 1, 2, 3 and 4
(b) 1, 2 and 3
(c) 3 and 4
(d) 1 and 2
Correct Answer is B
( 1, 2 and 3 )

288.) Which one of the following statements is correct?


(a) Kacchativu and Tin Bigha were territories acquired by the Indian
Republic from the French
(b) Kacchativu and Tin Bigha are territories handed over to Sri
Lankan and Bangladeshi sovereignty respectively by the
Government of India
(c) Kacchativu and Tin Bigha are areas that were annexed by the
Chinese in the 1962 Sino-Indian War
(d) Kacchativu and Tin Bigha are enclaves which are transferred to
India by lease arrangements with Sri Lanka and Pakistan
respectively.

Correct Answer is B
( Kacchativu and Tin Bigha are territories handed over to Sri Lankan
and Bangladeshi sovereignty respectively by the Government of India )

289.) A British citizen staying in India cannot claim Right to


(a) Freedom of trade and profession
(b) Equality before the law
(c) Protection of life and personal liberty
(d) Freedom of religion
Correct Answer is A
( Freedom of trade and profession )

290.) Panchayat Raj was first introduced in India in October, 1959 in


(a) Rajasthan
(b) Tamil Nadu
(c) Kerala
(d) Karnataka
Correct Answer is A
( Rajasthan )

291.) If the number of seats allocated to a state in the Lok Sabha is 42,
then the number of seats reserved for the Scheduled Castes in that state
will be
(a) 21
(b) 14
(c) 7
(d) 6

Correct Answer is D (6)

292.) The power of the Supreme Court of India to decide disputes


between the Centre and the States falls under its
(a) advisory jurisdiction
(b) appellate jurisdiction
(c) original jurisdiction
(d) constitutional jurisdiction
Correct Answer is C
( original jurisdiction )

293.) Given below are two statements, one labelled as Assertion (A) and
the other labelled as Reason (R):
Assertion (A): The British sovereignty continued to exist in free India.
Reason (R): The British sovereign appointed the last Governor General of
free India.
In the context of the above two statements, which one of the following is
correct ?
(a) Both A and R are true and R is the correct explanation of A
(b) Both A and R are true, but R is not a correct explanation of A
(c) A is true, but R is false
(d) A is false, but R is true
Correct Answer is D
( A is false, but R is true )

294.) When the Chief Justice of a High Court acts in an administrative


capacity, he is subject to
(a) the writ jurisdiction of any of the other judges of the High Court
(b) special control exercised by the Chief Justice of India
(c) discretionary powers of the Governor of the state
(d) special powers provided to the Chief Minister in this regard
Correct Answer is B
( special control exercised by the Chief Justice of India )

295.) According to the Constitution of India, the term ‘District Judge’


shall not include
(a) chief presidency magistrate
(b) sessions judge
(c) tribunal judge
(d) chief judge of a small cause court
Correct Answer is C
( tribunal judge )

296.) Which one of the following is part of the electoral college for the
election of the President of India but does not form part of the forum for
his impeachment?
(a) Lok Sabha
(b) Rajya Sabha
(c) State Legislative Councils
(d) State Legislative Assemblies
Correct Answer is D
( State Legislative Assemblies )

297.) Which one of the following statements is correct?


The prime minister of India
(a) is free to choose his ministers only from among those who are
members of either House of the Parliament
(b) can choose his cabinet colleagues after due counselling by the
President of India in this regard
(c) has full discretion in the choice of persons who are to serve as
ministers in his cabinet
d) has only limited powers in the choice of his cabinet colleagues
because of the discretionary powers vested with the President of
India

Correct Answer is C
( has full discretion in the choice of persons who are to serve as
ministers in his cabinet )

298.) Consider the following statements: No one can be compelled to


sing the National Anthem since
1. it will be violative of the Right to freedom of speech and
expression.
2. it will be violative of the Right to freedom of conscience and
practice and propagation of religion.
3. there is no provision obliging anyone to sing the National
Anthem. Of these statements:
(a) 1 and 2 are correct
(b) 2 and 3 are correct
(c) 1, 2 and 3 are correct
(d) none is correct
Correct Answer is C
( 1, 2 and 3 are correct )

299.) Corporation tax


(a) is levied and appropriated by the States
(b) is levied by the Union and collected and appropriated by the
States
(c) is levied by the Union and shared by the Union and the States
(d) is levied by the Union and belongs to it exclusively
Correct Answer is D
( is levied by the Union and belongs to it exclusively )
300.) Agricultural income tax is assigned to the State governments by
(a) the Finance Commission
(b) the National Development Council
(c) the Inter-State Council
(d) the Constitution of India
Correct Answer is D
( the Constitution of India )
301.) Consider the following statements regarding
the National Human Rights Commission of India:
1. Its Chairman must be a retired Chief Justice of India
2. It has formations in each state as State Human Rights
Commission
3. Its powers are only recommendatory in nature
4. It is mandatory to appoint a woman as a member of the
Commission
Which of the above statements are correct ?
(a) 1, 2, 3 and 4
(b) 2 and 4
(c) 2 and 3
(d) 1 and 3
Correct Answer is D
( 1 and 3 )

302.) The most short-lived of all of Britain’s constitutional experiments


in India was the
(a) Indian Councils Act of 1861
(b) Indian Councils Act of 1892
(c) Indian Councils Act of 1909
(d) Government of India Act of 1919
Correct Answer is C
( Indian Councils Act of 1909 )

303.) The Constitution of India recognises


(a) only religious minorities
(b) only linguistic minorities
(c) religious and linguistic minorities
(d) religious, linguistic and ethnic minorities
Correct Answer is C
( religious and linguistic minorities )

304.) In the new Panchayati Raj Bill enacted in 1993, there are several
fresh provisions deviating from the past. Which one of the following is
not one such provision?
(a) A number of added responsibilities in the area of agriculture,
rural development, primary education and social forestry among
others.
(b) Elections being made mandatory for all posts at the time they
are due
(c) A statutory representation for women in the panchayats, up to a
third of the strength
(d) Regular remuneration to the panchayat members, so as to ensure
their punctuality and accountability.
<
Correct Answer is D
( Regular remuneration to the panchayat members, so as to ensure their
punctuality and accountability. )

305.) Consider the following statements :


An amendment to the Constitution of India can be initiated by the
1. Lok Sabha
2. Rajya Sabha
3. State Legislatures
4. President
Which of the above statements is/are correct ?
(a) 1 alone
(b) 1, 2 and 3
(c) 2, 3 and 4
(d) 1 and 2
Correct Answer is D
( 1 and 2 )

306.) Consider the following statements about the recent amendments to


the Election Law by the Representation of the People (Amendment) Act
1996:
1. Any conviction for the offence of insulting the Indian National
Flag or the Constitution of India shall entail disqualification for
contesting elections to Parliament and State Legislatures for six
years from the date of conviction.
2. There is an increase in the security deposit which a candidate has
to make to contest the election to the Lok Sabha.
3. A candidate cannot now stand for election from more than one
Parliamentary constituency.
4. No election will now be countermanded on the death of a
contesting candidate.
Which of the above statements are correct ?
(a) 2 and 3
(b) 1, 2 and 4
(c) 1 and 3
(d) 1, 2, 3 and 4
Correct Answer is B
( 1, 2 and 4 )

307.) Which one of the following statements regarding the levying,


collecting and distribution of Income Tax is correct?
(a) The Union levies, collects and distributes the proceeds of
income tax between itself and the states
(b) The Union levies, collects and keeps all the proceeds of income
tax to itself
(c) The Union levies and collects the tax but all the proceeds are
distributed among the states
(d) Only the surcharge levied on income tax is shared between the
Union and the states
Correct Answer is A
(The Union levies, collects and distributes the proceeds of income tax
between itself and the states)

308.) The Speaker can ask a member of the House to stop speaking and
let another member speak. This phenomenon is known as
(a) decorum
(b) crossing the floor
(c) interpellation
(d) yielding the floor
Correct Answer is D

( yielding the floor )

309.) Consider the following statements about the Attorney General of


India:
1. He is appointed by the President of India.
2. He must have the same qualifications as are required for a Judge
of the Supreme Court.
3. He must be a member of either House of Parliament.
4. He can be removed by impeachment by Parliament.
Which of these statements are correct ?
(a) 1 and 2
(b) 1 and 3
(c) 2, 3 and 4
(d) 3 and 4
Correct Answer is A
( 1 and 2 )

310.) Consider the following functionaries :


1. Cabinet Secretary
2. Chief Election Commissioner
3. Union Cabinet Ministers
4. Chief Justice of India
Their correct sequence, in the Order of Precedence is
(a) 3, 4, 2, 1
(b) 4, 3, 1, 2
(c) 4, 3, 2, 1
(d) 3, 4, 1, 2
Correct Answer is C ( 4, 3, 2, 1 )

311.) Which one of the following duties is NOT performed by the


Comptroller and Auditor General of India ?
(a) To audit and report on all expenditure from the Consolidated
Fund of India
(b) To audit and report on all expenditure from the Contingency
Funds and Public Accounts
(c) To audit and report on all trading, manufacturing, profit and loss
accounts
(d) To control the receipt and issue of public money, and to ensure
that the public revenue is lodged in the exchequer

Correct Answer is D
( To control the receipt and issue of public money, and to ensure that the
public revenue is lodged in the exchequer)

312.)Which one of the following statements correctly describes the


Fourth Schedule of the Constitution of India?
(a) It lists the distribution of powers between the Union and the
states.
(b) It contains the languages listed in the Constitution.
(c) It contains the provisions regarding the administration of tribal
areas.
(d) It allocates seats in the Council of States.
Correct Answer is D
( It allocates seats in the Council of States.)

313.) In what way does the Indian Parliament exercise control over the
administration ?
(a) Through Parliamentary Committees
(b) Through Consultative Committees of various ministries
(c) By making the administrators send periodic reports
(d) By compelling the executive to issue writs
Correct Answer is A
( ----- )

314.) Consider the following statements about the minorities in India:


1. The Government of India has notified five communities, namely,
Muslims, Sikhs, Christians, Buddhists and Zoroastrians as
Minorities.
2. The National Commission for Minorities was given statutory
status in 1993.
3. The smallest religious minority in India are the Zoroastrians.
4. The Constitution of India recognizes and protects religious and
linguistic minorities.
Which of these statements are correct ?
(a) 2 and 3
(b) 1 and 4
(c) 2, 3 and 4
(d) 1, 2 and 4
Correct Answer is D
( 1, 2 and 4)
315.) In which one of the following areas does the State Government
NOT have control over its local bodies?
(a) Citizens’ grievances
(b) Financial
(c) Legislation
(d) Personnel matters

Correct Answer is A
( Citizens’ grievances)

316.) Consider the following statements regarding the High Courts in


India:
1) There are eighteen High Courts in the country.
2) Three of them have jurisdiction over more than one state.
3) No Union Territory has a High Court of its own.
4) Judges of the High Court hold office till the age of 62.
Which of these statements is/are correct ?
(a) 1, 2 and 4
(b) 2 and 3
(c) 1 and 4
(d) 4 only
Correct Answer is A
( 1, 2 and 4 )

317.) Match List I with List II and select the correct answer by using the
codes given below the lists :

List-I List II
Amendments to the
Contents
Constitution
A) The Constitution (Sixty- 1) Establishment of state level Rent
ninth Amendment) Act, 1991 Tribunals
B) The Constitution (Seventy- 2) No reservations for Scheduled Castes
fifth Amendment) Act, 1994 in Panchayats in Arunachal Pradesh
C) The Constitution (Eightieth 3)Constitution of Panchayats in Villages
Amendment) Act, 2000 or at other local levels
D) The Constitution (Eighty- 4) Accepting the recommendations of the
third Amendment) Act, 2000 Tenth Finance Commission
5) According the status of National
Capital Territory to Delhi

A B C D
(a) 5 1 4 2
(b) 1 5 3 4
(c) 5 1 3 4
(d) 1 5 4 2
Correct Answer is A ( )

318.) If a new state of the Indian Union is to be created, which one of the
following schedules of the Constitution must be amended ?
(a) First
(b) Second
(c) Third
(d) Fifth

Correct Answer is A ( First)


319.) Consider the following statements regarding the political parties in
India:
1) The Representation of the People Act, 1951 provides for the
registration of political parties.
2) Registration of political parties is carried out by the Election
Commission.
3) A national level political party is one which is recognised in four
or more states.
4) During the 1999 general elections, there were six national and 48
state level parties recognised by the Election Commission.
Which of these statements are correct ?
(a) 1, 2 and 4
(b) 1 and 3
(c) 2 and 4
(d) 1, 2, 3 and 4
Correct Answer is D
( 1, 2, 3 and 4)

320.) Match List I with List II and select the correct answer by using the
codes given below the lists :

List-I List II
Articles of the Content
Constitution
A) Article 54 1) Election of the President of India
2)Appointment of the Prime Minister And Council of
B) Article 75 Ministers
C) Article 155 3) Appointment of the Governor of a state
4) Appointment of the Chief Minister and Council of
D) Article 164 Ministers of a state
5) Composition of Leg islative Assemblies

A B C D
(a) 1 2 3 4
(b) 1 2 4 5
(c) 2 1 3 5
(d) 2 1 4 3
Correct Answer is A ( )

321.) In the Presidential election in India, every elected member of the


Legislative Assembly of a State shall have as many votes as there are
multiples of one thousand in the quotient obtained by dividing the
population of the State by the total number of the elected members of
the Assembly. As at present (1997) the expression “Population” here
means the population as ascertained by the
(a) 1991 Census
(b) 1981 Census
(c) 1971 Census
(d) 1961 Census
Correct Answer is C
( 1971 Census )

322.) Which of the following are/is stated in the Constitution of India ?


1. The President shall not be a member of either Houses of
Parliament.
2. The Parliament shall consist of the President and two Houses.
Choose the correct answer from the codes given below:
(a) Neither 1 nor 2
(b) Both 1 and 2
(c) 1 alone
(d) 2 alone
Correct Answer is B
( Both 1 and 2 )

323.) Match List I with List II and select the correct answer by using the
codes given below the lists :
List - I (Functionaries) List - II (Oaths or affirmations)
A President of India 1 Secrecy of Informatio
Judges of the Supreme
B 2 Faithful Discharge of Duties
Court
Members of Faith and Allegiance to the
C 3
Parliament Constitution of India
Ministers for the Upholding the Constitution and
D 4
Union the law

A B C D
a) 3 4 1 2
b) 4 3 2 1
c) 3 4 2 1
d) 4 3 1 2
Correct Answer is B
( ---------- )

324.) In the following quotation,


“WE, THE PEOPLE OF INDIA,
having solemnly resolved to constitute India into
a Sovereign Socialist Secular
Democratic Republic and to secure to all its citizens:
JUSTICE, social, economic and political;
LIBERTY of thought, expression, belief, faith and worship;
EQUALITY of status and of opportunity;
and to promote among them all
FRATERNITY assuring the dignity of the
individual and the unity and integrity of the Nation;
In our Constituent Assembly this ‘X’ – do hereby adopt, enact and give
to ourselves this Constitution”, ‘X’ stands for
(a) twenty-sixth day of January, 1950
(b) twenty-sixth day of November, 1949
(c) twenty-sixth day of January, 1949
(d) None of the above
Correct Answer is B
( twenty-sixth day of November, 1949 )

325.) Given below are two statements, one labelled as Assertion (A) and
the other labeled as Reason (R):
Assertion (A): Wilful disobedience or non-compliance of Court orders and
use of derogatory language about judicial behaviour amounts to Contempt of
Court.
Reason (R): Judicial activism cannot be practiced without arming the
judiciary with punitive powers to punish contemptuous behaviour.
In the context of the above two statements, which one of the following is
correct ?
(a) Both A and R are true and R is the correct explanation of A
(b) Both A and R are true, but R is not a correct explanation of A
(c) A is true, but R is false
(d) A is false, but R is true
Correct Answer is B
( Both A and R are true, but R is not a correct explanation of A )

326.) The Dinesh Goswami Committee recommended


(a) the constitution of state level election commissions
(b) List system of election to the Lok Sabha
(c) governmental funding of parliamentary elections
(d) a ban on the candidature of independent candidates in the
parliamentary elections
Correct Answer is C
( governmental funding of parliamentary elections )

327.) Which one of the following is NOT a principle of “Panchsheel” ?


(a) Non-alignment
(b) Peaceful Co-existence
(c) Mutual respect for each other’s territorial integrity and
sovereignty
(d) Mutual non-interference in each other’s internal affairs
Correct Answer is A
( Non-alignment )

328.) Which one of the following was NOT proposed by the 73rd
Constitutional Amendment in the area of Panchayati Raj ?
(a) Thirty per cent seats in all elected rural local bodies will be
reserved for women candidates at all levels
(b) The States will constitute their Finance commissions to allocate
resources to Panchayati Raj institutions
(c) The Panchayati Raj elected functionaries will be disqualified to
hold their offices if they have more than two children
(d) The elections will be held in six months time if Panchayati Raj
bodies are superseded or dissolved by the State government

Correct Answer is C
( The Panchayati Raj elected functionaries will be disqualified to hold
their offices if they have more than two children )

329.) Proportional representation is NOT necessary in a country where


(a) there are no reserved constituencies
(b) a two-party system has developed
(c) the first-past-post system prevails
(d) there is a fusion of Presidential and Parliamentary forms of
government

Correct Answer is B
( a two-party system has developed )
330.) Given below are two statements, one labelled
as Assertion (A) and the other labelled as Reason
(R):
Assertion (A): The world ‘minority’ is not defined in the Constitution of
India.
Reason (R): The Minorities Commission is not a constitutional body.
In the context of the above two statements, which one of the following is
correct ?
(a) Both A and R are true and R is the correct explanation of A
(b) Both A and R are true, but R is not a correct explanation of A
(c) A is true, but R is false
(d) A is false, but R is true
Correct Answer is B
( Both A and R are true, but R is not a correct explanation of A )

You can join our FREE test series at NetMock.com

You can also join telegram channel for latest updates

Telegram Exam Group (For Updates on UPSC)

Telegram UGC NET Channel (UGC Material)

UGC NET Group (To Meet Other Aspirants and Free Test Series)

UPSC NET Group (To Meet Other Aspirants and Free Test
Series)
You can also enjoy our Youtube videos on our channel.

Don’t Forget to rate us on Amazon and claim a Free UPSC or UGC


(Any One) test series worth ₹ 1999 on our telegram group given
above.

You might also like